Вы находитесь на странице: 1из 620
2125/2014 6:24.27 PM Mark this question => Question Id : 9808 Question 1 of 30 ‘A 6A-gear-old smoker has chronic cough, hemoptysis and weight loss His chest x-ray shows a cavitating lesion. What is the lcely diagnosis? a) Adenocarcinoma ) Squamous cell carcinoma 6) alveolar cell carcinoma © large cel carcinema ) stnall cell carcinctna Anower [UEARIRHEN) Otver User's Explanai Question Explanation: Squamous cell carcinomas characteristcally present with cavitating lung lesions on Chest X-Ray and metastasize late, Other causes of cavitaiing lung lesions include infection (Staphylococcus aurcus, tabsroulosis, Klebsiella, Pneumocystis carinii), pulmonary infarcts, ‘Wegener's Granalomatosis and Rheumatoid nodules. Report An Error 2125/2014 6:24.27 PM Mark this question => Question Id : 9808 Question 1 of 30 ‘A 6A-gear-old smoker has chronic cough, hemoptysis and weight loss His chest x-ray shows a cavitating lesion. What is the lcely diagnosis? a) Adenocarcinoma Y © b) Squamous cell carcinoma 6) alveolar cell carcinoma © large cel carcinema ) stnall cell carcinctna Anower [UEARIRHEN) Otver User's Explanai Question Explanation: Squamous cell carcinomas characteristcally present with cavitating lung lesions on Chest X-Ray and metastasize late, Other causes of cavitaiing lung lesions include infection (Staphylococcus aurcus, tabsroulosis, Klebsiella, Pneumocystis carinii), pulmonary infarcts, ‘Wegener's Granalomatosis and Rheumatoid nodules. Report An Error 2125/2014 6:24:42 PM “Mark this question >= Question Id : 10505 Question 2 of 30 A 59-year-old chronic smoker presents with weight loss and hemoptysis. Examination reveals that he is clubbed and has clinical evidence of right pleural effusion. His serum calcium is 12.8 mg/dL. A bone scanis normal, Which of the following type of lung cancer ishe most likely to suffer from? a) Adenocercinoma 6) Squamous cell carcinoma ) Large cell carcinoma 4) Mesothelioma @) Small cell carcinoma Question Explanation Hypercalcemia in absence of bony metastases occurs in about 15% of squamous cell lang carcinoma from parathyroid hormone related protein (PTHLP) production Thia is a feature of non-metastatic manifestation of malignancy. Tnappropriate antidiaretic hormone (ADH) secretion (hyponatremia) and ectopic acrenororticotropic hommone (ACTH) production (Cushing's syndrome) occur with smal cell lung cancer. Clubbing is predominantly associated with squamous cell cancers and occasionally adenocarcinoma, 2125/2014 6:24:42 PM “Mark this question >= Question Id : 10505 Question 2 of 30 A 59-year-old chronic smoker presents with weight loss and hemoptysis. Examination reveals that he s clubbed and has clinical evidence of right pleural effusion. His serum calcium is 12.8 mg/dL. A bone scanis normal, Which of the following type of lung cancer ishe most likely to suffer rem? a) Adenocarcinoma SY @ b) Squamous cell carcinoma ) Large cell carcinoma ) Mesothelioma ©) Small cell carcinoma Question Explanation Hypercalcemia in absence of bony metastases occurs in about 15% of squamous cell lang carcinoma from parathyroid hormone related protein (PTHLP) production Thia is a feature of non-metastatic manifestation of malignancy. Tnappropriate antidiaretic hormone (ADH) secretion (hyponatremia) and ectopic acrenororticotropic hommone (ACTH) production (Cushing's syndrome) occur with smal cell lung cancer. Clubbing is predominantly associated with squamous cell cancers and occasionally adenocarcinoma, 2/25/2014 6:24:56 PM Mark this question & => Question Td : 10541 Question 3 of 30 A 65-year-old man presents with hematuria. Cystoscopy discovers a transitional call carcinoma of the bladder, Occupational exposure to which of the following is a recognized risk factor for bladder cancer? a) Aflatoxin ) Beryliunn ©) Chimney sweeps 6) Aniline dye ©) Strongyloides Stercoralis, Anewer [UBER] omer users Explanation Report An Error Question Explanation: Smoking exposure to alanine dyes in the printing ond textile industry, rubber manufacture, cyclophosphamide and schistosomiasis are risk factors for bladder cancer. 2/25/2014 6:24:56 PM Mark this question & => Question Td : 10541 Question 3 of 30 A 65-year-old man presents with hematuria. Cystoscopy discovers a transitional call carcinoma of the bladder, Occupational exposure to which of the following is a recognized risk factor for bladder cancer? a) Aflatoxin ») Beryllium ©) Chimney sweeps VY © 4) Aniiine dye 6) Strongyleides Stercoralis Anewer [UBER] omer users Explanation Report An Error Question Explanation: Smoking exposure to alanine dyes in the printing ond textile industry, rubber manufacture, cyclophosphamide and schistosomiasis are risk factors for bladder cancer. 2125/2014 6:25:16 PM “Mark this question @& => Question Id : 13455 Question 4 of 30 A 63-year-old man with non-stnell cell lung cancer is being considered for surgical resection, Which of the following would best be regerded as a contraindication to surgery? a) FEV1 1.191, b) FEV] LISL ©) FEV12.051, FEVI255L 8) Previous history of Myocardial Infarction Question Explanation: Non-smell cel ung cancer (NSCLC) is 2 heterogeneous aggregate of at least 3 distinct istologies of Ling cancer including epidermoid or squamous carcinoma, adenocarcinoma, and large cel carcinoma. These histologies are often classified together ‘because, when localized, all have the potential for cure with surgical resection. Resection should be performed in the absence of contraindicalions, namely, evidence of spread beyond the lung, endobronchial location of tumor too close to the trachea, and other serious conditions such as poor pulmonary fimction. ‘As a general guideline, most patients with a preoperative forced expiration volume in one second of greater than 2.5 L are able to tolerate pneumonectoray. With a forced expiratory volume in one second of lese then 1.5L, patients are not considered candidates for surgery. “The British Thoracic Society published guidelines on selection of patients for hng cancer surgery in Thorax in 2001 n Report An Error 2125/2014 6:25:16 PM “Mark this question @& => Question Id : 13455 Question 4 of 30 A 63-year-old man with non-small cell ing cancer is being considered for surgical resection. Which of the following would best be regarded as a contraindication to surgery? Y © a FEVI119L b) FEVILISL 6) FEV1 2.051. QFEVI255L ©) Frevions history of Myorardial Infarction Question Explanation: Non-smell cel ung cancer (NSCLC) is 2 heterogeneous aggregate of at least 3 distinct istologies of Ling cancer including epidermoid or squamous carcinoma, adenocarcinoma, and large cel carcinoma. These histologies are often classified together ‘because, when localized, all have the potential for cure with surgical resection. Resection should be performed in the absence of contraindicalions, namely, evidence of spread beyond the lung, endobronchial location of tumor too close to the trachea, and other serious conditions such as poor pulmonary fimction. ‘As a general guideline, most patients with a preoperative forced expiration volume in one second of greater than 2.5 L are able to tolerate pneumonectoray. With a forced expiratory volume in one second of lese then 1.5L, patients are not considered candidates for surgery. “The British Thoracic Society published guidelines on selection of patients for hng cancer surgery in Thorax in 2001 n Report An Error 2125/2014 6:25:32 PM Mark this question & => Question Td : 13532 Question 5 of 30 A 56-year-old man from Bastern China preserts has painfil right upper quadrant mass He reported that he had been diagnosed with viral hepatitis several years previously. Investigatons showed: Serum o-fetoprotein 13.450 IUIL (< 10) What is the most likely underlving viral infection? a) Hepatitis A virus 'b) Hepattis B vims c) Hepatitis C virus 4) Hepatitis D virus ¢) Hepatitis E viras Answer | Explanation | Other User's Explanation Report An Error Question Explanatiot (Very cifficut! The patient has chronic viral hepatiis and presents with a hepatoma, The underlying cause must be ether HBV or ICV. There is a higher prevalence of HBV in the Farin the Far East, and since his couniry of ongin is the only other detail that gives clue to the cause of his hepatitis. The most likely viral agent is HBV. 2125/2014 6:25:32 PM Mark this question & => Question Td : 13532 Question 5 of 30 A 56-year-old man from Bastern China preserts has painfil right upper quadrant mass He reported that he had been diagnosed with viral hepatitis several years previously. Investigatons showed: Serum o-fetoprotein 13.450 IUIL (< 10) What is the most likely underlving viral infection? a) Hepatitis A vims JV © b) Hepattis B viras c) Hepattis C wins 4) HepatitieD virus ©) Hepatitis E virus Answer | Explanation | Other User's Explanation Report An Error Question Explanatiot (Very cifficut! The patient has chronic viral hepatiis and presents with a hepatoma, The underlying cause must be ether HBV or ICV. There is a higher prevalence of HBV in the Farin the Far East, and since his couniry of ongin is the only other detail that gives clue to the cause of his hepatitis. The most likely viral agent is HBV. ‘Mark this question & => Question 6 of 30 In asbestos related disorders which of the following statements are correct? 2) Basal fibrotic shadowing on CXR suggests coincidental idiopathic Sbrosing alveoliis ) Pleural effusion develops more than 20 years after causative asbestos exposure ©) Pleural plaques are recognized precursors of Mescthelioma 46) Increased incidence of primary lang cancer ©) The risk of malignant mesothelioma is greatly increased in smokers compared with non-smokers fy Increased risk for TB Question Explanation: The risk of mesothelioms is net affected by smoking but smoking and asbestos exposure greatly increases the risk of lung cancer. Tis pleural plaques which do not become apparent until 20 years or more after exposure. Pleural effusion may result from acute asbestos pleurisy. Pleural plaques are not precursors of malignant change, but they reflect previous asbestos exposure Basal flbrotic changes suagestthe presence of asbestosis as the Sbres are fbrogenc. ‘There is no increased tisk for TB. Report An Exror ‘Mark this question & => Question 6 of 30 In asbestos related disorders which ofthe following statements are cotrect? 2) Basal fibrotic shadowing on CXR euggests coincidental idiopathic Sbrocing alveolie +) Pleural effusion develops more than 20 years after causative asbestos exposure 6) Pleural plaques are recognized precursors af Mesctheliarna © 4) Increased incidence of primary lung cancer ©) The risk of malignant mesothelioma is greatly increased in smokers compared with non-smokers 4 Increased risk for TB Question Explanation: The risk of mesothelioms is net affected by smoking but smoking and asbestos exposure greatly increases the risk of lung cancer. Tis pleural plaques which do not become apparent until 20 years or more after exposure. Pleural effusion may result from acute asbestos pleurisy. Pleural plaques are not precursors of malignant change, but they reflect previous asbestos exposure Basal flbrotic changes suagestthe presence of asbestosis as the Sbres are fbrogenc. ‘There is no increased tisk for TB. Report An Exror 2/25:2014 6:25:57 PM “Mark this question => Question Id : 55272 Question 7 of 30 A 65 year old male present with low back pain, His lab results show anemva and hypercalcemia, You suspect multiple myeloma, The most appropriate test to confirm the dianosis is a) Bone marrow aspiration 6) WEC count 6) Platelets count 6) Ktays Question Explanation: ‘Mutiple myeloma is a malignancy of plasma cells that produce monoclonal insmunoglobulin and invade and destroy adjacent bone tissue. Common manifestations include bone pain, renal insufficiency. hypercalcemia, anemia, and recurrent infections. Diagnosis requires demonctvation of M-protein (cometimes present in wins and nct serum) and either Ipc bone lesions, light-chain proteimria, or excessive marrow plasma cells.A bone marrow biopsy is usualy needed. Specific treatment includes conventional chemotherapy. Report An Error 2/25:2014 6:25:57 PM “Mark this question => Question Id : 55272 Question 7 of 30 A. 65 year old male present with low back pain. His lab results show anemia and hypercalcerria, You suspect multiple myeloma. The most appropriate test to confirm the diagnosis is ~¥ © a) Bone marow aspiration 6) WEC count 6) Platelets sount 6) X-rays Question Explanation: ‘Mutiple myeloma is a malignancy of plasma cells that produce monoclonal insmunoglobulin and invade and destroy adjacent bone tissue. Common manifestations include bone pain, renal insufficiency. hypercalcemia, anemia, and recurrent infections. Diagnosis requires demonctvation of M-protein (cometimes present in wins and nct serum) and either Ipc bone lesions, light-chain proteimria, or excessive marrow plasma cells.A bone marrow biopsy is usualy needed. Specific treatment includes conventional chemotherapy. Report An Error 2/25 :2014 6:26:13 PM “Mark this question = Question Id : 59501 Question 8 of 30 A.45-year-old male is hospitalized for DVT treatment. His platelet count on admission was 250,000/mm3. He was started on heparin and warfarin. He showed clinical improvement and planned for discharge. Before discharge his platelet count is 45,000/mm3. PT and PTT are normal Most appropriate management i a) Stop the heparin but continue the warfarin 'b) Stop the warfarin but continue the heparin c) Stop the heparin and warfarin and substitute alow-molecular-weight heparin 4) Discharge the patient, a: clinical improvement has cocurred ©) Stop the heparin and warfarin and start @ non-heparin anticoagulent Question Explanation: Heparin induced thrombocytopenia (HIT) results from the formation of heparin dependent IgG antibody. Charactesistically developing 5-10 days after the Initation of heparin by any route, itis manifested by a fallin the platelet count to less than 50% of the baseline or to an absolute level of < 50.000/tmn3, mandating discontinuation of heparin HITis best envisioned as a state of excescive and warectrained thrombin generation, and in thic prcthrombotic mileu warfarin treatment may intensify this state and must also be stopped, Direct thrombin inhibitors have been approved by the FDA for use as anticoagulants in patients with HIT. The primary treatment of HID is to stop heparin and wasfarin and substitute a non-heparin anticoagulant. A hypercoagulable state exists for several days after heparinis stopped, mandating continuation of monitoring and firther antithrombotic measures. Although low- molecular-weight heparine appear lec: licely to stimulate HIT Ig aatibody formation, they are also contraindicated, cince HIT- Ig crossreacis with al low-molecular-weight heparins 2/25 :2014 6:26:13 PM “Mark this question = Question Id : 59501 Question 8 of 30 A.45-year-old male is hospitalized for DVT treatment. His platelet count on admission was 250,000/mm3. He was started on heparin and warfarin. He showed clinical improvement and planned for discharge. Before discharge his platelet count is 45,000/mm3. PT and PTT are normal Most appropriate management i a) Stop the heparin but continue the warfarin 'b) Stop the warfarin but continue the heparin c) Stop the heparin and warfarin and substitute alow-molecular-weight heparin 4) Discharge the patient, a: clinical improvement has cocurred Y © ©) Stop the heparin and warfarin and start a non-heparin anticoagulant Question Explanation: Heparin induced thrombocytopenia (HIT) results from the formation of heparin dependent IgG antibody. Charactesistically developing 5-10 days after the Initation of heparin by any route, itis manifested by a fallin the platelet count to less than 50% of the baseline or to an absolute level of < 50.000/tmn3, mandating discontinuation of heparin HITis best envisioned as a state of excescive and warectrained thrombin generation, and in thic prcthrombotic mileu warfarin treatment may intensify this state and must also be stopped, Direct thrombin inhibitors have been approved by the FDA for use as anticoagulants in patients with HIT. The primary treatment of HID is to stop heparin and wasfarin and substitute a non-heparin anticoagulant. A hypercoagulable state exists for several days after heparinis stopped, mandating continuation of monitoring and firther antithrombotic measures. Although low- molecular-weight heparine appear lec: licely to stimulate HIT Ig aatibody formation, they are also contraindicated, cince HIT- Ig crossreacis with al low-molecular-weight heparins 2:25/2014 6:26:28 PM ‘Mark this question & => Question Td : 63415 Question 9 of 30 Drug resistance to chemotherapeutic agents is a major reascn for treatment failure. All ofthe following are tre regarding drug resistance EXCEPT a) Mutideug resistance (due to P-glycoprotein overexpression) to one class of chemotherapeutic agents generally implies cross tesistance to other classes of chemotherapeutic classes, even those which are functionally end structurally-disparate +b) Overexpression of P- glycoprotein decreases intracellslar concentrations of antineoplastic agents by actively extruding them across the cell membrane 6) Tnherent smtideug resistance occurs in tumces developing fram normal tissues that express the mdrl gene, which codes for P-glycoprotein. 4) Mutidrug resistance involving top oisomerase I can be observed with all classes of antineoplastic. ©) Apoptosis, an active form of cellular suicide, may be important in the development of muiticrug resistance. Anower (JERVRNENER) otnor User's Exp Question Explanation: ‘Moubidmg resistance (MDR) has been well characterizedin vitro, The clinical role still remains to be defined. MDR denotes the ability of malignancies to withstend exposure to lethal doses of several structurally and functionally uncelated antineoplastics. Several mechanisms of MDR have been identified. The "classic" MDR involves overexpression of P-plycoprotein. Many normal tissues express VP-glycoprotein. This protein fimctions as an energy-dependent pump, which decreases the intracellular concentrations of some antineoplastics by actively extruding them across the cell membrane. Another mechanism of MDR is through overexpression or ‘mutation of genes than control-cell cycle kinetics, Inherent mubidrug resistance occurs in those tumors which arise from tissues that normaly express the MDR1 gene. Another mechanism of MDR is programmed cell death, also known as "apoptosis" Stil another means of MDR is through atteration: of topoisomerase I. These muclear enzymes modulate topologic structure of DNA. Only certain classes of antineoplastics are affected by topoisomerase alterations. jon Report An Error 2:25/2014 6:26:28 PM ‘Mark this question & => Question Td : 63415 Question 9 of 30 Drug resistance to chemotherapentic agents is a major reason for treatment failure, all ofthe flowing are tme regarding drug resistance EXCEPT a) Multidrug resistance (due to P-glycoprotein overexpression) to one class of chemotherapeutic agents generally implies cross resistance to other classes of chemotherapeutic classes, even those waich are functionally and structurally-disparate b) Overexpression of P- glycoprotein decreases intracellular concentrations of antineoplastic agents by actively extruding ‘them across the cell membrane. c) Inherent multidrug resistance occurs in tumors developing from normal tissues that express the mdrl gene, which codes for P-glycopretein. V © A) Muiticrug resistance involving topoisomerase II can be observed with all classes of antineoplastics, ¢) Apoptosis, an active form of cellular suicide, may be important in the development of multidrug resistance. Anower (JERVRNENER) otnor User's Exp Question Explanation: ‘Moubidmg resistance (MDR) has been well characterizedin vitro, The clinical role still remains to be defined. MDR denotes the ability of malignancies to withstend exposure to lethal doses of several structurally and functionally uncelated antineoplastics. Several mechanisms of MDR have been identified. The "classic" MDR involves overexpression of P-plycoprotein. Many normal tissues express VP-glycoprotein. This protein fimctions as an energy-dependent pump, which decreases the intracellular concentrations of some antineoplastics by actively extruding them across the cell membrane. Another mechanism of MDR is through overexpression or ‘mutation of genes than control-cell cycle kinetics, Inherent mubidrug resistance occurs in those tumors which arise from tissues that normaly express the MDR1 gene. Another mechanism of MDR is programmed cell death, also known as "apoptosis" Stil another means of MDR is through atteration: of topoisomerase I. These muclear enzymes modulate topologic structure of DNA. Only certain classes of antineoplastics are affected by topoisomerase alterations. jon Report An Error ‘Marke this question <=> Question 10 of 30 Epithelial ovarian cancer is characterized by all ofthe following EXCEPT a) The normal function of the p53 gene may be lost. 'b) Oncogenes con encode growth factors or their receptors ) Itis a polyclonal disease, arising from a multiple cell 4) Surface epithelial cells may express trensforming growth factor alpha and its epidermal receptor. #) Cytokines stinmlate its growth, Question Explanation: Epithelial ovarian cancer is a clonal disease, which arises from a single cell Cytokines, such as tumor necrosis Factor alpha, interleukin 1, and interleukin 6, can stimulate the growth of these cancers. A majority of these cancers express both transforming growth facior alpha and the epidetmal growth factor receptor to which it binds, allowing autocrine stimulation of tumor cell prolferation. Oncogenes associated with these cancers can also encode growth factors or their receptors. In approximately 50% of these cancers, the normal fiction of p52, 2 DNA-binding phosphoprotein necessary for normal growth regulation, has been lost ‘Marke this question <=> Question 10 of 30 Epithelial ovarian cancer is characterized by all ofthe following EXCEPT a) The normal function of the p53 gene may be lost. 'b) Oncogenes con encode growth factors or their receptors Y © cjltisa polyclonal disease, arising from a mubiple cells 4) Surface epithelial cells may express trensforming growth factor alpha and its epidermal receptor. #) Cytokines stinmlate its growth, Question Explanation: Epithelial ovarian cancer is a clonal disease, which arises from a single cell Cytokines, such as tumor necrosis Factor alpha, interleukin 1, and interleukin 6, can stimulate the growth of these cancers. A majority of these cancers express both transforming growth facior alpha and the epidetmal growth factor receptor to which it binds, allowing autocrine stimulation of tumor cell prolferation. Oncogenes associated with these cancers can also encode growth factors or their receptors. In approximately 50% of these cancers, the normal fiction of p52, 2 DNA-binding phosphoprotein necessary for normal growth regulation, has been lost ‘Mark this question & => Question Td : 63868 Question 11 of 30 ‘The best test to confirm the diagnosis of hereditary spherocytosis is 8) Coombs test ) Bone marrow aspiration ©) Bed blood cell enzyme studies 6) Osmotic fragility sradies ¢) Human leukocyte antigens (HLA) studies Answer | Bevianaton | Other User's Explanation Report An Error (Question Explanation Heredtary spherocytosis and hereditary eliptocytosis are congenital REC membrane disorders, Symptoms, generaly milder in hereditary elliptorytosis, incnde variable dearees of anemia, jaundice, and splenomegaly. Diagnosis requires demonstration of increased RBC osmotic agility and a negetive direct antigobulin test. Rarely, patients < 45 years with symptometic disease require splenectomy. ‘Mark this question & => Question Td : 63868 Question 11 of 30 ‘The best test to confirm the diagnosis ofhereditary spherocytosis is 8) Coombs test +b) Bone marrow aspiration ©) Red blood cell enzyme studies Y © & Osmotic fragiity studies ¢) Human leukocyte antigens (HLA) studies Answer | Bevianaton | Other User's Explanation Report An Error (Question Explanation Heredtary spherocytosis and hereditary eliptocytosis are congenital REC membrane disorders, Symptoms, generaly milder in hereditary elliptorytosis, incnde variable dearees of anemia, jaundice, and splenomegaly. Diagnosis requires demonstration of increased RBC osmotic agility and a negetive direct antigobulin test. Rarely, patients < 45 years with symptometic disease require splenectomy. 225/204 6:27:12 PM Mark this question & => Question Td : 81146 Question 12 of 30 A 27 year old female presented to her physician with symptoms of a fever of 103°, enlarged auillary lymph nodes, weight loss, and right sweats for the duration of one monta, Her PPD test was negative. Bone-marrow biopsy revealed Reed-Sternberg cells. The most appropriate diagnosis is 2) Systemic lupus erythematosns ) Pulnenary tuberouloss ©) Crohn's disease ) Hodgkin's disease ¢) Sarcoidesis Question Explanation: Reed-Stemberg cells are lymphoreticular cells in the bone marrow that makes the histopathological diagnosis of Hodgkin's lymphoma. It is unusual for lupus patients to present without a rash and enlarged lymph glands (A). Tuberculosis should be associated with a positive PPD and respiratory symptoms (B). Crohns disease (C) should be associated with gastrointestinal symptoms, Reec- Steonberg cells ace aot present in sarcoidosis ) n Report An Error 225/204 6:27:12 PM Mark this question & => Question Td : 81146 Question 12 of 30 A 27 year old female presented to her physician with symptoms of a fever of 103°, enlarged auillary lymph nodes, weight loss, and right sweats for the duration of one monta, Her PPD test was negative. Bone-marrow biopsy revealed Reed-Sternberg cells. The most appropriate diagnosis is 2) Systemic lupus erythematosns ) Pulnenary tuberouloss ©) Crohn's disease Y © 4) Hodgkin's disease ¢) Sarcoidesis Question Explanation: Reed-Stemberg cells are lymphoreticular cells in the bone marrow that makes the histopathological diagnosis of Hodgkin's lymphoma. It is unusual for lupus patients to present without a rash and enlarged lymph glands (A). Tuberculosis should be associated with a positive PPD and respiratory symptoms (B). Crohns disease (C) should be associated with gastrointestinal symptoms, Reec- Steonberg cells ace aot present in sarcoidosis ) n Report An Error 2252014 6:27:26 PM Mark this question & => Question Td : 82136 Question 13 of 30 ‘What is the most common form of cancer in American women? 2) Cervical cancer. ) Colon cancer. ©) Breast cancer. 4) Lung cancer 6) Uterine cancer Question Explanation: This is the most common form of cancer in American women, undoubtedly as a result oftheir increased use of cigarettes The second most common form of cancer in this population is breast cancer (C), which is, however, the leading cause of cancer death in women in this country Cervical cancer (A), colon cancer (B), and uterine cancer ) are also common, but muck less so than jung and breast cancers, 2252014 6:27:26 PM Mark this question & => Question Td : 82136 Question 13 of 30 “What isthe most common form of cancer in American women? 2) Cervical cancer, ) Colon cancer. ©) Breast cancer, SY © 4) Lang cancer. 6) Uterine cancer. Question Explanation: This is the most common form of cancer in American women, undoubtedly as a result oftheir increased use of cigarettes The second most common form of cancer in this population is breast cancer (C), which is, however, the leading cause of cancer death in women in this country Cervical cancer (A), colon cancer (B), and uterine cancer ) are also common, but muck less so than jung and breast cancers, 2125/2014 6:27:40 PM “Mark this question = Question Id : 85086 Question 14 of 30 A 59 year old mele presents with increased fatigue and exertional dyspnea. He is diagnosed with leukemia, He undergoes genetic testing, which reveals 2 t (9: 22) translocation, Which of the following would a complete blood court most licely show? a) Increased lymphocyte ') Ihoreased neutrophil count ©) Numerous lymphoblasts 6) Numerous myeloblasts ©) Pancyioperia Anower (UERPRNERERI) otner User's Exp Question Explanation: ‘The Philadelphia chromosome results ian a BCR-ABL fusion gene that encodes a protein with tyrosine Iinase activity. I is the hallmark of chronic myeloid leukemia (CML). CML. typically presents with markedly increased aumbers of ciculating neutrophils and metamyelocytes, with lesser aumbers of eosinophils and basophils and a smell number of blasts. The disease follows an indolent course, and usually progresses to an accelerated phase | with increased numbers of circulating blasts only after several years ‘An increased lymphocyte count (shoics A) would be ceen in chronio lymphocytic leukoreia (CLL), another indolent leulcemnia. Increased numbers of blasts (choices C end D) are seen in the late stages of CML and CLL or in the acute Leukemias. The Philadelphia chromosome is occasionally associated with acute lymphoblastic leukemia and acute myeloblastic leukemia, but these are diseases of children and young adults Pancytopenia (choice E) is most typical of the myelodysplastic syndromes (MDS) in which ineffective hematopoiesis of a pluripotent siem cell produces abnormal development of all cell types. No specific chromoscmal abnormality is associared with MDS. jon Report An Error 2125/2014 6:27:40 PM “Mark this question = Question Id : 85086 Question 14 of 30 4.5) year old male presents with increased fatigue and exertional dyspnea. He is diagnosed with leukemia. He undergoes genetic testing, which reveals a t (9; 22) translocation, Which of the folowing would a complete blood court most licely show? a) Increased lymphocyte ¥ © b) Increased neutrophil count c) Numerous lymphoblasts ) Numerous myeloblasts ©) Pancyioperia Anower (UERPRNERERI) otner User's Exp Question Explanation: ‘The Philadelphia chromosome results ian a BCR-ABL fusion gene that encodes a protein with tyrosine Iinase activity. I is the hallmark of chronic myeloid leukemia (CML). CML. typically presents with markedly increased aumbers of ciculating neutrophils and metamyelocytes, with lesser aumbers of eosinophils and basophils and a smell number of blasts. The disease follows an indolent course, and usually progresses to an accelerated phase | with increased numbers of circulating blasts only after several years ‘An increased lymphocyte count (shoics A) would be ceen in chronio lymphocytic leukoreia (CLL), another indolent leulcemnia. Increased numbers of blasts (choices C end D) are seen in the late stages of CML and CLL or in the acute Leukemias. The Philadelphia chromosome is occasionally associated with acute lymphoblastic leukemia and acute myeloblastic leukemia, but these are diseases of children and young adults Pancytopenia (choice E) is most typical of the myelodysplastic syndromes (MDS) in which ineffective hematopoiesis of a pluripotent siem cell produces abnormal development of all cell types. No specific chromoscmal abnormality is associared with MDS. jon Report An Error 2:25/2014 6:27:57 PM J % wv interFace.2c pk) % € c www. interface.edu.pk/medical-exams/test-analysis php 2utid=14759 apps EQ Google A Settings [1 Signin Cl imported From IB ... elelge Lula 2 autll Saye we C. Other bookmar ‘Mark this question & => Question Td : 86774 Question 15 of 30 A 16 year old boy presents because ofhemahuria He recenily had a sinus infection and got treated with sulfonamide antibiotic. Serum haptaglobin is decceased, and serum bilirubin and LDH are increased Urine red cell count is1 2 million/mm3. A peripheral blood smear shows normocytic red cells. Numerous red blood cells with inclusions are seen on peripheral smear (shown below). The inclusions seen in this patient are best described by which one of the following? 8 0 Meee a) Hemoglobin crystals ) Iron-containing granules ©) Naciear remnants 4) Ribosomal precipitates ¢) Denatured hemoglobin newer [SERNA omer sors Explanation Report An Error Question Explanation: The presence of increased bilirubin and LDH in the serum is highly suggestive of acute hemolysis, likely related to sulfonamide ingestion, The peripheral smear shows characteristic red cells with inclusions of oxidized and denatured hemoglobin, These inclusions are known as Heinz bodies. Hemolysis following oxdant injury by crugs (sulfonamides, for example) or infection suggests glucose 5- phosphate dehydrogenase deficiency or the related deficiencies of gutathione synthetase, pyruvate kinase, and hexokinase. These conditions are typically asymptomatic between cpisodes of hemolysis. Hemoglobin crystals may be seen in hemoglobin C or hemoglobin SC disease, The crystals are often hexagonal or rhomboid in shape. Pappenheimer bodies are iron containing, dark blue granules found in Waight stained red cells in patients with sideroblastic anemia. Howell Jolly bodies are nuclear remnants within red cells that are normally removed by the spleen. The presence of Howell Jolly bodies indicates either absence of the spleen ((e., surgica removal) or a uoufluctioning spleen. Basophilic stippling refers to the presence of blue granules throughout the cytoplasm of red cell, and represents ribosomal precipitates. This is most often seen in the idivecaecine “abechal abuce: sad lead and athe: hears apteloaisccinc: Total Questions 1 BEL [as BREBERRERBEBKEEE xxxXxXXXKXXKXKXKKKXKKXKXKXKXKXKKXKKKKKKKXKXK 2:25/2014 6:27:57 PM J % wv interFace.2c pk) % € c www. interface.edu.pk/medical-exams/test-analysis php 2utid=14759 apps EQ Google A Settings [1 Signin Cl imported From IB ... elelge Lula 2 autll Saye we C. Other bookmar ‘Mark this question & => Question Td : 86774 Question 15 of 30 A 16 year old boy presents because ofhemahuria He recenily had a sinus infection and got treated with sulfonamide antibiotic. Serum haptaglobin is decceased, and serum bilirubin and LDH are increased Urine red cell count is1 2 million/mm3. A peripheral blood smear shows normocytic red cells. Numerous red blood cells with inclusions are seen on peripheral smear (shown below). The inclusions seen in this patient are best described by which one of the following? 2 0 Meee a) Hemoglobin crystals 'b) Iron-containing granules ©) Nuclear reromants 4) Ribosomal precipitates Y © e)Denatwed hemoglobin newer [SERNA omer sors Explanation Report An Error Question Explanation: The presence of increased bilirubin and LDH in the serum is highly suggestive of acute hemolysis, likely related to sulfonamide ingestion, The peripheral smear shows characteristic red cells with inclusions of oxidized and denatured hemoglobin, These inclusions are known as Heinz bodies. Hemolysis following oxdant injury by crugs (sulfonamides, for example) or infection suggests glucose 5- phosphate dehydrogenase deficiency or the related deficiencies of gutathione synthetase, pyruvate kinase, and hexokinase. These conditions are typically asymptomatic between cpisodes of hemolysis. Hemoglobin crystals may be seen in hemoglobin C or hemoglobin SC disease, The crystals are often hexagonal or rhomboid in shape. Pappenheimer bodies are iron containing, dark blue granules found in Waight stained red cells in patients with sideroblastic anemia. Howell Jolly bodies are nuclear remnants within red cells that are normally removed by the spleen. The presence of Howell Jolly bodies indicates either absence of the spleen ((e., surgica removal) or a uoufluctioning spleen. Basophilic stippling refers to the presence of blue granules throughout the cytoplasm of red cell, and represents ribosomal precipitates. This is most often seen in the idivecaecine “abechal abuce: sad lead and athe: hears apteloaisccinc: Total Questions 1 BEL [as BREBERRERBEBKEEE xxxXxXXXKXXKXKXKKKXKKXKXKXKXKXKKXKKKKKKKXKXK 2:25/2014 6:28:21 PM 7% wove interface ec k/h % e CL www. interface edu.pk/med apps cose A senngs [sgn Glimporedrromie 2. wklbr token Eon ye well Sans |-exams /test-analysis.pt w Gi other bookmar ‘Matic this question > (Question Id : 87838 Question 16 of 30 A34 year old man presents to his physician for a routine physical examination, The man adits to recent loss of 30 pounds (20% of bodyweight) and occasional fatgue but ascribes these to increase in his workload. A 2-3 cm firm, freely moveable, nontender mass in his neck on the right side is found on physical examination. Biopsy of the neck mess reveals large binucleate cells with prominent nucleolin a mixed in lammatory infiltrate. CT scan shows marked enlargement of mediastinal nodes and the presence of several very large tyinph nodes along the abdominal aotta, How should his disease be staged? a) Stage TA b) Stage IB ©) Stage TA 4) Stage TB, @) Stage TTA, ) Stage TB ) Stage TVA fh) Stage IVB Question Explanation: This is a classic presentation for Hodgkin disease, a form of lymphoma characierized by neoplastic proliferation of Reed Stemberg cells admixed with variable, aumbers of reactive lymphocytes, neutrophils, and eosinophils. About half the patients have usually painless adenopathy in the neck or supraclavicular area at the time of diagnosis. A minority have constitutional symptoms such as fatigue, weight loss, and night sweats, but these can be importent clics. Staging of Hodgkin discese is based on the extent of spread and the presence of absence of constitutionel symptoms. The man in question has inwolvement of cervical Iymph nodes, mediastinal nodes, and abdominal nodes at the time of diagnosis, so his disease would be stage III (involvement of lymph nodes or extralyrphatic organs on both sides of the diaphragm). The presence of constitutional symptoms makes this stage TTB if constitutional symptoms were absent, it would be ITA [choice E)) Stage I disease is ited to a single lymph node region or a single extralymphatic organ. Stege TI disease cen involve two or mote fymph node regions on one side of the diaphragm, or can involve contiguous extralymphatic organs or tissues, Stage IV disease is defined by the presence of multiple or disseminated disease foci in extralymphatic organs or tissues. Lymphatic involvement may occur, but need not be present for the diagnosis. Total Questions Ia IG ks SBBARBEBBEBEE ak XxXxXXXXKXXKXXKKXKXXKKXKRKKKXKKKKKKKKKK 2:25/2014 6:28:21 PM 7% wove interface ec k/h % e CL www. interface edu.pk/med apps cose A senngs [sgn Glimporedrromie 2. wklbr token Eon ye well Sans |-exams /test-analysis.pt w Gi other bookmar ‘Matic this question > (Question Id : 87838 Question 16 of 30 A 34 year old man presents to his physician for a routine physical examination, The man admits to recentloss of 30 pounds (20% of bodyweight) and occasional fatgue but ascribes these to merease in his workload, A 2-3 cm firm, freely moveable, nontender mass in his neck on the right side is found on physical examination. Biopsy of the neck mess reveals large binucleate cells with prominent nucleoli in a mixed inflammatory irflrate. CT ssan shows marked enlargement of mediastinal nodes and the presence of several very lange lymph nodes along the abdominal aorta. How shuld his disease be staged? a) Stage TA ) Stage IB ©) Stage TA 4) Stage IB ©) Stage TTA Y © 8) Stage IB 2) Stage VA fh) Stage IVB Question Explanation: This is a classic presentation for Hodgkin disease, a form of lymphoma characierized by neoplastic proliferation of Reed Stemberg cells admixed with variable, aumbers of reactive lymphocytes, neutrophils, and eosinophils. About half the patients have usually painless adenopathy in the neck or supraclavicular area at the time of diagnosis. A minority have constitutional symptoms such as fatigue, weight loss, and night sweats, but these can be importent clics. Staging of Hodgkin discese is based on the extent of spread and the presence of absence of constitutionel symptoms. The man in question has inwolvement of cervical Iymph nodes, mediastinal nodes, and abdominal nodes at the time of diagnosis, so his disease would be stage III (involvement of lymph nodes or extralyrphatic organs on both sides of the diaphragm). The presence of constitutional symptoms makes this stage TTB if constitutional symptoms were absent, it would be ITA [choice E)) Stage I disease is ited to a single lymph node region or a single extralymphatic organ. Stege TI disease cen involve two or mote fymph node regions on one side of the diaphragm, or can involve contiguous extralymphatic organs or tissues, Stage IV disease is defined by the presence of multiple or disseminated disease foci in extralymphatic organs or tissues. Lymphatic involvement may occur, but need not be present for the diagnosis. Total Questions Ia IG ks SBBARBEBBEBEE ak XxXxXXXXKXXKXXKKXKXXKKXKRKKKXKKKKKKKKKK 2/25:2014 6:28:37 PM “Mark this question => Question Td : 109437 Question 17 of 30 A 36 year old female is brought to the emergency room because of a bleeding gastric ulcer. She is anemic and loosing blood continuously. She is immediately transfused four units of packed red blood cells. Which of the fellowing complication is NOT associated with transfising the four units of blood? a) Hepatitis B. ) Transfision reaction. ©) Cytomegalovirus exposure 4) Iron overload resulting in hemochromatosis. ©) Fulmonary leukoaggtutinin reaction. Question Explanation: ron overload usvally occurs after 100 or more units of blood are wransfiised, and only after massive units of blood are transfused can hemochromatosis occur. Infections such as hepatitis B, and non-A non-B hepatitis can potentially occur after a blood transfusion, but this is sare because of donor screening programs. Petentielly fatal transfusion reactions can occur and can be immediate or delayed CMV expomre can occur after transfusions and with transplantation of organs. Pulmonary leukoagghutiin reaction after a blood transfusion is very rare, but it can occur. 2/25:2014 6:28:37 PM “Mark this question => Question Td : 109437 Question 17 of 30 A. 36 year old female is brought to the emergency room because of a bleeding gastric ulcer. She is anemic and loosing blood continuously. She is immediately transfused four units of packed red blood cells. Which of the fellowing complication is NOT ascociated with transfasing the four units of blood? 2) Hepattis B. +) Transfusion reaction. ©) Cytomegalovirus exposure. Y © 4) Fon overload resulting in hemochromatosis, 6) Fulmonary leukoagelutini reaction Question Explanation: ron overload usvally occurs after 100 or more units of blood are wransfiised, and only after massive units of blood are transfused can hemochromatosis occur. Infections such as hepatitis B, and non-A non-B hepatitis can potentially occur after a blood transfusion, but this is sare because of donor screening programs. Petentielly fatal transfusion reactions can occur and can be immediate or delayed CMV expomre can occur after transfusions and with transplantation of organs. Pulmonary leukoagghutiin reaction after a blood transfusion is very rare, but it can occur. Mark this question & => Question Td : 112425 Question 18 of 30 A.55 year old man complains of fatigue. Examination shows marked splenomegaly, but the iver is not enlarged. Blood analysis demonstrates pancytopenia and stain positive with tartrate resistant acid phosphatase (TRAP) and CD22 The most likely diagnosis is a) Waldensirom’s macroglobulinemnia ) Multiple myeloma ©) Hairy cell leukertia 6) Non-Hodgkin's lymphoma €) Hodgkin's disease Answer | bplanation Other User's Explanation Report An Error Question Explanation: Hairy cell leukemia s predominantly a disease of males in their mle aged years. tis an indolent lymphoma of B-cell origia, and these clonal cells stain positive for TRAP and CD22, two diagnostic features of the disease. The other choices-Waldenstrom’s macroglobulinemia, multiple myeloma, non Hodgkin's lymphoma, and Hodgkins disease-do not stain positive for these two cell surface antigens. Mark this question & => Question Td : 112425 Question 18 of 30 A.55 year old man complains of fatigue. Examination shows marked splenomegaly, but the liver is not enlarged, Blood analysis demonstrates pancytopenia and stain positive with tartrate resistant acid phosphatase (TRAP) and CD22. The most likely diagnosis is a) Waldenstrom’s macroglobulinemia b) Multiple myeloma © oc) Hairy cell leukemia d Non-Hodgkin's lymphoma @) Hodgkin's disease Answer | bplanation Other User's Explanation Report An Error Question Explanation: Hairy cell leukemia s predominantly a disease of males in their mle aged years. tis an indolent lymphoma of B-cell origia, and these clonal cells stain positive for TRAP and CD22, two diagnostic features of the disease. The other choices-Waldenstrom’s macroglobulinemia, multiple myeloma, non Hodgkin's lymphoma, and Hodgkins disease-do not stain positive for these two cell surface antigens. 225/204 62 18 PM Mark this question = => Question Td : 126371 Question 19 of 30 All of the following are precipitating factors for attacks in acute intermittent porphyria, EXCEPT a) Low calorie diets ) Infections ©) Surgery 4) Sulfonamides ©) Aspirin Question Explanation: “Acute intermittent porphyria is an autosomal dominant disorder in which there is an approximately 50% deficiency of hydoxymethybilane (IMB) synthase, also known as pycrcle porphobilinogen (PEG) deaminase. This is one of the essential enzymes required for normal heme biosynthesis. Because levels of aminolevulinate (ALA) and pyrrole porphobiinogen (PBG) are normal, heterozygotes usualy remain asymptomatic unless the demand for hepatic heme is increased. Ih such cases. the deficient enzyine becomes rate limiting. Cenditions in which this ocows include a wide variety of druge. Particularly important are barbiturates and sulfonamides. Aspirin does no! cause symploms. Low calorie diets, intercurrent infections, and major surgeries can also provoke attacks. 225/204 62 18 PM Mark this question = => Question Td : 126371 Question 19 of 30 All of the following are precipitating factors for attacks in acute intermittent porphyria, EXCEPT a) Low calorie diets +) Infections ©) Surgery 4) Sulfonamides Y © ©) Aspirin Question Explanation: “Acute intermittent porphyria is an autosomal dominant disorder in which there is an approximately 50% deficiency of hydoxymethybilane (IMB) synthase, also known as pycrcle porphobilinogen (PEG) deaminase. This is one of the essential enzymes required for normal heme biosynthesis. Because levels of aminolevulinate (ALA) and pyrrole porphobiinogen (PBG) are normal, heterozygotes usualy remain asymptomatic unless the demand for hepatic heme is increased. Ih such cases. the deficient enzyine becomes rate limiting. Cenditions in which this ocows include a wide variety of druge. Particularly important are barbiturates and sulfonamides. Aspirin does no! cause symploms. Low calorie diets, intercurrent infections, and major surgeries can also provoke attacks. 2:25/2014 6:29:31 PM “Mark this question <=> Question Id: 131457 Question 20 of 30 4.37 years old man presents with complaints of excess fatigue. He is enemic with an clevated ESR and calcium level. His ribs were tender on palpation. Peripheral smear revealed a rouloux pattern of the red blood cells. The most likely diagnosis is a) Lymphoma ) Multiple myeloma ©) Acute lymphocytic leukemia 6) Endocarditis 8) Chronic fatigue syndrome Question Explanation: ‘Mutiple mycloma is a hematologic malignancy associated with plasma cell infiltration in the bone marrow and peripheral smear. Lytic lesions are seen on the chest X-ray and can cause rib fractures, The calcium levelis elevated in 20% of cases, and alignment of red blood cells in a rowis called a rouloux pattern on the peripheral smear. Lymphoma would present as lymphadenopathy and an elevated lactic dehydrogenase level. Acute lymphocytic leukemia occurs in the extremes of ife-children and the elderly. Isis ascociated with abnormal lymphocytes in the bone marrow. Endocarditis would also be associated with anemia, but fever would also tbe present. Thore would not be associated hypercalcemia, Chronic fatigue syndrome is associated with adenopathy and multiple somatic complaints. 2:25/2014 6:29:31 PM “Mark this question <=> Question Id: 131457 Question 20 of 30 4.37 years old man presents with complaints of exxcess fatigue. He is anemic with an elevated BSR. and calciura level. His ribs were tender on palpation. Peripheral smear revealed a rouloux pattem ofthe red blood cells “The most likely diagnosis is a) Lymphoma oY © & Multiple myeloma ©) Acute lymphocytic leuicemia 9) Endocarditis ©) Chromic fatigue syndrome Question Explanation: ‘Mutiple mycloma is a hematologic malignancy associated with plasma cell infiltration in the bone marrow and peripheral smear. Lytic lesions are seen on the chest X-ray and can cause rib fractures, The calcium levelis elevated in 20% of cases, and alignment of red blood cells in a rowis called a rouloux pattern on the peripheral smear. Lymphoma would present as lymphadenopathy and an elevated lactic dehydrogenase level. Acute lymphocytic leukemia occurs in the extremes of ife-children and the elderly. Isis ascociated with abnormal lymphocytes in the bone marrow. Endocarditis would also be associated with anemia, but fever would also tbe present. Thore would not be associated hypercalcemia, Chronic fatigue syndrome is associated with adenopathy and multiple somatic complaints. Mark this question & => Question Td : 139680 Question 21 of 30 A leukemic patient who develops DIC is diagnosed with acute promyelocytic leukemia. ‘The translocation that is associated with the development of this disorder is 2 t@ 11) 169 ©) eM) 4)1@ 21) 6) t(15; 17) Question Explanation: Acute promyelocytic leukemia QMS by the FAR classification) is associated with at (15: 17) (q22: a1) translocation. The reddish appearing structures are Auer rods. Disseminated intravascular coagulation can occur in this disorder due to the release of T- procoagulant substances from the leukemic cells especially during treatment, ‘The t ; 11) (q21; q23) translocation is associated with acute lymphocytic leukemia (ALL) and undifferentiated leukemia, The t (6; 9) (023; q34) translocation is found in subtypes of AML with basophilia (M1, M2, MA) Burkitt leukemia, which is related to Burkitt lymphoma, is associated with t (8; 14) (q 24; 422). The t @; 21) (422; 422) translocation is sven in M2 leulcemia, also known as acute myeloid leukemia (AML) with maturation, and some NA (AML with gramilocytic and monocytic maturation). Mark this question & => Question Td : 139680 Question 21 of 30 A leukemic patient who develops DIC is diagnosed with acute promyelocytic leukemia. ‘The translocation that is associated with the development of this disorder is 2 t@ 11) 169 ©) eM) 4)1@ 21) Y @ )t(15;19 Question Explanation: Acute promyelocytic leukemia QMS by the FAR classification) is associated with at (15: 17) (q22: a1) translocation. The reddish appearing structures are Auer rods. Disseminated intravascular coagulation can occur in this disorder due to the release of T- procoagulant substances from the leukemic cells especially during treatment, ‘The t ; 11) (q21; q23) translocation is associated with acute lymphocytic leukemia (ALL) and undifferentiated leukemia, The t (6; 9) (023; q34) translocation is found in subtypes of AML with basophilia (M1, M2, MA) Burkitt leukemia, which is related to Burkitt lymphoma, is associated with t (8; 14) (q 24; 422). The t @; 21) (422; 422) translocation is sven in M2 leulcemia, also known as acute myeloid leukemia (AML) with maturation, and some NA (AML with gramilocytic and monocytic maturation). ‘Mark this question & => Question Td : 141834 Question 22 of 30 A 15 year old girl with globin gene mutation is admitted to the hospital with complaints of diffuse bone pain and respiratory distress She thiaks she ‘caught a cold" from her younger brother. She is aggressively treated, including hydration ard paia control, with good ecuits, Ac part of continued patient care vaccination against which disease chould be epecifically offered ifit hac net already been given? a) Influenza ) Mumps c) Measles a e) Rubella Question Explanatioy Early in sickle cell anemia affected children usually have an enlarged spleen. The spleen enlarges because the children are in a state of hhypersplenism, in which the spleen vigorously phagocstizes any abnormally shaped erythrocytes. During the first decade of if, sickling within the spleen occhudes its blood flow causing repeated smell infarctions, leacing to "autospleacctomy." The autosplenectomy is actually helpful to the patient hematologcaly because it may ameliorate the chronic anemia. Unfortunately, the lack of a functionzl spleen renders the patients susceptible to infections with Salmonella and encapsulated organisms such as Streptococcus pneumoniae, and Haemophilus influenzae. Tis now common practice to give pneumococcal vaccination, Infiuenza vaccinations are given to at risk populations that include children, Measles, mumps, and rubella vaccinations ere routinely given. Sickle cell patients are not at particularly increased risk. Preumococcus Report An Error ‘Mark this question & => Question Td : 141834 Question 22 of 30 A 15 year old girl with globin gene mutation is admitted to the hospital with complaints of diffuse bone pain and respiratory distress She thinks she ‘caught a cold! fiom her younger brother. Ske is aggressively treated, including hydration and pain control, with good ecuits, Ac part of continued patient care vaccination against which disease chould be epecifically offered ifit hac net already been given? a) Influenza ) Mumps c) Measles ¢ © d) Pueumococcus e) Rubella Question Explanatioy Early in sickle cell anemia affected children usually have an enlarged spleen. The spleen enlarges because the children are in a state of hhypersplenism, in which the spleen vigorously phagocstizes any abnormally shaped erythrocytes. During the first decade of if, sickling within the spleen occhudes its blood flow causing repeated smell infarctions, leacing to "autospleacctomy." The autosplenectomy is actually helpful to the patient hematologcaly because it may ameliorate the chronic anemia. Unfortunately, the lack of a functionzl spleen renders the patients susceptible to infections with Salmonella and encapsulated organisms such as Streptococcus pneumoniae, and Haemophilus influenzae. Tis now common practice to give pneumococcal vaccination, Infiuenza vaccinations are given to at risk populations that include children, Measles, mumps, and rubella vaccinations ere routinely given. Sickle cell patients are not at particularly increased risk. Report An Error 2:25/2014 6:30:20 PM “Mark this question = Question Id : 202099 Question 23 of 30 Agil aged 16 years with sickle cell disease presented with malaise and rapidly increasing dyspnoea A full blood count snowed Hb of5 1 g/dL and reticulocyte count of 5.5 x109/L (25-85 2109). The most lixely cause is 8) Epstein-Bagr virus ) Hepatitis E vis ©) Human imrauno deficiency virus 6) Human parillomavius -16 HPV 16) ©) Parvovirus B19 Anewor (UENERER) otnor UeorsExptan Question Explanation: Aplastic crisis in SSA is caused by infection with the Prrvovirus B19. The virus infects red cell progentors in hone marrow, resulting in cessation of erythropoiesis and a very rapid drop in hemoglobin. The condition is sef-Lmited, with bons morrow recovery occurring in 7-10 days, followed by brisk reticulocytosis, jon Report An Error 2:25/2014 6:30:20 PM “Mark this question = Question Id : 202099 Question 23 of 30 Agil aged 16 years with sickle cell disease presented with malaise and rapidly increasing dyspnoea A full blood count snowed Hb of5 1 g/dL and reticulocyte count of 5.5 x109/L (25-85 2109). The most lixely cause is 8) Epstein-Bagr virus ) Hepatitis E vis ©) Human imrauno deficiency virus 6) Human parillomavius -16 HPV 16) V © 8) Parvovirus B19 Anewor [UEQNERER) otnor usar Exptan Question Explanation: Aplastic crisis in SSA is caused by infection with the Prrvovirus B19. The virus infects red cell progentors in hone marrow, resulting in cessation of erythropoiesis and a very rapid drop in hemoglobin. The condition is sef-Lmited, with bons morrow recovery occurring in 7-10 days, followed by brisk reticulocytosis, jon Report An Error 2/25/2014 6:30:31 PM “Mark this question €&=> Question 24 of 30 Armale aged 30 years has sudden deterioration and haematuria 15 minutes after starting blood transfusion. His pulse rate is 120 beats per minute and blood pressure is 70/40 mmHg. The most likely cause is 2) ABO incompatibility ') Anaphylastis to anaesthetic agents 6) Disseminated intravasculer coagulation 6) Graft versus host disease 6) Bhesus izcorapatbility newer [FESRIRRSHN) Otho: User's Exptan Question Explanation: n Report An Error Tnmediate life threatening reactions with intravascular haemolysis are caused by complement activating IgG or IeM antibodies They are usual ABO antibodies and these reactions can occur after transfusion of a few millilitres of blood. 2/25/2014 6:30:31 PM “Mark this question €&=> Question 24 of 30 Armale aged 30 years has sudden deterioration and haematuria 15 minutes after starting blood transfusion. His pulse rate is 120 beats per minute and blood pressure is 70/40 mmHg. The most likely cause is Y © 2 ABO incompatibility ') Anaphylastis to anaesthetic agents 6) Disseminated intravasculer coagulation 6) Graft versus host disease 6) Bhesus izcorapatbility newer [FESRIRRSHN) Otho: User's Exptan Question Explanation: n Report An Error Tnmediate life threatening reactions with intravascular haemolysis are caused by complement activating IgG or IeM antibodies They are usual ABO antibodies and these reactions can occur after transfusion of a few millilitres of blood. 2:25/2014 6:30:43 PM Mark this question & => Question Td : 202936 Question 25 of 30 A 25 year oldman presents after developing a bhuish discolouration ofthe body, lips andnnails. He kas no relevant past history Exam reveals contral cyanosis and a grey complexion. His haemoglobin is 17.0 g/dL, PaO2 is 13.0 kPa (11.3-12 6) and Sa02 (using an oximeter) is 85% (95). The likely diegnosis is a) Argyria b) Cyanotic congenital heart disease ¢) Haemochromatosis & Methaemoglobinaemia ©) Methylene blue poisoning Quostion Explanation: ‘This patientis otherwise well and has no specific features of congerital heart disease (clubbing etc). He appears desaturated with, saturation of 85% yet good pO2. This is a typical description of methaemoglobinaemia which is the accumnulation of reversibly oxidised methaemoglobin cansing reduced oxygen affnity ofthe Hb molecule with consequent cyanosis. It can occur dne to an inherited condition or as a consequence of drugs such as nitites. Asgytia is colloidal siver toxicity. Report An Error 2:25/2014 6:30:43 PM Mark this question & => Question Td : 202936 Question 25 of 30 A 25 year oldman presents after developing a bhuish discolouration ofthe body, lips andnnails. He kas no relevant past history Exam reveals contral cyanosis and a grey complexion. His haemoglobin is 17.0 g/dL, PaO2 is 13.0 kPa (11.3-12 6) and Sa02 (using an oximeter) is 85% (95). The likely diegnosis is a) Argyria b) Cyanotic congenital heart disease ¢) Haemochromatosis Y © & Methaemoglobinaemia ©) Methylene blue poisoning Quostion Explanation: ‘This patientis otherwise well and has no specific features of congerital heart disease (clubbing etc). He appears desaturated with, saturation of 85% yet good pO2. This is a typical description of methaemoglobinaemia which is the accumnulation of reversibly oxidised methaemoglobin cansing reduced oxygen affnity ofthe Hb molecule with consequent cyanosis. It can occur dne to an inherited condition or as a consequence of drugs such as nitites. Asgytia is colloidal siver toxicity. Report An Error 2125/2014 6:30:57 PM Mark this question & => Question Td : 202997 Question 26 of 30 4.37 year old female is on warfarin after suffering a DVT. Her INR is & 2 (NR <1.4) and has conjunctival haemorrhage. Her BP is 125/55 mmElg, pulse is 65imin and ECG reveals anormal sirus rhythm, The appropriate treatment is a) FEP ) Factor VII ) Oral vitamin K Img 4) Prothrombin complex concentrate ) Stop warfarin only Question Explanation: ‘A conjunctival bleed is defined as a minor bleed and cumeat guidelines suggest that oral or TY vitamin K together with the omission of warfarin s the treatment of choice. Local guidelines should also be available. One must always weigh up the risks and benefits of reversing the anticoagulation. The patient described ic at low risk ifthe warfarin induced coagulopathy is reversed There is no suggestion from the ECG and the haemodynamic statis that there is pulmonary embolus. Major bleeds are defined as intraorbital, intracrancl, retroperitoneal or muscular bleeding causing compartment syndrome. Any acute bleeding with BP<90mmHg, oliguria ox Hb drop to less than Sg, alsa count as major bleeding episodes and require aggressive reversal ofthe coagulopathy with vitamin K., stopping warfarin and prothrombin complex concentrate or FEP. 2125/2014 6:30:57 PM Mark this question & => Question Td : 202997 Question 26 of 30 37 yeer eld female is on warfarin after suffering a DVT. Her INR is 8 2 (NR <1.4) and has conjunctival haemorrhage. Her BP is 125/55 mmElg, pulse is 65/inin and ECG reveals a normal sinus chythm, The appropriate treatment is a) FEP ) Factor VIZ Y © ©) Oral vitamin K Img 4) Prothrombin complex: concentrate ) Stop warfarin only Question Explanation: ‘A conjunctival bleed is defined as a minor bleed and cumeat guidelines suggest that oral or TY vitamin K together with the omission of warfarin s the treatment of choice. Local guidelines should also be available. One must always weigh up the risks and benefits of reversing the anticoagulation. The patient described ic at low risk ifthe warfarin induced coagulopathy is reversed There is no suggestion from the ECG and the haemodynamic statis that there is pulmonary embolus. Major bleeds are defined as intraorbital, intracrancl, retroperitoneal or muscular bleeding causing compartment syndrome. Any acute bleeding with BP<90mmHg, oliguria ox Hb drop to less than Sg, alsa count as major bleeding episodes and require aggressive reversal ofthe coagulopathy with vitamin K., stopping warfarin and prothrombin complex concentrate or FEP. 2:25/2014 6:31:10 PM ‘Merk this question & => Question Td : 203140 Question 27 of 30 ATO year old chronic smoker and drinker male presents with a tender let calf and has past history of headaches, tiredness and dizriness. On exam he has a BP of 186/102 mmHg and has a swollen, hot tender and erythematcus left calf Dopplers confirms DVT. His Hb is 19 g/dL, hacmatoctit is 0.58 (0.40-0,52) and WBC count is 12.5 109/L and platelet count is 500 =109/L. Which one is the most appropriate investigation to establish the diagnosis? 8) Abdorninal ulteasound sean ») Arterial blood gases ©) Bone marrow trephine 6) Leucocyte alkaline phosphatase score 6) Red blood sel macs Anewor [UBQNEANY) Other teorsExplanation Report An Error Question Explanati ‘The most signiicant abnormality is the raised haemoglobin and haematocrit suggesting p olycythaemia which in the presence of all the other fatures suggests secondary polycythaemia Therefore, the most useful and appropriate investigation will be red cell mass stucies which will distinguish between true and relative polycythaemia Further investigations will hen be dictated by the results of this initial test, an ultrasound scan (USS) bone matrow etcetera, and blood gases may be needed after the initial red cell mass stucies The leucocyte alkaline phosphatase score is rather outdated and seldom performed. The raised WEC and platelet count would, suggest that this is primary polycythaemaia not apparent polycythaemia but red blood cell mass is stil the best answer. 2:25/2014 6:31:10 PM ‘Merk this question & => Question Td : 203140 Question 27 of 30 ATO year old chronic smoker and drinker male presents with a tender let calf and has past history of headaches, tiredness and dizriness. On exam he has a BP of 186/102 mmHg and has a swollen, hot tender and erythematcus left calf Dopplers confirms DVT. His Hb is 19 g/dL, hacmatoctit is 0.58 (0.40-0,52) and WBC count is 12.5 109/L and platelet count is 500 =109/L. Which one is the most appropriate investigation to establish the diagnosis? 8) Abdorninal ulteasound sean ») Arterial blood gases ©) Bone marrow trephine 6) Leucocyte alkaline phosphatase score WV © 8) Red blood cell mass Anewor [UBQNEANY) Other teorsExplanation Report An Error Question Explanati ‘The most signiicant abnormality is the raised haemoglobin and haematocrit suggesting p olycythaemia which in the presence of all the other fatures suggests secondary polycythaemia Therefore, the most useful and appropriate investigation will be red cell mass stucies which will distinguish between true and relative polycythaemia Further investigations will hen be dictated by the results of this initial test, an ultrasound scan (USS) bone matrow etcetera, and blood gases may be needed after the initial red cell mass stucies The leucocyte alkaline phosphatase score is rather outdated and seldom performed. The raised WEC and platelet count would, suggest that this is primary polycythaemaia not apparent polycythaemia but red blood cell mass is stil the best answer. 2252014 6:31:24 PM “Mark this question ec: Question Id : 203589, Question 28 of 30 A.42 year old man has bleeding gums and ease of bruising, He is taking Omeprazole for dyspepsia. Labs show Hb of 12.5 g/dL, MCV of 9041, platelet count of 20 x109/L and PT of 13.5s (11.5-15.5), Blood fil shows occasional giant platelets. The most likely diagnosis is a) Amegakaryocytic thrombocytopenia ) Disseminated intravascular coagulation c) Drug-induced thrombecytopenia 4) Immune thrombocytopenia ©) Thrombotic thrombocytopenic purpura Avewor [JEQVIRISHAR) other usorsExplanation Repost An Error Question Explanation: ‘The orly abnormality is the very low platelet court, The bone marrow is still working as there are giant platelets seen on fil, which, you see when there is peripheral consumption of the platelets. The large platelets are a sign that the bone marrow is churning them out prematurely to keep up with demand, With disseminated intravascular coagulation the prothrombin ime would be prolonged, With thrombotc thrombocytopenic purpura the haemoglobin would be low- as haemolysis is a feature Drug-induced thrombocytopenia in itselfis an immine mechanism, and while lansoprazole can cause a reduction in platelet court, itis not classically a drug you associate with dmg induced thrombocytopenia. Immune thrombocytopenia is very common and would give this very lowr platelet count, and by choosing this answer it covers drug ncuced thrombocytopenia as well. 2252014 6:31:24 PM “Mark this question ec: Question Id : 203589, Question 28 of 30 A.42 year old man has bleeding gums and ease of bruising, He is taking Omeprazole for dyspepsia. Labs show Hb of 12.5 g/dL, MCV of 9041, platelet count of 20 x109/L and PT of 13.5s (11.5-15.5), Blood fil shows occasional giant platelets. The most likely diagnosis is a) Amegakaryocytic thrombocytopenia ) Disseminated intravascular coagulation c) Drug-induced thrombecytopenia Y © 4) Immune thrombocytopenia ©) Thrombotic thrombocytopenic purpura Avewor [JEQVIRISHAR) other usorsExplanation Repost An Error Question Explanation: ‘The orly abnormality is the very low platelet court, The bone marrow is still working as there are giant platelets seen on fil, which, you see when there is peripheral consumption of the platelets. The large platelets are a sign that the bone marrow is churning them out prematurely to keep up with demand, With disseminated intravascular coagulation the prothrombin ime would be prolonged, With thrombotc thrombocytopenic purpura the haemoglobin would be low- as haemolysis is a feature Drug-induced thrombocytopenia in itselfis an immine mechanism, and while lansoprazole can cause a reduction in platelet court, itis not classically a drug you associate with dmg induced thrombocytopenia. Immune thrombocytopenia is very common and would give this very lowr platelet count, and by choosing this answer it covers drug ncuced thrombocytopenia as well. 2/25/2014 6:31:41 PM "Mark this question & => Question 29 of 30 ‘A.33 year old man was prescribed an oral antibiotic for a UTL Two days later he noticed thet his urine was increasingly dak in colour, His Hb is 8.5g/dL and Reticulocyte count is 147x109/L (25-85 x109). Peripheral flm shows marked anisopokilocytosis and bite cells. The likely diagnosis is a) Acute Myeloid Leukaemia }) Autoimmune haemelytic anaemia ) Haemogiobin H disease 4d) Hereditary spherocytosis ¢) Paroxysmal cold haemoglobinuria Answer | Explanation) Other User's Explanation Report An Error Question Explanation: Think that his is a difficult question! Paroxystnal cold hemoglobinuria (PCHL is a rare type of autoimmune hemolytic anemia (ATHA), occurring primarily in children. The classic symptom of PCHis a sudden onset of hemoglobinuria following expomure to cold, even for a few minutes. Symptoms may occur minutes to hours following exposure to cold, Hemoglobinuria is not always present because, in some persons wth PCH, the autoantibody level is not high enough to cause intravascular hemolysis PCH is usuelly of abrupt onset in the setting of an infectious disease. Given the patients age, and the specific history in this case, the diagnosis is unlikely to he FCH TF the diagnosis was hereditary spherocytosis, then the blood film would show spherocytes. In Haemoglobin H disease, the typical inclusions can be demonstrated in erythrocytes stained with biiliant cresyl blue, anda chronic microcyte, hypochromic anaemia would be present. Haemolstic anaemia may be precipitated by suifonamides, and also by penicilins. This gentleman may have been treated with trimethoprim or pericilin, which then caused ATHIA, with the typical blood film, I feel that the most appropriate answer to this question is B, given the patient's age, the lack of history of exposure to cold, and the history which is given ofanibiotic prescription 2/25/2014 6:31:41 PM "Mark this question & => Question 29 of 30 A.33 year old man was presenbed an oral antibiotic for a UTL. Two days later he noticed that his urine was increasingly dark in colour, His Hb is 8 Sg/dL and Reticulocyte count is 147x109/L (25-85 #109), Peripheral fl shows merked anisopoikilocytosis and. bite celle. The lleely diagnosis is a) Acute Myeloid Leuicaemia ¥ © b) Autoimmune haemolytic anaemia ) Haemogobin H disease a) Hereditary spherocytosis «) Paroxysmal cold haemoglobinuria Answer | Explanation) Other User's Explanation Report An Error Question Explanation: Think that his is a difficult question! Paroxystnal cold hemoglobinuria (PCHL is a rare type of autoimmune hemolytic anemia (ATHA), occurring primarily in children. The classic symptom of PCHis a sudden onset of hemoglobinuria following expomure to cold, even for a few minutes. Symptoms may occur minutes to hours following exposure to cold, Hemoglobinuria is not always present because, in some persons wth PCH, the autoantibody level is not high enough to cause intravascular hemolysis PCH is usuelly of abrupt onset in the setting of an infectious disease. Given the patients age, and the specific history in this case, the diagnosis is unlikely to he FCH TF the diagnosis was hereditary spherocytosis, then the blood film would show spherocytes. In Haemoglobin H disease, the typical inclusions can be demonstrated in erythrocytes stained with biiliant cresyl blue, anda chronic microcyte, hypochromic anaemia would be present. Haemolstic anaemia may be precipitated by suifonamides, and also by penicilins. This gentleman may have been treated with trimethoprim or pericilin, which then caused ATHIA, with the typical blood film, I feel that the most appropriate answer to this question is B, given the patient's age, the lack of history of exposure to cold, and the history which is given ofanibiotic prescription 22512014 6:32:06 PM ‘Mark this question & Question Id : 216834 Question 30 of 30 A 14 year-old girl (child of consanguineous matriage) had a severe nosebleed, When a blood sample wes, the clot that formed failed to retract. Peripheral blood smear showed normal-sized platelets that were individual, without clumping, Her platelet's faled to aggregate with any physiologic aggregating agent, inciuding a high concentration of exogenous ADP. Which of the following is the most litely ciagnosis? a) Bernard-Soulier syndrome b) Chédiak-Hizashi syndrome c) May-Hegglin anomaly 4) Thrombasthenia €) Von Willebrand disease Anewor [UBIIRNEHAR) otnerUsors Explanation Report An Error Question Explanatioy ‘This patient hac thrombasthenia, This rere genetic disorder of platelet function has autoscmal recessive inheritance. The biochemical detect appears to be alack of the platelet membrene glycoprotein GPIIb-Ila, This protein normally can bind to Sbrinogen, and in its absence, platelet aggregation (and the resulting clot retracton) cannot occu. The laboratory findings listed in the question stem are typical. Affected individuals have a particular problem with severe bleeding from mucosal surfaces, which commonly undergo minor traumas, Bernard-Soulier syndrome is characterized by very large platelets Chédiak-Higashi syndrome is en immunodeficiency disease with detective microbicidal activity after phagocytosis and unasvally large platelets ‘May-Hegglin anomdly is a thrombocytopenic disease with abnocmal neutrophils ‘Von Willebrand disease is due to an abnormality of factor VIII related von Willebrand factor that secondatily affects platelet function, 22512014 6:32:06 PM ‘Mark this question & Question Id : 216834 Question 30 of 30 A 14 year-old girl (child of consanguineous matriage) had a severe nosebleed, When a blood sample wes, the clot that formed failed to retract. Peripheral blood smear showed notmal-sized platelets that were individual, without clumping Her platelet’s failed to aggregate with any physiologic aggregating agent, inciuding a high concentration of exogenous ADP. Which of the following is the most litely ciagnosis? a) Bernard-Soulier syndrome b) Chédiak-Hizashi syndrome c) May-Hegglin anomaly V © 4) Thrombasthenia €) Von Willebrand disease Anewor [UBIIRNEHAR) otnerUsors Explanation Report An Error Question Explanatioy ‘This patient hac thrombasthenia, This rere genetic disorder of platelet function has autoscmal recessive inheritance. The biochemical detect appears to be alack of the platelet membrene glycoprotein GPIIb-Ila, This protein normally can bind to Sbrinogen, and in its absence, platelet aggregation (and the resulting clot retracton) cannot occu. The laboratory findings listed in the question stem are typical. Affected individuals have a particular problem with severe bleeding from mucosal surfaces, which commonly undergo minor traumas, Bernard-Soulier syndrome is characterized by very large platelets Chédiak-Higashi syndrome is en immunodeficiency disease with detective microbicidal activity after phagocytosis and unasvally large platelets ‘May-Hegglin anomdly is a thrombocytopenic disease with abnocmal neutrophils ‘Von Willebrand disease is due to an abnormality of factor VIII related von Willebrand factor that secondatily affects platelet function, ‘Merk this question => Question Td : 10654 Question 1 of 30 A. 46-year-old man presents with facial sweling and breathlessness. His chest X-ray seveals paratracheal lymphadenopathy. Which of the following statements is most accurate regarding the superior vena caval obstruction? 8) The most common cause is squamous cell carcinoma +) is associated with Kussmaul's sign 6) Tkmay be associated with Pupilary dialation 6) Timay be associeted with Pupilary dialation 6) The commonest symptom cough and chest pain Avewor (UERVIRNSWAR) otnorUsore Explanation Report An Eiror Question Explanation: ‘The commonest symptoms are usually cough and chest pain, due to the distortion of mediastinal anatomy. Phhysical signs are often absent or minimal, but classicaly there is facial and periorbital edema, chemosis and distended veins SVCO is most likely caused by bronchogenic carcinotra especially smell cell carcinoma (10% small cell cancers present with SVC) due to mediastinal lymphadenopathy Other canses inchid= lymphoma, aortic aneurysm, and mediastinal fibrosis and mediastinal gozer Chemotherapy + radiotherapy is the treatment of choice in small cell carcinoma, Radio therapy may be the treatment of choice for non-smal cell carcinoma, Median survival of lung cancer presenting with SVOC, even with treatments 5 months. Lymphoma hes better prognosis and will recnire specific chemotherapy + radiotherapy Recurrent laryngeal nerve palsy usually occurs with malignant tumour but can occur with aneurysm ofaortic arch. There may also be ‘Homer's syndrome due to involvement of sympathetc chain and hence pupillary constriction, Compression of vital structures cant result in stridor and dysphagia. SVCO is associated with elevated non-pulsatile jugular venous pressure ussmaul’s sign is the paradozical rise in VP on inspiration due to constrictive pericarditis or significant pericardial effusion ‘Merk this question => Question Td : 10654 Question 1 of 30 A. 46-year-old man presents with facial sweling and breathlessness. His chest X-ray seveals paratracheal lymphadenopathy. Which of the following statements is most accurate regarding the superior vena caval obstruction? 8) The most common cause is squamous cell carcinoma +) is associated with Kussmaul's sign 6) Tkmay be associated with Pupilary dialation 6) Timay be associeted with Pupilary dialation ¥ © 6) The commonest symptom cough and chest pain Avowor (UBRVIRNSWAR) otnorUsore Explanation Roport An Error Question Explanation: ‘The commonest symptoms are usually cough and chest pain, due to the distortion of mediastinal anatomy. Phhysical signs are often absent or minimal, but classicaly there is facial and periorbital edema, chemosis and distended veins SVCO is most likely caused by bronchogenic carcinotra especially smell cell carcinoma (10% small cell cancers present with SVC) due to mediastinal lymphadenopathy Other canses inchid= lymphoma, aortic aneurysm, and mediastinal fibrosis and mediastinal gozer Chemotherapy + radiotherapy is the treatment of choice in small cell carcinoma, Radio therapy may be the treatment of choice for non-smal cell carcinoma, Median survival of lung cancer presenting with SVOC, even with treatments 5 months. Lymphoma hes better prognosis and will recnire specific chemotherapy + radiotherapy Recurrent laryngeal nerve palsy usually occurs with malignant tumour but can occur with aneurysm ofaortic arch. There may also be ‘Homer's syndrome due to involvement of sympathetc chain and hence pupillary constriction, Compression of vital structures cant result in stridor and dysphagia. SVCO is associated with elevated non-pulsatile jugular venous pressure ussmaul’s sign is the paradozical rise in VP on inspiration due to constrictive pericarditis or significant pericardial effusion 2:25/2014 6:33:30 PM Mark this question & => Question Td : 10677 Question 2 of 30 A 40-year-old tale is receiving Cisplatin based chemotherapy as adnvant therapy for Iymphotia Which of the following is a typical side effect of Cisplatin? a) Acoustic Nerve damage b) Cerebellar ataxia c) Hemorrhagic eystits ) Cardiotonicity 6) Rhabdomyolysis Question Explanation: Typical side effects of cisplatin incinde marrow toxicity, ototcxicity (CN VID, peripheral nenropahy, nephrotoxicity, alopecia and changes in taste 2:25/2014 6:33:30 PM Mark this question & => Question Td : 10677 Question 2 of 30 A 40-year-old tale is receiving Cisplatin based chemotherapy as adnvant therapy for Iymphotia Which of the following is a typical side effect of Cisplatin? Y © 2) Acoustic Nerve damage b) Cerebellar ataxia c) Hemorrhagic eystits ) Cardiotonicity 6) Rhabdomyolysis Question Explanation: Typical side effects of cisplatin incinde marrow toxicity, ototcxicity (CN VID, peripheral nenropahy, nephrotoxicity, alopecia and changes in taste 2252014 6:33:12 PM “Mark this question €& => Question 3 of 30 The following are recognized features of Fancoast’s tumour except: 2) Ipsilateral Miosis and Ptosis 6) Wasting of the dorsal interossei ©) Pain in the arm radiating to the fourth and fifth fingers 46) Erosion of the first rib 6) Weakness of abduction at the shoulder Question Explanation: Report An Error ‘The tunour causes pain in the C8 and T1 distribution and Homer's syndrome (Miosis and Ptosis). It may cause small muscle wasting of the hands and erosion of 1*trib. The nerve root for abduction of shoulder is C5 2252014 6:33:12 PM “Mark this question €& => Question 3 of 30 ‘The folowing are recognized feanures of Fancoast's tumour except: 2) Ipsilateral Miosis and Ptosis 'b) Wasting of the dorsal interossei 6) Pain in the crm radiating to the fourth and fifth fingers 6) Erosion of the first sib JY © ®) Weakness of abduction at the shoulder Question Explanation: Report An Error ‘The tunour causes pain in the C8 and T1 distribution and Homer's syndrome (Miosis and Ptosis). It may cause small muscle wasting of the hands and erosion of 1*trib. The nerve root for abduction of shoulder is C5 “Mark this question & => Question Id : 14135 Question 4 of 30 A 23-year-old otherwise being well has a petechial rash on the lower legs and conjunctival pallor. He takes no medication and denies any ilict drug use, All investigations are normal except that Hemoglobin = 4 1 g/dL, WBC count=1 x10°//L and Platelets = 20 x10°/L Which of the following is the likely diagnosis? a) Acute lymphocytic leukemia +) Acute myeloid lenleoria c) Aplaste anaemia 4) Henoch-Schonlein purpura €) Hodgkin's ymphoma Question Explanation: This patient appears to have complete suppressicn of all his marrow components suggesting aplastic anemia The acquired condition may be associated with: + Dmg therapy such as cytotoxics * Infections such as viral hepatitis + CHoramphenicel + Tonising radiation and + Chemicals “Mark this question & => Question Id : 14135 Question 4 of 30 A 23-year-old otherwise being well has a petechial rash on the lower legs and conjunctival pallor. He takes no medication and denies any ilict drug use, All investigations are normal except that Hemoglobin = 4 1 g/dL, WBC count=1 x10°//L and Platelets = 20 x10°/L Which of the following is the likely diagnosis? a) Acute lymphocytic leukemia +) Acute myeloid lenleoria Y¥ © ¢) Aplaste anaemia 4) Henoch-Schonlein purpura €) Hodgkin's ymphoma Question Explanation: This patient appears to have complete suppressicn of all his marrow components suggesting aplastic anemia The acquired condition may be associated with: + Dmg therapy such as cytotoxics * Infections such as viral hepatitis + CHoramphenicel + Tonising radiation and + Chemicals 2:25/2014 6:34:11 PM Mark this question = => Question Td : 54721 Question 5 of 30 A child develops easy bruising and excessive bleeding from small skin cuts. Tests show bleeding time is increased and a deficiency of vont Willebrand's factor. The most appropriate treatment is a) Factor 8 b) Factor 9 ) Platelet transfusion &) DDAVP Question Explanation: Von Willebrand's disease is a hereditary deficiency of von Willebrand's factor (VW), which canses platelet dysfinction Bleeding, tendency is usually mild, Screening tests show a prolonged bleeding time, normal platelet count, and, possibly, a slightly prolonged PIT. Diagnosis is based on low levels of VWE antigen and abnormal ristocetin cofactor activity. Treatinent volves control of bleeding with replacement therapy (cryoprecipitate or pasteurized intermediate-purty Factor VII concentrate) or desmopressin (@DAVE), Factors 8 and 9 are given in hemophill 2:25/2014 6:34:11 PM Mark this question = => Question Td : 54721 Question 5 of 30 A child develops easy bruising and excessive bleeding from small skin cuts. Tests show bleeding time is increased and a deficiency of vont Willebrand's factor. The most appropriate treatment is a) Factor 8 b) Factor 9 c) Platelet transfusion ¥ © dDDAVP Answer | Explanation Other User's Explanation Report An Error Question Explanation: Von Willebrand's disease is a hereditary deficiency of von Willebrand's factor (VW), which canses platelet dysfinction Bleeding, tendency is usually mild, Screening tests show a prolonged bleeding time, normal platelet count, and, possibly, a slightly prolonged PIT. Diagnosis is based on low levels of VWE antigen and abnormal ristocetin cofactor activity. Treatinent volves control of bleeding with replacement therapy (cryoprecipitate or pasteurized intermediate-purty Factor VII concentrate) or desmopressin (@DAVE), Factors 8 and 9 are given in hemophill 2125/2014 6:34.26 PM Mark this question => Question Id : 56975 Question 6 of 30 A patient attends for breast screening and asks you the following questions. Which statement is correct? 2) p53 mutation is 2 commonly identified in subjects with a strong famly history of breast cancer +b) Memmographic screening is offered to all women aged 20-75 years ©) Barly diagnosis does not change the rate of death from breast cancer 4) Mammographic screening it more cencitive in patient groups with less dense breact ticme «) In young patients with a BRCA mutation, Mammographic screening has a high sensitivity for detecting tumors Question Explanation: Ta young paticats, MRI may help to delineate breast cancer in breast tissue, which in younger patients is denser than the breast tissue of older patients. Mammography is more sensitive in older patient groups partly because in general the breast tissue is less dense. Mammographic screening of all women between the ages of 50 to 70 years can reduce mortality from breast cancer 25%, There is no consensus regarding the value of breast cancer screening among women who are 40 to 49 years old, The presence of a germ-line mutation ofthe BRCAI er BRCA2 gene increases the riske of breact cancer considerably, and patients with these mautatione should be offered the opportunity to receive screening at much younger than 50 years. P53 mutaiions are rarely found in palients even with a family history of breast cancer. 2125/2014 6:34.26 PM Mark this question => Question Id : 56975 Question 6 of 30 A patient attends for breast screening and asks you the following questions. Which statement is correct? 2) p53 mutation is 2 commonly identified in subjects with a strong famly history of breast cancer +b) Memmographic screening is offered to all women aged 20-75 years ©) Barly diagnosis does not change the rate of death from breast cancer Y © 4) Mammograptic screening is more sensitive in patient groups with less dense breast tissue ¢) In young patients with a BRCA mutation, Mamnmographis screening has a high sensitinity for detecting tumors Question Explanation: Ta young paticats, MRI may help to delineate breast cancer in breast tissue, which in younger patients is denser than the breast tissue of older patients. Mammography is more sensitive in older patient groups partly because in general the breast tissue is less dense. Mammographic screening of all women between the ages of 50 to 70 years can reduce mortality from breast cancer 25%, There is no consensus regarding the value of breast cancer screening among women who are 40 to 49 years old, The presence of a germ-line mutation ofthe BRCAI er BRCA2 gene increases the riske of breact cancer considerably, and patients with these mautatione should be offered the opportunity to receive screening at much younger than 50 years. P53 mutaiions are rarely found in palients even with a family history of breast cancer. ‘Mark this question & => Question Td : 61638 Question 7 of 30 A 24 year old female arrives at her famrily physician's office complaining of easy brussing over the past month, She is not as concerned about her "rash-lite" skin bruising as she is about her gums bleeding with brushing her teeth over the past three days. The only abnormality detected is a platelet count of 70,00(/mm. The least ikcely cause is a) FolatelB12 deficiency b) Lymphomalleukernia ©) Factor VIII deficiency 9) SLE Question Explanatio: ‘A vitamin deficiency, lymphoma or leukemia or upus are in this patients differential diagnoses. Factor 8 deficiency (nemophilia A) (factor VIII deficiency) has an X-linked genetic transirission. Because these genes are located on the X chromosome, hemophilia affects males almost exclusively. Danghters of hemophilic males are obligate, carriers, but sons are normal, Bach son of a carrier has 2.50% chance of having hemophilia, and cach danghter has a 50% chencs of being a carticr. Paticnts with hemophila bleed into tissues (eg, hemarthroses, muscle hematomas, retroperitoneal hemorrhage), and the bleeding may be delayed after trauma Pain often occurs as bleeding commences, sometines before other signs of bleeding develop. Chronic or recurrent hemarthroses can lead to synovitis and arthropathy. Even a trivial blow to the head can cause intracranial bleeding, Bleeding into the base of the tongue can cause life-threatening airway compression. ‘Mark this question & => Question Td : 61638 Question 7 of 30 ‘A 24 year old female arrives at her family physician's office complaining of easy bruising over the past month, She is not as concerned about her "rash-like" skin bruising as she is about her gums bleeding with brushing her teeth over the past three days. The orly abnormality detected is a platelet count of 70,000/mm. The least likely cause is a) FolatelB 12 deficiency b) Lymphomalleukemia Y © ¢) Factor VIII deficiency @ SLE Question Explanatio: ‘A vitamin deficiency, lymphoma or leukemia or upus are in this patients differential diagnoses. Factor 8 deficiency (nemophilia A) (factor VIII deficiency) has an X-linked genetic transirission. Because these genes are located on the X chromosome, hemophilia affects males almost exclusively. Danghters of hemophilic males are obligate, carriers, but sons are normal, Bach son of a carrier has 2.50% chance of having hemophilia, and cach danghter has a 50% chencs of being a carticr. Paticnts with hemophila bleed into tissues (eg, hemarthroses, muscle hematomas, retroperitoneal hemorrhage), and the bleeding may be delayed after trauma Pain often occurs as bleeding commences, sometines before other signs of bleeding develop. Chronic or recurrent hemarthroses can lead to synovitis and arthropathy. Even a trivial blow to the head can cause intracranial bleeding, Bleeding into the base of the tongue can cause life-threatening airway compression. 2:25/2014 6:35:02 PM CO Mark this question —& => Question 8 of 30 A 64 year old man who is a cigar and pipe smoker develops a circular lesion on the lower lp. See picture: The most llely diagnosis is a) Squamous cell carcinema ') Basal cell carcinoma ©) Actinic keratosis ) Malignant melanoma Anower [UERQBNENONI) Otner User's Exp Question Explanat Report An Error Question Td : 63687 Squamous cell carcinoma of the oral cavily occurs mostly in smokers. About 38% of all oral squamous cell carcinomas occur on the lower ip. Fortunately for lower lip lesions, 5 year survwal is 90%, and metastases are rare, Surgery and radiation therapy are the treatments of choice. Basal cell carcinoma most often appears on sun exposed areas such as the face. scalp, ears, chest, back. and legs. Tae most common appearance of basal cell cancer is that of a small dome shaped bump thet bas a pearly white color. Basel call cancer can also appear as a pimple like growth that heals, only to come back again. 2:25/2014 6:35:02 PM CO Mark this question —& => Question 8 of 30 A 64 year old man who is a cigar and pipe smoker develops a circular lesion on the lower lp. See picture: The most likely diagnosis is Y © a) Squamous cell carcinoma b) Basal cell carcinoma c) Actinic keratosis oD) Malignant melanoma Anower [UERQBNENONI) Otner User's Exp Question Explanat Report An Error Question Td : 63687 Squamous cell carcinoma of the oral cavily occurs mostly in smokers. About 38% of all oral squamous cell carcinomas occur on the lower ip. Fortunately for lower lip lesions, 5 year survwal is 90%, and metastases are rare, Surgery and radiation therapy are the treatments of choice. Basal cell carcinoma most often appears on sun exposed areas such as the face. scalp, ears, chest, back. and legs. Tae most common appearance of basal cell cancer is that of a small dome shaped bump thet bas a pearly white color. Basel call cancer can also appear as a pimple like growth that heals, only to come back again. ‘Mark this question & => Question Id : 74047 Question 9 of 30 ‘The most appropriate treatment for chronic myelocytic leukemia is a) Hycroxyurea plus phlebotomy +b) Cisplatin plus etoposide. } Cisplatin plus fuorouracil 4 @) Chlorambucil plus prednisone. Question Explanation Mone of the other choices are indicated for the treatment of chronic myelocytic leukemia. Hydroxyurea plus alpha interferon ‘Mark this question «=> Question 9 of 30 ‘Tne most appropriate treatment for chronic myelocytic leukemias a) Hycroxyurea plus phlebotomy +) Cisplatin plus etoposide. } Cisplatin plus fuorouracil Y © d) Hydroxyurea plus alpha interferon. @) Chlorambucil plus prednisone. Question Explanation Mone of the other choices are indicated for the treatment of chronic myelocytic leukemia. 2252014 6:35:37 PM “Mark this question €&c=> Question 10 of 30 ‘Which ofthe fellowingis NOT true of hemochromatosis? a) Itis produced by excessive intestinal absorption of iron. ') Itresuts from a tmutant gene linked to the HLA on chromosome 6 ©) Troccurs in ron-loading anemias, such as thalasseania ) This rare following excessive oral iron ingestion, ¢) Routine treatment involves chelating agent (Deferoxamine) administration Question Explanation: ‘Hemochromatosis is a parenchymal iron-storage disorder. There is an inappropriate increase in intestinal iron absorption. Both a genetic as well as an acquired form exist, The two forms are clitically indistinguisheble. The genetic form results from a mutant gene linked to the HLA on the short arm of chromosome 6. The acquired form results from iron-loading anerrias, such as thalassemia and sideroblastic anemias. Excluding the South African Bantu, excessive oral ingestion of ron rarely results in hemochromatosis, Parenteral administration produces reticuloendothelial cell ion overload, Rouine treatment in an otherwise-healthy person consists of regularly scheduled phlebotomies (bloodletting) For those unable to tolerate routine blood draws, there is a chelating agent available for use e.g, Deferoxamine. 2252014 6:35:37 PM “Mark this question €&c=> Question 10 of 30 ‘Which ofthe followingis NOT true of hemochromatosis? 2) Ikis produced by excessive intestinal absorption of iron, ') Itresuts from a tmutant gene linked to the HLA on chromosome 6 ©) It ovewrs in ron-loading anemizs, such as thalassemia. ) This rare following excessive oral iron ingestion, WV © ¢) Routine treatment involves chelating agent (Deferoxamine) administration. Question Explanation: ‘Hemochromatosis is a parenchymal iron-storage disorder. There is an inappropriate increase in intestinal iron absorption. Both a genetic as well as an acquired form exist, The two forms are clitically indistinguisheble. The genetic form results from a mutant gene linked to the HLA on the short arm of chromosome 6. The acquired form results from iron-loading anerrias, such as thalassemia and sideroblastic anemias. Excluding the South African Bantu, excessive oral ingestion of ron rarely results in hemochromatosis, Parenteral administration produces reticuloendothelial cell ion overload, Rouine treatment in an otherwise-healthy person consists of regularly scheduled phlebotomies (bloodletting) For those unable to tolerate routine blood draws, there is a chelating agent available for use e.g, Deferoxamine. 2:25/2014 6:35:49 PM Mark this question —& => Question Td : 82239 Question LL of 30 Fyperviscosity syndrome is mast characteristic of which of the following? a) Wegener's granulomatosis. ) Familial Mediterranean fever. ©) Syphilis 49) Systemic lupus erythematosus. ©) Weldenstrom’s macroglobulinemia Question Explanation: Fyperviscosity is often a majer manifestation of Waldenstrom’s macroglobulnemia. None of the other syndromes is typically associated with hyperviscosity 2:25/2014 6:35:49 PM Mark this question —& => Question Td : 82239 Question LL of 30 ‘Hyperviscosity syndrome is most characteristic of which of the following? a) Wegener's granulomatosis ‘t) Familial Mediterranean fever. €) Syphilis ) Systemic lupus erythematosus. ¥ © © Waldenstrom’s macroglobulinemia Question Explanation: Fyperviscosity is often a majer manifestation of Waldenstrom’s macroglobulnemia. None of the other syndromes is typically associated with hyperviscosity Marke this question >> Question Td : 84870 Question 12 of 30 ‘A physician palpates an ovatian mass in a patient on pelwic examination Resection of the mass demonstrates a lymphoma composed of small lymphocytes with interspersed macrophages surrounded by clear spaces. This lesion would be most likely to be associated with a) abl-ber hybrid ) bel-2 activation ) e-mye activation 1, 22) et (14, 18) Answer [UEIRIRHBNY) Other Users Explanation Report An Esor Question Explanation: The disease is Burkitt lymphoma (the major clue is the "starry skey" pattem described) which occurs as a javr lesion in epidemic form in Aftica (associated with Epsicin-Barc virus) and in a sporadic form that usually involves the pelvic or abdominal organs. Burkitt Iymphoma is associated with c- mye activaton due to at (B, 14) translocation that places the c- myc containing segment of chromosome 8 near an actively transcribed gene for immanoglobulin heavy chains. + (9,22) (choice D) and abl-bcr hybrid (choice A) are associated with chronic myeloid leulcemia (CML) 1(14,18) (choice B) and bel-2 (choize B) are associated with follicular lymphomas, ao1 Burkitt lymphoma Marke this question >> Question Td : 84870 Question 12 of 30 ‘A physician palpates an ovatian mass in a patient on pelwic examination Resection of the mass demonstrates a lymphoma composed of small lymphocytes with interspersed macrophages surrounded by clear spaces. This lesion would be most likely to be associated with a) abl-ber hybrid ) bel-2 activation Y © 0) e-mye activation 1, 22) et (14, 18) Answer [UEIRIRHBNY) Other Users Explanation Report An Esor Question Explanation: The disease is Burkitt lymphoma (the major clue is the "starry skey" pattem described) which occurs as a javr lesion in epidemic form in Aftica (associated with Epsicin-Barc virus) and in a sporadic form that usually involves the pelvic or abdominal organs. Burkitt Iymphoma is associated with c- mye activaton due to at (B, 14) translocation that places the c- myc containing segment of chromosome 8 near an actively transcribed gene for immanoglobulin heavy chains. + (9,22) (choice D) and abl-bcr hybrid (choice A) are associated with chronic myeloid leulcemia (CML) 1(14,18) (choice B) and bel-2 (choize B) are associated with follicular lymphomas, ao1 Burkitt lymphoma 2125/2014 6:36:21 PM “Mark this question = Question Id : 85055, Question 13 of 30 A patienthas long-standing hemolyte anemia characterized by hypochromic cells. Hlectrophoresis studies demonstrate a near complete absence of beta chains. Several years later, the patient develops pulmonary edema, ascites, and peripheral edema, and complains that he feels weak all the time. The most ilcely contributing factor is the intracardiac deposit of which of the following? @) Amyloid deposition ) Calcium ©) Fibrous tissue 4) Iron 8) Sodium. Question Explanation: ‘The disease is beta thalassemia major, which is a severe hemolytic anemia characterized by a failure to produce the beta chains of hemoglobin resulting in unpaired alpha chains. The excess aipha chains ate insoluble, leading to intra and extravascular hemolysis. ‘These patients require large mumbers of transfisions and iron overload resuits in hemosiderosis that can contribute eventually to cardiac failure. The heartis also damaged by the chronic hich output state needed to compensate for the anor. “Amyloid deposition (choice A) in cardiac tissue can induce arrhythmias and restrictive heart physiology without significane hypertrophy. Ibis not associated with anemia and there is a near complete absence of beta chains on electrophoresis. Calcium (choice B) deposition is seen in damaged tissues and in states with high serum calcium such as hypeparathyroidism Fibrous tissue (choise C) is evidence of kecled tisoue of a myocardial infarction and forms 6 weeks post infarct. Sodium (choice ) is tighly soluble and does not usually precipitate in tissues 2125/2014 6:36:21 PM “Mark this question = Question Id : 85055, Question 13 of 30 A patienthas long-standing hemolyte anemia characterized by hypochromic cells. Hlectrophoresis studies demonstrate a near complete absence of beta chains. Several years later, the patient develops pulmonary edema, ascites, and peripheral edema, and complains that he feels weak all the time. The most ilcely contributing factor is the intracardiac deposit of which of the following? 2) Arnyleid deposition ) Cali ©) Fibrous tissue YO dion 6) Sodium Question Explanation: ‘The disease is beta thalassemia major, which is a severe hemolytic anemia characterized by a failure to produce the beta chains of hemoglobin resulting in unpaired alpha chains. The excess aipha chains ate insoluble, leading to intra and extravascular hemolysis. ‘These patients require large mumbers of transfisions and iron overload resuits in hemosiderosis that can contribute eventually to cardiac failure. The heartis also damaged by the chronic hich output state needed to compensate for the anor. “Amyloid deposition (choice A) in cardiac tissue can induce arrhythmias and restrictive heart physiology without significane hypertrophy. Ibis not associated with anemia and there is a near complete absence of beta chains on electrophoresis. Calcium (choice B) deposition is seen in damaged tissues and in states with high serum calcium such as hypeparathyroidism Fibrous tissue (choise C) is evidence of kecled tisoue of a myocardial infarction and forms 6 weeks post infarct. Sodium (choice ) is tighly soluble and does not usually precipitate in tissues 2125/2014 J % worw interface. edu.ck/i: * © C 0 www. interface.edu.pk/medical-exarns /test-analysis pho ?utd=14759 apps EQ Google A Settings [i Signin Cl imported Frome Bw slelgr dala © gy cell Saye » 2 other bookrmar ‘Maule this question Question Td: 85076] Total Questions «=> Question 14 of 30 a A.24 year old man presents with a 4 cin supraclavicular mass. CXR demonstrates marked mediastinal widening. Excisional biopsy of 3 the mass demonctrates areas with the appearance shown in the photomicrograph below. The most licely finding in the cell marked 4 with the anrow is 5 & 2 & 2 10 u 12 13 14 15 a) Down-regulation of NF-KB. 'b) Epstein-Barr virus genome in episome from. c) Mummification 4) Production of CD20 e) Somatic hypermutation Answer (Botha) Other User's Explanation Report An Error Question Explanatios The cell illustrated is a Reed-Sternberg cell. These cells characteristicaly are neoplastic giant cells with a bibbed mitror-image nucleus that may have large nucleoli surrounded by 2 clear halo. Most commonly, these cells are associated with Hodgkin lymphoma, and in the vast majority of cases, these neoplastic cells are derived from germinal center or post-germinal center B lymphocytes. Studies employing microdissection and analysis of single isolated Reed-Sternberg cells have shown thern to be B lymphocytes with evidence of somatic hypermmutation. Down-regulation of NE-KB (choice A) would not be expected in any of the forms of Hodgkin lymphoma, Instead, up-regulation of this transcription factor is found in almost all cases of classic Hodgkin lymphoma Epstein Barr virus genome in episome form (choice B) is found in 3 of the 5 types of Hodgkin lymphoma, but notin the most common type, nodular sclerosis, The EBV genome ss present in mixed celtanty, lymphocyte-tich, and lymphocyte depletion forms of Hodgkin lymphoma, but not in nodular sclerosis or Iymphocyte predominance forms. “Mammifcation (choice C)) is an umasual form of cell death that is observed in classic forms of Hodgkin lymphoma Such cells would bbe shrunken and pyknotic and act large and robust as the cells shown here. Production ofCD20 (choice D) is a selatively uncommon finding in Reed-Stemberg cells. Of the five subtypes of Hodgkin lymphoma, only the lymphocyte predominance form is characterized g by the possession of this marker This is an uncommon variant of the Pee ae ar re a em eer xxXxxXXKXKKXKXKXKXKXKXKRKXKXKXKKKKKKKKKKKK BEBAEREEBSESBEE bk 2125/2014 J % worw interface. edu.ck/i: * © C 0 www. interface.edu.pk/medical-exarns /test-analysis pho ?utd=14759 apps EQ Google A Settings [i Signin Cl imported Frome Bw slelgr dala © gy cell Saye » 2 other bookrmar ‘Maule this question Question Td: 85076] Total Questions «=> Question 14 of 30 a A.24 year old man presents with a 4 cin supraclavicular mass. CXR demonstrates marked mediastinal widening. Excisional biopsy of 3 the mass demonctrates areas with the appearance shown in the photomicrograph below. The most licely finding in the cell marked 4 with the anrow is 5 & 2 & 2 10 u 12 13 14 15 a) Down-regulation of NF-KB +b) Epstein-Barr virus genome in episome from c) Mummication d) Production of CD20 Y © e) Somatic kypermutation Answer (Botha) Other User's Explanation Report An Error Question Explanatios The cell illustrated is a Reed-Sternberg cell. These cells characteristicaly are neoplastic giant cells with a bibbed mitror-image nucleus that may have large nucleoli surrounded by 2 clear halo. Most commonly, these cells are associated with Hodgkin lymphoma, and in the vast majority of cases, these neoplastic cells are derived from germinal center or post-germinal center B lymphocytes. Studies employing microdissection and analysis of single isolated Reed-Sternberg cells have shown thern to be B lymphocytes with evidence of somatic hypermmutation. Down-regulation of NE-KB (choice A) would not be expected in any of the forms of Hodgkin lymphoma, Instead, up-regulation of this transcription factor is found in almost all cases of classic Hodgkin lymphoma Epstein Barr virus genome in episome form (choice B) is found in 3 of the 5 types of Hodgkin lymphoma, but notin the most common type, nodular sclerosis, The EBV genome ss present in mixed celtanty, lymphocyte-tich, and lymphocyte depletion forms of Hodgkin lymphoma, but not in nodular sclerosis or Iymphocyte predominance forms. “Mammifcation (choice C)) is an umasual form of cell death that is observed in classic forms of Hodgkin lymphoma Such cells would bbe shrunken and pyknotic and act large and robust as the cells shown here. Production ofCD20 (choice D) is a selatively uncommon finding in Reed-Stemberg cells. Of the five subtypes of Hodgkin lymphoma, only the lymphocyte predominance form is characterized g by the possession of this marker This is an uncommon variant of the Pee ae ar re a em eer xxXxxXXKXKKXKXKXKXKXKXKRKXKXKXKKKKKKKKKKKK BEBAEREEBSESBEE bk Mark this question —&= => Question Td : 85096 Question 15 of 30 A 16 year old Asian boy has a history of fever end increased abdominal gith, Radiographic examination reveals an ileocecal mass. ‘which is identified as a lymph node. Molecular studies on an abdominal lymph node containing lymphoma demonstrate t(2;8) (12;q24) translocation, The most likely diagnosis is a) Burkitt lymphoma 6) Mantie cel! Imphorna ©) Multiple myeloma 4) Stnall cell lymphoma ¢) Small cleaved cell lymphoma Ancwer (UBUIRIRAN oer ucors expan Question Explanation: Burkitt lymphoma is actually associated with three translocations. The comman variant (8; 14)(424;432), involing the oncogene myc on chromosome 8, and the heavy immunoglobulin chain on chromosome 14. The other two variants are: (8;22)(q24,9 11)" invoWving mye and the lambda ight chain immunoglobulin site and t(2:8)(p12;q24), involving the kappa light chain and myc Mantle cell lymphoma (choice B), multiple myeloma (choice C), and small (not cleaved) cell lymphome (choice D) are associated with the #(11; 14)(q 13,432) translocation involving bcl-1 and the heavy chain site Small cleaved cell lymphoma (choice E) is associated with (14;18)(q 32,q21.) involving the immunoglobulin chain site and bel-2. Report An Error Mark this question —&= => Question Td : 85096 Question 15 of 30 A 16 year old Asian boy has a history of fever end increased abdominal gith, Radiographic examination reveals an ileocecal mass. ‘which is identified as a lymph node. Molecular studies on an abdominal lymph node containing lymphoma demonstrate t(2;8) (12;q24) translocation, The most likely diagnosis is Y © a) Burkitt lymphoma 6) Mantie cel! Imphorna ©) Multiple myeloma 4) Stnall cell lymphoma ¢) Small cleaved cell lymphoma Ancwer (UBUIRIRAN oer ucors expan Question Explanation: Burkitt lymphoma is actually associated with three translocations. The comman variant (8; 14)(424;432), involing the oncogene myc on chromosome 8, and the heavy immunoglobulin chain on chromosome 14. The other two variants are: (8;22)(q24,9 11)" invoWving mye and the lambda ight chain immunoglobulin site and t(2:8)(p12;q24), involving the kappa light chain and myc Mantle cell lymphoma (choice B), multiple myeloma (choice C), and small (not cleaved) cell lymphome (choice D) are associated with the #(11; 14)(q 13,432) translocation involving bcl-1 and the heavy chain site Small cleaved cell lymphoma (choice E) is associated with (14;18)(q 32,q21.) involving the immunoglobulin chain site and bel-2. Report An Error 2/25:2014 6:37:08 PM Mark this question & => Question Td : 85116 Question 16 of 30 A 32 year old man presents with history of fevers and night sweats. CT scan reveals masses involving the mediastinal nodes, abdominal nodes, and ver. Biopsy of one of the nodes demonstrates a nodular lymphoma with aodular areas composed predominately of small (6-12 micrometer) lymphosytes with scant cytoplasm and irregular, twisted, and indented nuclei. This lymphoma is most closely linked to a translocation inolving @) Immunoglobulin heany chain and bel-1 ') Immunoglobulin heavy chain and bel-2 ©) Tnmunoglobutin heavy chain and mye 6) Kappa Immunoglobulin light chain and mye 8) Lambda Iinmmnoglobulin fight chain and mye Question Explanation: ‘The lymphoma described in the question stemn is the follicular type of stall cleaved cell lymphoma, and its characteristic translocation is t(14,18)(q32;q21). The site on chromosome 14 involves the heavy chain of immunoglobuline (gE), while that on chromosome 12 involves the bcl-2 oncogene. The translocation produces a hybrid bcl- 1 2fmmmumoglobuln heavy-chain transcript ‘At (11, 14) (q13, 432) translocaton involving bcl-1 and IgH (choice A) is seen in srrall non- cleaved lymphocyte lymphoma, myeloma, and some mantle cell lymphomas. Burkctt lymphoma is usually associated with t(8: 14)(myc and IgH. choice C). but sometimes with t(2;8) (capa immunoglobulin light chain and myc; choice D ) or t(8;22) (lambda immanoglobulin light chain and myc; choice B), n Report An Error 2/25:2014 6:37:08 PM Mark this question & => Question Td : 85116 Question 16 of 30 A 32 year old man presents with history of fevers and night sweats. CT scan reveals masses involving the mediastinal nodes, abdominal nodes, and ver. Biopsy of one of the nodes demonstrates a nodular lymphoma with aodular areas composed predominately of small (6-12 micrometer) lymphosytes with scant cytoplasm and irregular, twisted, and indented nuclei. This lymphoma is most closely linked to a translocation inolving @) Immunoglobulin heany chain and bel-1 Y¥ © 6) Immunoglobulin heavy chain and bel-2 ©) Tnmunoglobutin heavy chain and mye 6) Kappa Immunoglobulin light chain and mye 8) Lambda Iinmmnoglobulin fight chain and mye Question Explanation: ‘The lymphoma described in the question stemn is the follicular type of stall cleaved cell lymphoma, and its characteristic translocation is t(14,18)(q32;q21). The site on chromosome 14 involves the heavy chain of immunoglobuline (gE), while that on chromosome 12 involves the bcl-2 oncogene. The translocation produces a hybrid bcl- 1 2fmmmumoglobuln heavy-chain transcript ‘At (11, 14) (q13, 432) translocaton involving bcl-1 and IgH (choice A) is seen in srrall non- cleaved lymphocyte lymphoma, myeloma, and some mantle cell lymphomas. Burkctt lymphoma is usually associated with t(8: 14)(myc and IgH. choice C). but sometimes with t(2;8) (capa immunoglobulin light chain and myc; choice D ) or t(8;22) (lambda immanoglobulin light chain and myc; choice B), n Report An Error ‘Mark this question ¢-= Question 17 of 30 A.53 year old man presents with fevers, right sweats, and a recent 17 Ib weight loss. Medastinal lmmph nodes are promment and visible on routine chest X-ray. Lymph node biopsy shows prominent nodules and fibrosing bands separeting the nodules. The biopsy alse shows unacval celle with mulilobulated nuclei and abundant, pale cyteplacm producing an empty epace. They are surrounded by lymphocytes, plasma cells, and histiocytes. Above mentioned findings ase most consistent with which of ihe folowing diseases? a) Burkitt lymphoma ) Miked cellulaity Hodgkin disease ) Nodular sclerosing Hodkin disease «) Normal lymph node €) Predominately stall cleaved cell lnmphoma Answer | Explanation Other User's Explanation Report An Error Question Explanation ‘A characteristic cell of Hodgkin disease is the Reed Stembeng cel. The findings described specifically suggest the noduler sclerosing variant of Hodgkin lymphoma, Tn formalin ized tissue, the cytoplasm around Reed Stemberg cell mciei retracts, leaving a cleared space possibly spanned by a fow cytoplasmic strands, Broad collagen bands aro also seen in the biopay that represents the sclerosing component of the neoplasm, The actual tumor cell in this lesion is the lacunar cell, and classic Reed-Stemberg cells are difficult to find. In Burkitt lymphoma sheets of small lymphocytes are interspersed with larger histiocytes, producing a "starry sky" pattern. Tn tied celliarty Hodgkin disease, classic Reed-Stemberg cells ate easly identified and band-forming sclerosis is not a feature Lacunar cells would not be seen in a normal lymph node or in predomizately small cleaved cell raphoma, a non-Hodgkin iymphoma. ‘Mark this question ¢-= Question 17 of 30 ‘A.53 year old man preseats with fevers, sight sweats, and a recent 17 Ib weight loss, Mecastinal lymph nodes are promment and visible on routine chest X-ray. Lymph aode biopsy shows prominent nodules and fibrosing bands separating the nodules. The biopsy also shows unusual cells with mubilobulated auclei and abundant, pale cytoplasm producing an empty space. They are surcunded by lymphocytes, plasma cells, and histiocytes. Above mentioned findings are most consistent with which of the following diseases? a) Burkitt lymphoma +b) Mixed cellularity Hodgkin disease ¥ © 0) Nodular sclerosing Hodgkin disease d) Normal lymph node ©) Predominately sina cleaved cell lymphoma Answer | Explanation Other User's Explanation Report An Error Question Explanation ‘A characteristic cell of Hodgkin disease is the Reed Stembeng cel. The findings described specifically suggest the noduler sclerosing variant of Hodgkin lymphoma, Tn formalin ized tissue, the cytoplasm around Reed Stemberg cell mciei retracts, leaving a cleared space possibly spanned by a fow cytoplasmic strands, Broad collagen bands aro also seen in the biopay that represents the sclerosing component of the neoplasm, The actual tumor cell in this lesion is the lacunar cell, and classic Reed-Stemberg cells are difficult to find. In Burkitt lymphoma sheets of small lymphocytes are interspersed with larger histiocytes, producing a "starry sky" pattern. Tn tied celliarty Hodgkin disease, classic Reed-Stemberg cells ate easly identified and band-forming sclerosis is not a feature Lacunar cells would not be seen in a normal lymph node or in predomizately small cleaved cell raphoma, a non-Hodgkin iymphoma. % www. interface. adupk/i * € > @ DB www.interface.edu.pk/medical-examrs/test-analysis pho ®utid=14759 apps Elicoogle A seungs [synin Glimporedrromie S .. plier dutuw © on ye well ans >») other bookmar Mark this question <> Question Td: 86824 | Total Questions i Question 18 of 30 A.55 year old female presents with fatigue, early satiety, and a 30 Ib weight loss over the past 2 months. Routine blood studies show leukocytosis and thrombocytosis. A peripheral smezr is performed and is shown below. There are large numbers oP neutrophils and elevated numbers of notmal appearing eosinophils and basophils. No blast forms are seen. The cytogenetic abnormality that is associated with this condition is ho Wo 1 I ka IR ke a) bel-2 activation b) c-mye activation o)t(8, 14) At, 22) et (14, 18) Question Explanation: “The disease descnbed ss chrome myelogenous leukemia (CML), which usually affects adults between 40 and 59 years of age ‘The clinical hallmark of CML is the uncontrolled production of maturing granulocytes, which are predominantly neutrophils but eosinophils and basophils are also seen, CML is associated with the Philadelphia Ph) chromosome which is actually a translocation involving chromosomes 9 and 22 that produce an abl-bcr hybrid fusion protein, The fusion protein commonly possesses an oncogenic dysregulated kinase activity and is the hallmarke of CML. Diagnosis involves identifying the Ph chromosome or its products, the BCR- ABL fasion mRNA or the BCR-ARL protein, Bel-2 activation and at (14, 18) translocation are features of follicular lymphomas. In follicular lymphoma, the bel-2 oncogene is translocated from 18 to the Tg heavy chain locns on chromosome 14. The bcl-2 protein differs from oncogenes (which promote cellular proliferation) and tumor suppressor genes (which inhibit cell growth), in that the boh2 protein blocks apoptoss, leading to prolonged cell survival, C-myc activation and t (8, 14) translocation are featares of Burkitt lymphoma ln IG fe bs ks Ek I x xXxKXXXKKKKKKKKKKKKKKKKKKKKKKKKXK SR BARRERBE % www. interface. adupk/i * € > @ DB www.interface.edu.pk/medical-examrs/test-analysis pho ®utid=14759 apps Elicoogle A seungs [synin Glimporedrromie S .. plier dutuw © on ye well ans >») other bookmar Mark this question <> Question Td: 86824 | Total Questions i Question 18 of 30 A.55 year old female presents with fatigue, early satiety, and a 30 Ib weight loss over the past 2 months. Routine blood studies show leukocytosis and thrombocytosis. A peripheral smezr is performed and is shown below. There are large numbers oP neutrophils and elevated numbers of notmal appearing eosinophils and basophils. No blast forms are seen. The cytogenetic abnormality that is associated with this condition is ho Wo 1 I ka IR ke a) bel-2 activation 'b) c-myc activation o)t(8, 14) Y Od, 22) et (14, 18) Question Explanation: “The disease descnbed ss chrome myelogenous leukemia (CML), which usually affects adults between 40 and 59 years of age ‘The clinical hallmark of CML is the uncontrolled production of maturing granulocytes, which are predominantly neutrophils but eosinophils and basophils are also seen, CML is associated with the Philadelphia Ph) chromosome which is actually a translocation involving chromosomes 9 and 22 that produce an abl-bcr hybrid fusion protein, The fusion protein commonly possesses an oncogenic dysregulated kinase activity and is the hallmarke of CML. Diagnosis involves identifying the Ph chromosome or its products, the BCR- ABL fasion mRNA or the BCR-ARL protein, Bel-2 activation and at (14, 18) translocation are features of follicular lymphomas. In follicular lymphoma, the bel-2 oncogene is translocated from 18 to the Tg heavy chain locns on chromosome 14. The bcl-2 protein differs from oncogenes (which promote cellular proliferation) and tumor suppressor genes (which inhibit cell growth), in that the boh2 protein blocks apoptoss, leading to prolonged cell survival, C-myc activation and t (8, 14) translocation are featares of Burkitt lymphoma ln IG fe bs ks Ek I x xXxKXXXKKKKKKKKKKKKKKKKKKKKKKKKXK SR BARRERBE Mark this question e& => Question Td : 86864 Question 19 of 30 ‘The most common site for development of embryonal rhabdomyosarcoma is 2) Gastrointestinal tract ) Head and neck ©) Kidneys, ) Liver e) Lungs Answer | Baplanation Other User's Explanation Report An Error Question Explanation: Embryoral rhabdomyosarcoma is the most common form of thabdomyosarcome, The tuner is composed predominantly of smal, round cells resembling cells found in developing muscle. The head and neck (particularly the nose, nasopharyn, and orbit) is the most common site for the development of embryonal shabdomyosarcoma, Other favored sites are the genitourinary tract (e.g, sarcoma botyroides, or embryonal thabdomyosarcoma of the vagina) and the exiremities. Mark this question e& => Question Td : 86864 Question 19 of 30 ‘The most commen site for development of embryonal rhabdomyosarcoma is 2) Gastrointestinal tract Y © ») Head and neck, ©) Kidneys 4) Liver e) Lungs Answer | Baplanation Other User's Explanation Report An Error Question Explanation: Embryoral rhabdomyosarcoma is the most common form of thabdomyosarcome, The tuner is composed predominantly of smal, round cells resembling cells found in developing muscle. The head and neck (particularly the nose, nasopharyn, and orbit) is the most common site for the development of embryonal shabdomyosarcoma, Other favored sites are the genitourinary tract (e.g, sarcoma botyroides, or embryonal thabdomyosarcoma of the vagina) and the exiremities. 2:25/2014 6:38:08 PM “Mark this queston €&=> Question Id : 118583 Question 20 of 30 ‘Tumor thatis often curable by chemotherapy is which one ofthe following? 2) Acute lymphocytic leukemia ) Kaposi’s sarcoma c) Prostate cancer Aswooyioma €) Melanoma Answer (Explanation Other User's Explanation Report An Error Question Explanation: “Acute lymphocytic leuicemia is often curable by chemotherapy. Unfortunately, the same is not tre for the other tumors listed inclnding melanoma, Prostate cancer usually responds to radical surgery or radiation therapy. though the effectiveness varies with the age of the patient and severity of disease, Astrocytoma may require radiation therapy or resection Current methods of treatment for Kaposi's sarcoma usually do not significantly prolong the patient's life. 2:25/2014 6:38:08 PM “Mark this queston €&=> Question Id : 118583 Question 20 of 30 ‘Tumor that is often curable by chemotherapy is which one of the following? Y © 8) Acate iympheoytic leukemia b) Kaposi’s sarcoma ©) Prostate cancer 4) Asirocyioma ©) Melanoma Answer (Explanation Other User's Explanation Report An Error Question Explanation: “Acute lymphocytic leuicemia is often curable by chemotherapy. Unfortunately, the same is not tre for the other tumors listed inclnding melanoma, Prostate cancer usually responds to radical surgery or radiation therapy. though the effectiveness varies with the age of the patient and severity of disease, Astrocytoma may require radiation therapy or resection Current methods of treatment for Kaposi's sarcoma usually do not significantly prolong the patient's life. ‘Marke this question e& => Question Ta : 126330 Question 21 of 30 FALSE statement regarding Rh immune system is which one of the following? a) Rho (D) is the mostimmunogenic alloantigen ) Rh-nezative blood lacks the D antigen, ©) A single unit of Rh postive blood cells wransfused into an Rh negative individual resalt in anti-D antibodies in 70-80% of persors, 9) To avoid hemolytic disease of the newbom, a nonimmunized premenopausal Rh negative woman must not receive Rh- postive blood products ©) Bh antigens Cand E are more immunogeric than D. Answer | Banat) Other User's Explanation Report An Error Question Explanation: The Rh antigens C, , E, and e are considerably less immunogenic than D. The Rhesus (Rh) system is of particular importance in pregnency. The Rho (D) is the most immunogenic alloantigen Persons lacking the D antigen are classified as Rh-negative. “Approximately 70- 80% of Rh-negative persons who are transfused with a single unit of Rh-positive red blood cells will develop anti-D. This antibody may result in a severe transfusion reaction or hemolytic disease of the newbom. A premenopausal Rh-negative woman who receives Rh-positive blood components without adequate immunization is llcely to deliver a child with hemolytic disease of the newborn ina subsequent pregnancy ifthat child is Rh positive, dus to stimulation of a secondary immune response ‘Marke this question e& => Question Ta : 126330 Question 21 of 30 FALSE statement regarding Rh immune system is which one of the following? a) Rho (D) is the most immunogenic alloantigen ) Rh-nezative blood lacks the D antigen, ©) A single unit of Ra postive blood cells transfused into an Rh negative individual result in anti-D antibodies in 70-80% of persons. ) To avoid hemalytic dissase of the newbom, a sonimmunized premenopausal Rh negetive woman must not seceive Rh- postive blood products SY © ©) Bhantigens C and E are more immunogenic than D. Answer | Banat) Other User's Explanation Report An Error Question Explanation: The Rh antigens C, , E, and e are considerably less immunogenic than D. The Rhesus (Rh) system is of particular importance in pregnency. The Rho (D) is the most immunogenic alloantigen Persons lacking the D antigen are classified as Rh-negative. “Approximately 70- 80% of Rh-negative persons who are transfused with a single unit of Rh-positive red blood cells will develop anti-D. This antibody may result in a severe transfusion reaction or hemolytic disease of the newbom. A premenopausal Rh-negative woman who receives Rh-positive blood components without adequate immunization is llcely to deliver a child with hemolytic disease of the newborn ina subsequent pregnancy ifthat child is Rh positive, dus to stimulation of a secondary immune response 2:25/2014 6:38:39 PM J € ‘www interface.edupk/i x \\_ CL www. interface edu.pk/med est-analysis.php tutid=14759 apps Elicooge A seungs [)synin Cl imported rromie So. wllbe tortie 92 caull Bans Ww Gi other bookmar ‘Maute this question & => Question 22 of 30 Question Td : 188265 A S6-year-old has a ventilation-perfision defect in the right lung and a positive D-dimer assay: Lawer-extremity ultrasound stucies demonstrate a thrombus on the right side. She is given oxygen and warfarin loading dose. Heparin is not utilized duc to a history of “heparin allergy." Four days later, the respiratory stazus improves and tendemess in her right lez decreases, but she develops pain and redness in her left breast. The breast has erythema and tenderness of the left breast and right thigh with a central area of scar formation, Microscopic examination reveals extensive thrombosis with microvascular injury and fbbrin deposits in the postcapilary venules and small veins. Loss of finction of which of the following most likely underies this patient's new complaint? a) Factor VII ) Factor IX c) Platelets 4) Protein C ¢) Von Willebrand factor Question Explanation: This patient has suffered a pulmonary embolism secondary to a deep vein thrombosis in her right lower extremity, Because she claimed an allergy to heparin, she was anticoagulated with warfarin alone. Warfarin functions as an anticcagulent by depleting all of the vitamin K-cependent clotting factors, namely, factors TI (prothrombin), WTI, TX, and X€ The anticoagulant proteins C! and S are also vitamin K-depeadent, and treatment with warfarin will decrease the circulating levels of these proteins as well. The circulating half-life of protein Cis much shorter than that of any of the doffing factors, so early in the course of warfarin therapy a hhypercoagulable state exists. For this reason, patients starting warfarin therapy are often treated with low molecular-weight heparin as ‘well uni their INR is within the target range. This procoagulant state predisposes to cutaneons necrosis resilting from thrombosis and occlusion of the dermal vasculature. The most common sites for warfarin induced skin necrosis are areas with considerable subcutaneous fat, such as the breasts, hips, and thighs Factor VIlis a vitamin-K dependent clotiing factor that together with issue factor plays a key role in amplifying the doffing cascade Inhibition of this factor isthe goal of warfarin therapy. Factor [i is also a vitamin-K dependent factor that forms part of the intrinsic offing cascade. Inhibition of this factor is the goal of warfarin therapy. Tts deficiency causes hemophilia B. Platelets are not inhibited by warfarin or heparin, Antiplatelet agents include aspirin, clopidogrel, and glycoprotein I/LIa inhibitors ‘Von Willebrand factor is a gycoprotein involved m hemostasis and produced m endotheluma, megakaryocytes, and subendothelial connective tissue. Its deficiency is associated with spontaneous bleeding from musous membranes. prolonged bleeding from wounds, and menorrhagia in young females, Von Willebrand is not inhibited by warfarin Total Questions 1 bbe BEBE be BEBS 8 x xXKXXXKXKKXKKXKKXKKKKKKKKRKKRKKRKXKKKXKXK 2:25/2014 6:38:39 PM J € ‘www interface.edupk/i x \\_ CL www. interface edu.pk/med est-analysis.php tutid=14759 apps Elicooge A seungs [)synin Cl imported rromie So. wllbe tortie 92 caull Bans Ww Gi other bookmar ‘Maute this question & => Question 22 of 30 Question Td : 188265 A S6-year-old has a ventilation-perfision defect in the right lung and a positive D-dimer assay: Lawer-extremity ultrasound stucies demonstrate a thrombus on the right side. She is given oxygen and warfarin loading dose. Heparin is not utilized duc to a history of “heparin allergy." Four days later, the respiratory stazus improves and tendemess in her right lez decreases, but she develops pain and redness in her left breast. The breast has erythema and tenderness of the left breast and right thigh with a central area of scar formation, Microscopic examination reveals extensive thrombosis with microvascular injury and fbbrin deposits in the postcapilary venules and small veins. Loss of finction of which of the following most likely underies this patient's new complaint? a) Factor VII ) Factor IX c) Platelets V © d)Protein C ¢) Von Willebrand factor Question Explanation: This patient has suffered a pulmonary embolism secondary to a deep vein thrombosis in her right lower extremity, Because she claimed an allergy to heparin, she was anticoagulated with warfarin alone. Warfarin functions as an anticcagulent by depleting all of the vitamin K-cependent clotting factors, namely, factors TI (prothrombin), WTI, TX, and X€ The anticoagulant proteins C! and S are also vitamin K-depeadent, and treatment with warfarin will decrease the circulating levels of these proteins as well. The circulating half-life of protein Cis much shorter than that of any of the doffing factors, so early in the course of warfarin therapy a hhypercoagulable state exists. For this reason, patients starting warfarin therapy are often treated with low molecular-weight heparin as ‘well uni their INR is within the target range. This procoagulant state predisposes to cutaneons necrosis resilting from thrombosis and occlusion of the dermal vasculature. The most common sites for warfarin induced skin necrosis are areas with considerable subcutaneous fat, such as the breasts, hips, and thighs Factor VIlis a vitamin-K dependent clotiing factor that together with issue factor plays a key role in amplifying the doffing cascade Inhibition of this factor isthe goal of warfarin therapy. Factor [i is also a vitamin-K dependent factor that forms part of the intrinsic offing cascade. Inhibition of this factor is the goal of warfarin therapy. Tts deficiency causes hemophilia B. Platelets are not inhibited by warfarin or heparin, Antiplatelet agents include aspirin, clopidogrel, and glycoprotein I/LIa inhibitors ‘Von Willebrand factor is a gycoprotein involved m hemostasis and produced m endotheluma, megakaryocytes, and subendothelial connective tissue. Its deficiency is associated with spontaneous bleeding from musous membranes. prolonged bleeding from wounds, and menorrhagia in young females, Von Willebrand is not inhibited by warfarin Total Questions 1 bbe BEBE be BEBS 8 x xXKXXXKXKKXKKXKKXKKKKKKKKRKKRKKRKXKKKXKXK 2/25:2014 6:38:54 PM “Mark this question —& => Question 23 of 30 ‘Which offthe following ss least likely to precipitate an acute attack in porphyria? a) Aspirin b) Menstruation ©) Pheaytoin ) Starvation ©) Thicpentone Question Explanatios Porphstia is a group of diseases characterized by excess production an excretion of porphyrins and their precursors. They are caused by enzyme defects within the haem metabclic pathway. Stress, infection, pregnancy, menstruation, starvation and cestain daugs may precipitate acute attacks. Definite precipitants include sulphonamices, barbiturates and phenytoin 2/25:2014 6:38:54 PM “Mark this question —& => Question 23 of 30 “Which ofthe following is least lcely to precipitate an acute attack in porplayria? SV © a) Aspirin b) Menstruztion ©) Pheaytoin 4) Starvation ¢) Thiopentone Question Explanatios Porphstia is a group of diseases characterized by excess production an excretion of porphyrins and their precursors. They are caused by enzyme defects within the haem metabclic pathway. Stress, infection, pregnancy, menstruation, starvation and cestain daugs may precipitate acute attacks. Definite precipitants include sulphonamices, barbiturates and phenytoin Merk this question & => Question Td : 202451 Question 24 of 30 A aitl aged 17 years with von Willebrand's disease is schedaled for dental extraction, Previous extraction resuted in bleeding that had required two unit transfusion. The most appropriate treatment prior to deatal surgery is a) Cryoprecipitate +) DDAVP ©) Fresh froren plastna 6) High purity factor VI concertrate ¢) Recombinant factor VII concentrate newer [REIIIRHBN) Othe: Users Explanation Repost An Esor Question Explanation: DDAVP is the choice treatment for mild von Willebrand's disease, which would include Type I, and the majority of Type TI, although there is some controversy in Type IIB as itis thought that DDAVP can exacerbate thrombocytopaenia that can accompany this type of von Willebrand's Itis ofno use in Type IIT - severe von wilebrend’s disease. The history tells us that she has mild disease. You would not use cryoprecipitate or fresh frozen plasma in these patients in this era due to potential viral transmission risk from blood products. For severs disease you would use a Von wilebrand factor conceatrate, not factor VIII concentrate Merk this question & => Question Td : 202451 Question 24 of 30 Agirl aged 17 years with von Willebrand's disease is schedaled for dental extraction. Previous extraction resuited in bleeding that had required two uit rensfasion, The most appropriate treatment prior to dental surgery is a) Cryoprecipitate v © b) DDAVP ©) Fresh frozen plasma 6) High purity factor VID concentrate ©) Recombinant factor VII concentrate newer [REIIIRHBN) Othe: Users Explanation Repost An Esor Question Explanation: DDAVP is the choice treatment for mild von Willebrand's disease, which would include Type I, and the majority of Type TI, although there is some controversy in Type IIB as itis thought that DDAVP can exacerbate thrombocytopaenia that can accompany this type of von Willebrand's Itis ofno use in Type IIT - severe von wilebrend’s disease. The history tells us that she has mild disease. You would not use cryoprecipitate or fresh frozen plasma in these patients in this era due to potential viral transmission risk from blood products. For severs disease you would use a Von wilebrand factor conceatrate, not factor VIII concentrate 2125/2014 6:39:24 PM Mark this question & => Question 25 of 30 A.17 year old gl with non-Hodgkin's lymphoma underwent splenectomy. She has an enhanced risk of developing pneumococcal sepsis and wishes to know how long this ris would persist. The duration of the tisk is, a) Up to six months ') Up to one year ¢) Up to five years 4) Five to 10 years €) More than 10 years Question Explanation: ‘The risk is thought to persist lifelong, and lifelong penicillin prophylaxis is recommended. n Report An Error 2125/2014 6:39:24 PM Mark this question & => Question 25 of 30 A.17 year old gl with non-Hodgkin's lymphoma underwent splenectomy. She has an enhanced risk of developing pneumococcal sepsis and wishes to know how long this ris would persist. The duration of the tisk is, 2) Up to six months b) Up to one year ©) Up to five years d) Five to 10 years Y © €) Mere than 10 years Question Explanation: ‘The risk is thought to persist lifelong, and lifelong penicillin prophylaxis is recommended. n Report An Error 2125/2014 63944 PM “Mark this question => Question Id : 203007 Question 26 of 30 A.55 year old mele was admitted for a total hip replacement. Physical exam was normal. Routine pre-operative blood study showed Hb of 11 g/dL and WBC count of 25 x10°/L with Inmohocyte court of 19 x10°/L (1.5-4 x10"), Blood fim shows mature lymphocytes. The most appropriate initial management is a) Cancel the patient's operation 6) Chlorambucil ©) Fludarabine ) Observation ©) Frednisolone tomes EEE 08 = nse Question Explanation: ‘The most signiicant abnormality on the fll blood count is the lymphocytosic, with mature lymphocytes seen on filn. Tn this age group the most likely diagnosis is « low grade Iymphoproliferative disorder, for example, chronic Iyinphocytic leulcaesniz, This, as mentioned, is a low grade condition, and does not require immediate treatment; patients undergo a period of observation often quite Jong, before any treatment is indicated, The indication for treatment would include: 1. Disabling B symptoms 2. Lymphocyte doubling time of less than 6 months 3, Bone masrow compromise 4, Autoimmune haemolysis or immune thrombocytopaenia He is never going to be cured from this condition, and therefore it would not be necessary to delay/ cancel surgery. He may be slightly more at risk ofinfection, due to immune dysfunction that accompanies these conditions, and the surgeons should be aware of this, jon Report An Error 2125/2014 63944 PM “Mark this question => Question Id : 203007 Question 26 of 30 4.55 year old male was admitted for a total hip replacement. Physical exam was normal. Routine pre-operative blood study showed Hb of 11 g/dL and WBC count of 25 x10°/L with Inmohocyte court of 19 x10°/L (1.5-4 x10"), Blood fim shows mature lymphocytes. The most appropriate initial management is a) Cancel the patient’s operation +b) Chlorambucil c) Fudarabine Y © & Observation e) Frednisolone tomes EEE 08 = nse Question Explanation: ‘The most signiicant abnormality on the fll blood count is the lymphocytosic, with mature lymphocytes seen on filn. Tn this age group the most likely diagnosis is « low grade Iymphoproliferative disorder, for example, chronic Iyinphocytic leulcaesniz, This, as mentioned, is a low grade condition, and does not require immediate treatment; patients undergo a period of observation often quite Jong, before any treatment is indicated, The indication for treatment would include: 1. Disabling B symptoms 2. Lymphocyte doubling time of less than 6 months 3, Bone masrow compromise 4, Autoimmune haemolysis or immune thrombocytopaenia He is never going to be cured from this condition, and therefore it would not be necessary to delay/ cancel surgery. He may be slightly more at risk ofinfection, due to immune dysfunction that accompanies these conditions, and the surgeons should be aware of this, jon Report An Error 2/25/2014 6:40:00 PM ‘Mark this question & => Question Td : 203067 Question 27 of 30 A 60 year old male presented with a flu-like illness which has subsided but FBC revealed a platelet count of 800 x109/L which has remained persistently elevated but with no other abnormality on the FBC. Heis otherwise asymptomatic and no abnormalities are noted on exam The most appropriate treatment for him is a) Anegrelide ) Aspirin ©) Hydroxycarbamide ©) Observation 6) Plateletpheresis Answer (| Birisnaton) Other User's Explanation Report An Error Question Explanation: There are a number of adverse prognostic markers for essential thrombocythaemia (ET) * Age above 60 + Symptomatology -particularly thrombosis and © Platelet count above 1500. Generally the prognosis is extremely good in ET with survival of over two decades expected. This patient would be regarded as low risk and hence observation only employed. ‘The risk of bleeding can also be a problem and although you may think thar aspirin would be appropriate the evidence is conflicting. 2/25/2014 6:40:00 PM ‘Mark this question & => Question Td : 203067 Question 27 of 30 A 60 year old male presented with a flu-like illness which has subsided but FBC revealed a platelet count of 800 x109/L which has remained persistently elevated but with no other abnormality on the FBC. Heis otherwise asymptomatic and no abnormalities are noted on exam The most appropriate treatment for him is a) Anegrelide ) Aspirin ©) Hydroxycarbamide Y © PD Observation 6) Plateletpheresis Answer (| Birisnaton) Other User's Explanation Report An Error Question Explanation: There are a number of adverse prognostic markers for essential thrombocythaemia (ET) * Age above 60 + Symptomatology -particularly thrombosis and © Platelet count above 1500. Generally the prognosis is extremely good in ET with survival of over two decades expected. This patient would be regarded as low risk and hence observation only employed. ‘The risk of bleeding can also be a problem and although you may think thar aspirin would be appropriate the evidence is conflicting. 2:25/2014 6:10:12 PM Mark this question => Question Td : 203554 Question 28 of 30 A 30-year-old woman presents with jaundice and her investigations reveal Haemoglobin [9.0 g/dl. 1.5-16.5) [Reticulocyte count|180 x109/L (25-852109) (Scrum tilirubin [S0umolL (1-22) Her blood film reveals the presence of spehrocytes Which of the following is the next most useful investigation? 2) abdominal ultrasound scan 6) direct antiglobulia test ©) glucose-6-phosphate dehydrogenase activity ) haemoglobin electrophoresis ) red cell osmotic agility Anowor [REIBIRNBNY other User's Explanat Question Explanation: ‘The resuits given indicate a haemoloytic anaemia of which spherocytes are typical and given the age of patient the most likely cause is jmunune. Therefore the mos} usefil testis the Direct Autiglobulin Test, n Report An Error 2:25/2014 6:10:12 PM Mark this question => Question Td : 203554 Question 28 of 30 A 30-year-old woman presents with jaundice and her investigations reveal Haemoglobin [9.0 g/dl. 1.5-16.5) [Reticulocyte count|180 x109/L (25-852109) (Scrum tilirubin [S0umolL (1-22) Her blood film reveals the presence of spehrocytes Which of the following is the nest most useful investigation? a) abdominal ultrasound scan Y © b) direct antigiobulia test ©) gucose-5-phosphate dehydrogenase activity 4) haemoglobin electrophoresis ¢) red cell osmotic fragility Anowor [REIBIRNBNY other User's Explanat Question Explanation: ‘The resuits given indicate a haemoloytic anaemia of which spherocytes are typical and given the age of patient the most likely cause is jmunune. Therefore the mos} usefil testis the Direct Autiglobulin Test, n Report An Error 2i25'2014 6:40:26 PM ‘Mark this question = => Question Td : 203569 Question 29 of 30 A.50 year old man has abistory of lethargy and epistaxis over the last one month, Exam reveals numerous bruises over arms and legs, splenomegaly and retinal hasmorshages. His Hp is 7 g/dL, WBC countis 14 x109/L and platelet count is 20x109/L. Blood film showss white cells predominantly myeloblasts and promyelocytes. Investigation that would be of most prognostic value is a) Bone marrow aspiration ) Bone marrow trephine biopsy ¢) Cerebrospinal uid exarination a) Cytogenetic karyotype 2) Immunophenetyping Question Explanation: The history. fill blood count results and the blood film is suggestive of Acute Myeloid Leukaemia, as there are numerous myeloblasts onfilm. Ofthe answers given the Cytogenetic Karyotype is of most prognostic value. The cytogenstic karyotype divides people into see categories. Good Risk, Standard Risk and Poor Risk. 2i25'2014 6:40:26 PM ‘Mark this question = => Question Td : 203569 Question 29 of 30 A.50 year old man has abistory of lethargy and epistaxis over the last one month, Exam reveals numerous bruises over arms and legs, splenomegaly and retinal hasmorshages. His Hp is 7 g/dL, WBC countis 14 x109/L and platelet count is 20x109/L. Blood film showss white cells predominantly myeloblasts and promyelocytes. Investigation that would be of most prognostic value is a) Bone marrow aspiration ) Bone marrow trephine biopsy ¢) Cerebrospinal uid exarination Y © &) Cytogenetic karyotype 2) Immunophenetyping Question Explanation: The history. fill blood count results and the blood film is suggestive of Acute Myeloid Leukaemia, as there are numerous myeloblasts onfilm. Ofthe answers given the Cytogenetic Karyotype is of most prognostic value. The cytogenstic karyotype divides people into see categories. Good Risk, Standard Risk and Poor Risk. 2:25/2014 6:40:40 PM T wen vietawodaah * Vo TSIST ie ¢ www. interface edu.pk/med xarns /test-analy' we Apps [El.cosgle Settings [) Signin Ci imported From iE 2. wslelsr tusluw oe Lal Baye > Ci other bookmar ‘Mark this question = Question 1d: 217240] Total Questions f Question 30 of 30 4 A 26-year-old woman for the past 2 weeks has had small, red, non-blanching, non-pruritic spots on her arms and legs. Additionally, 3 she now has easy bruising and occasional nose bleeds. She takes no medications aside. Labs show: 4 fen 12.5 gid 4 [Lenkoeyte comnt:[6000/mm 6 [Piatelets [20000%mm z [prr: [24 seconds| a [pr [10 seconds 2 Petipheral blood smear, basic chemistries and electrolytes, and tests of liver function are unremarkable, Based on this woman's i u presentation, which of the following is the mostlikely diagnosis? a) Disseminated intravascular coagulation, ) Idiopathic thrombocytopenic purpura ©) Scurvy ) Thrombotic thrombocytopenic purpura ©) Vitamin K deficiency Von Willebrand disease Question Explanation: Idiopathic thrombocytopenic purpura is defined by an isolated decline in platelet counts without any abnormalties in other cell linsages in a setting in which thero is no other obvious condition that may cance thrombocytopenia. Patients classicelly present with signs of platelet deficiency, such as peteckiae, gingival bleeding or epistaxis, or easy bmuising. Bone marrow shows megakaryocytes which indicate platelet; destruction rather than a malfunction of platelet production. inital treatment begins with steroids and, if necessary intravenous immunoglabutin Disseminated intravascular coagulation is unusval is unusual in patients who do net seem toxic. Laboratory evidence would include schistocytes seen on the peripheral blood smear, anemia, and elevated coagulation times. Scurvy is the disease of bleeding gums caused by vitamin C deficiency Bleeding time is increased in these patients; however, clotting profile ac well platelet countis normal Thrombotic thrombocytopenia purpura presents with fever, anemia, thrombocytopeni, renal failure, and neurologic changes, not isclated thombocytopenia ‘Vitamin K deficiency would present with a similar picture (easy bruisability. multiple petechiae). The clotting factors require vitamin K for eyntheric and hence doffing pathwaye are aifected reculting in increased PT and PU. Platelet count would be expected to be normal ‘Von Willebrand disease results in a sinilar presentation (mucosal bleeding) but normal platelet courts. The main functional deficiency ie in von Willebrand factor. a kev component of clot stabilization. BESBREBBEBKKEEKGEKELE AAA RAMA AAA AAA AAA RAR aR a a 2:25/2014 6:40:40 PM T wen vietawodaah * Vo TSIST ie ¢ www. interface edu.pk/med xarns /test-analy' we Apps [El.cosgle Settings [) Signin Ci imported From iE 2. wslelsr tusluw oe Lal Baye > Ci other bookmar ‘Mark this question = Question 1d: 217240] Total Questions f Question 30 of 30 4 A 26-year-old woman for the past 2 weeks has had small, red, non-blanching, non-pruritic spots on her arms and legs. Additionally, 3 she now has easy bruising and occasional nose bleeds. She takes no medications aside. Labs show: 4 fen 12.5 gid 4 [Lenkoeyte comnt:[6000/mm 6 [Piatelets [20000%mm z [prr: [24 seconds| a [pr [10 seconds 2 Petipheral blood smear, basic chemistries and electrolytes, and tests of liver function are unremarkable, Based on this woman's i u presentation, which of the folowing is the mostlilcely diagnosis? a) Disseminated intravascular coagulation V © b) Idiopathic thrombocytopenic purpura c) Scurvy 4) Thrombotic thrombocytopenic purpura e) Vitamin K deficiency £) Von Willebrand disease Question Explanation: Idiopathic thrombocytopenic purpura is defined by an isolated decline in platelet counts without any abnormalties in other cell linsages in a setting in which thero is no other obvious condition that may cance thrombocytopenia. Patients classicelly present with signs of platelet deficiency, such as peteckiae, gingival bleeding or epistaxis, or easy bmuising. Bone marrow shows megakaryocytes which indicate platelet; destruction rather than a malfunction of platelet production. inital treatment begins with steroids and, if necessary intravenous immunoglabutin Disseminated intravascular coagulation is unusval is unusual in patients who do net seem toxic. Laboratory evidence would include schistocytes seen on the peripheral blood smear, anemia, and elevated coagulation times. Scurvy is the disease of bleeding gums caused by vitamin C deficiency Bleeding time is increased in these patients; however, clotting profile ac well platelet countis normal Thrombotic thrombocytopenia purpura presents with fever, anemia, thrombocytopeni, renal failure, and neurologic changes, not isclated thombocytopenia ‘Vitamin K deficiency would present with a similar picture (easy bruisability. multiple petechiae). The clotting factors require vitamin K for eyntheric and hence doffing pathwaye are aifected reculting in increased PT and PU. Platelet count would be expected to be normal ‘Von Willebrand disease results in a sinilar presentation (mucosal bleeding) but normal platelet courts. The main functional deficiency ie in von Willebrand factor. a kev component of clot stabilization. BESBREBBEBKKEEKGEKELE AAA RAMA AAA AAA AAA RAR aR a a 2/25/2014 6: ‘Mark this question => Question Ti : 9656 07 PM Question 1 of 10 A70-year-old man with one-month old dyspnea and 3kg weight loss has a large left pleural effusion confirmed on chest X-ray. Pleural fluid analysis reveals a Protein of 39 g/L woth a few lymphocytes and red blood cells. Which one of the fellowing investigetions should be considered next? a) Bronchoscopy £) Repeat pleural aspiration with biopsy ©) CT scan pleural aspiration with biopsy 6) Thoracoscopic pleural biopsy ©) Tuberculin test Question Explanation: ‘We assume from the giving information that the paticat is not symptomatic - in which case a further therapeutic pleural aspiration should be performed. The next most step would be a contrast CT thorax which can help differentiate benveen benign and malgnant disease and also guide futher investigations Video-assisted thoracoscopic surgery (VATS) can Report An Error 1. Do good pleural biopsies 2. Clear all the pleural fluid 3. Allow pleurodesis to prevent recurrence Ti should be considered after other non-invasive tests have proven negative, Tevally the surgeons can do bronchoscopy at the same tine under general anesthetic. Percutancous pleural biopsies do net produce good samples and are less often done. 2/25/2014 6: ‘Mark this question => Question Ti : 9656 07 PM Question 1 of 10 A.70-year-old man with one-month old dyspnea and 3kg weight loss has a large let pleural effusioa confirmed on chest X-ray. Pleural flid analysis reveals a Protein of 39 g/L with a few lymphocytes and red blood cells. Which one of the following investigations should be considered nes? a) Bronchoscopy +) Repeat pleural aspiration with biopsy © © CT scan pleural aspiration with biopsy ©) Thoracoscopic pleural biopsy ©) Tuberculin test Question Explanation: ‘We assume from the giving information that the paticat is not symptomatic - in which case a further therapeutic pleural aspiration should be performed. The next most step would be a contrast CT thorax which can help differentiate benveen benign and malgnant disease and also guide futher investigations Video-assisted thoracoscopic surgery (VATS) can Report An Error 1. Do good pleural biopsies 2. Clear all the pleural fluid 3. Allow pleurodesis to prevent recurrence Ti should be considered after other non-invasive tests have proven negative, Tevally the surgeons can do bronchoscopy at the same tine under general anesthetic. Percutancous pleural biopsies do net produce good samples and are less often done. ‘Marke this question & => Question 2 of 10 Positron emission tomography (PET) scans used to detect cancer most commonly use radioactive tracer tagged to a molecule that is an analogue of which of the following? 2) oxygen 6) Glucose ©) Hemoglobin 6) Nitrate ©) Phosphate Anowor [UERVIRNRHAN) otter Usovs Explanation Report An Error Question Explanation: The fluorine. 18 labeled tracer Suorodeoxyglacose (EDG) is a ghucose 4 analogue taleen up by metabolically active cells using glucose as a substrate for their metabolism, This enables the PET scanner io deteci metabolically active tissues such as cancer metastases ‘Marke this question & => Question 2 of 10 Positron emission tomography (PET) scans used to detect cancer most commonly use radioactive tracer tagged to a molecule that is an analogue of which of the following? 2) oxygen Y © 6) Ghicose ©) Hemoglobin 6) Nitrate ©) Phosphate Anowor [UERVIRNRHAN) otter Usovs Explanation Report An Error Question Explanation: The fluorine. 18 labeled tracer Suorodeoxyglacose (EDG) is a ghucose 4 analogue taleen up by metabolically active cells using glucose as a substrate for their metabolism, This enables the PET scanner io deteci metabolically active tissues such as cancer metastases 2:25/2014 6:42:42 PM Mark this question & => Question Id : 57811 Question 3 of 10 Lymphadenopathy is most likely to be malignant at which of the following sites of head and neck? 4) Anterior cervical ) Supraclavioular ©) Postetior cervical ) Preauricular ¢) Submandibular Answer | Exsianation | Other User's Explanation Report An Error Question Explanation: dn patents with head and neck lymphadenopathy, supraclavicular nodes are the most likely to be malignant. Lymphadenopathy of these nodes should always be investigated, even in children. Overall, the prevalence of malignancy with this presentation is unknown, but rates of 54-85% have been scen in biopsy series reports. 2:25/2014 6:42:42 PM Mark this question & => Question Id : 57811 Question 3 of 10 Lymphadenopathy is most likely to be malignant at which of the following sites of head and neck? 4) Anterior cervical Y © }) Supractavicular ©) Postetior cervical ) Preauricular ¢) Submandibular Answer | Exsianation | Other User's Explanation Report An Error Question Explanation: dn patents with head and neck lymphadenopathy, supraclavicular nodes are the most likely to be malignant. Lymphadenopathy of these nodes should always be investigated, even in children. Overall, the prevalence of malignancy with this presentation is unknown, but rates of 54-85% have been scen in biopsy series reports. 2125/2014 6:42:57 PM Mark this question ec Question 4 of 10 Interferon alpha immunotherapy is used as treatment of which for the following condition? a) Acute lymphoblastic leukemia ) Hairy cell Ieukesnie ©) Acute myeloid leukemia ) Buckit's lymphoma €) Myelodysplastic syncrome Question Explanation: interferon-alpha is an immune system hormone which is very helpfilto a relatively small number of patients, and somewhat helpfil to most patients. Most commonly, the drug helps stabilize the disease or produce a slow, minor improvement. The typical dosing schedule injects 3 ruillion unite of Ixterferox-aipha (act pegylated versione) three times a week, although the original protocol began ‘with sic months of daly injections." 2125/2014 6:42:57 PM Mark this question ec Question 4 of 10 Interferon alpha immunotherapy is used as treatment of which for the following condition? a) Acute lymphoblastic leukemia Y © ») Hairy cell leukemia ©) Acute myeloid leukemia 4) Burkitt's lymphoma €) Myelodysplastic syncrome Question Explanation: interferon-alpha is an immune system hormone which is very helpfilto a relatively small number of patients, and somewhat helpfil to most patients. Most commonly, the drug helps stabilize the disease or produce a slow, minor improvement. The typical dosing schedule injects 3 ruillion unite of Ixterferox-aipha (act pegylated versione) three times a week, although the original protocol began ‘with sic months of daly injections." 2:25/2014 6:43:10 PM ‘Mark this question & => Question Id : 61718 Question 5 of 10 A.67 year old woman with aortic sclerosis is admitted with chest pain. Aa infarcts ruled ovt by cardiec enzymes, but the patient has recurrent symptoms when weaned offheparin. On 2nd day of hospitalization, she has sight arm pain, absent brachial pulse on the right, and a cold distal right arm, Hler hematocrit is 342%, and platelets are 30,000/mm3. Her partial thromboplastin time is 64 sec. ‘The mostlikely cause of this patient's absert brachial pulse is 2) Heparin-induced thrombocytopenia +) Embolization from aortic sclerosis ©) Paradoxical embolus 6) Hypetcoagalable state from immobilization ©) Vasospasm of the brachial artery Question Explanation: “Heparin indaced thrombocytopenia (HIT) is the result ofplatelet aggregation caused by heparin-induced antibodies Tis seen in 1% to 5% of patients on heparin. Therapy is discontinuation of the heparin and use of another anticoagulant, such as lepirudin, When the platelet count falls below 5(.000imm3 the keparin should be stopped. HIT can lead to limb-threatening thromboses, es in this patient, and constitutes a medical emergency. Arterial thrombosis is a manifestation of the HIT syndrome 2:25/2014 6:43:10 PM ‘Mark this question & => Question Id : 61718 Question 5 of 10 A.67 year old woman with aortic sclerosis is admitted with chest pain. Aa infarcts ruled ovt by cardiec enzymes, but the patient has recurrent symptoms when weaned off heparin. On 2nd day ofhosptalzation, che has right arm pain, absent brachial pulse on the right, and a cold distal right arm, Her hematoctit is 34%, and platelets are 3,000/nmn3. Her partial thomboplastin ime is 4 sec. ‘The most likely cause of this patient's absent brachial pulse is JM © 2) Heparin-induced thrombocytopenia ') Embolization from aortic sclerosis 6) Paradonical embolus 4) Bypercoaguleble state from immobilization ©) Vasospasin of the brachial artery Question Explanation: “Heparin indaced thrombocytopenia (HIT) is the result ofplatelet aggregation caused by heparin-induced antibodies Tis seen in 1% to 5% of patients on heparin. Therapy is discontinuation of the heparin and use of another anticoagulant, such as lepirudin, When the platelet count falls below 5(.000imm3 the keparin should be stopped. HIT can lead to limb-threatening thromboses, es in this patient, and constitutes a medical emergency. Arterial thrombosis is a manifestation of the HIT syndrome 2252014 6:43:23 PM “Mark this question <=> Question Id : 76216 Question 6 of 10 ‘The best way to screen men for prostate cancer is 2) Prostate specific antigen (PSA) assays every year beginning at puberty. b) Digtal rectal exam every year beginning at puberty ©) Anmal uitrasonography of the prostate in all men over 50. 6) PSA in all men over 50. ©) PSA and digital rectal exam in all men over 50 Question Explanation: ‘A combination of PSA and digital rectal exam (DRE) is the best method of identifying early prostate cancer in men over 50, the group at greatest ri for this disease, There is no need to begin screening at puberty. There ic no evidence that ultrasonography of the prostate has any benefit in screening 2252014 6:43:23 PM “Mark this question <=> Question Id : 76216 Question 6 of 10 ‘The best way to screen men for prostate cancer is 2) Prostate specific antigen SA) assays every year beginning at puberty b) Digtal rectal exam every year beginning at puberty ©) Anmal ultrasonography of the prostate in all men aver 50 6) PSA in all men over 50. Y © &) PSA and digital rectal exam in all men over 50. Question Explanation: ‘A combination of PSA and digital rectal exam (DRE) is the best method of identifying early prostate cancer in men over 50, the group at greatest ri for this disease, There is no need to begin screening at puberty. There ic no evidence that ultrasonography of the prostate has any benefit in screening 2:25/2014 6:13:39 PM “Mark this question => Question Id : 94554 Question 7 of 10 ‘All are frequently found in Chronic Myelogenous Leukemia (CML), except ) Elevated WBCs ) Elevated vitarnin B12 level ©) Elevated LDH 9) Increesed wie acid level 6) Translocation between chromosomes 9 and 14 Question Explanation: The diagnosis of CML is suspected besed on the results of a simple blood test The test may show an abnormally high white blood cell count. Ih blood samples examined under a microscope, less manure white blood cells, normally found only in bone marrow, are seen. Tests that analyze chromosomes (cytogenetics or molecular genetics) are needed to confitm the diagnosis. Chromosomal analysis of the leulcemia cells always shows a rearrangement of two particular chromosomes into whatis called the Philadelphia chromosome. The Philadelphia chromosome produces an abnormal enzyme (tyrosine kinase), which is responsible for the abnormal growth pattern of the white blood cells in CML, Philadetphia chromosome or Philadelphia translocation is a specific chromosomal abnormality that is associated with chronic myelogenous leukemia (CML) Tris due to a reciprocal translocation designated as t (9, 22) which means an exchange of genetic material between chromosome 9 and chromosome 22 n Report An Error 2:25/2014 6:13:39 PM “Mark this question => Question Id : 94554 Question 7 of 10 ‘All are frequently found in Chronic Myelogenous Leukemia (CML), except ) Elevated WBCs ) Elevated vitarnin B12 level ©) Elevated LDH 9) Increesed wie acid level Y © 6) Translocation between chromosomes 9 and 14 Question Explanation: The diagnosis of CML is suspected besed on the results of a simple blood test The test may show an abnormally high white blood cell count. Ih blood samples examined under a microscope, less manure white blood cells, normally found only in bone marrow, are seen. Tests that analyze chromosomes (cytogenetics or molecular genetics) are needed to confitm the diagnosis. Chromosomal analysis of the leulcemia cells always shows a rearrangement of two particular chromosomes into whatis called the Philadelphia chromosome. The Philadelphia chromosome produces an abnormal enzyme (tyrosine kinase), which is responsible for the abnormal growth pattern of the white blood cells in CML, Philadetphia chromosome or Philadelphia translocation is a specific chromosomal abnormality that is associated with chronic myelogenous leukemia (CML) Tris due to a reciprocal translocation designated as t (9, 22) which means an exchange of genetic material between chromosome 9 and chromosome 22 n Report An Error 2/25/2014 6:43:55 PM Mark this question & => Question Td : 107566 Question 8 of 10 ‘Which of the following is NOT a part of the staging of a sarcoma? a) Chest X-ray 6) Bone scan. c) MRI of the lesion. ) MRI of the brein. €) CT ofthe chest Answer (Beaten) Other User's Explanation Report An Error Question Explanation: MRI of the brain is not part of the intial staging of a sarcoma, MRI of the brain is reserved for cases with neurologic symptoms in association with tumors with knovin metastasis to the brain The other answer choices are part of routine staging, 2/25/2014 6:43:55 PM Mark this question & => Question Td : 107566 Question 8 of 10 ‘Which of the following is NOT a part of the staging of a sarcoma? a) Chest X-ray 6) Bone scan. c) MRI of the lesion. ¥ © 4) MRI of the brain. €) CT ofthe chest Answer (Beaten) Other User's Explanation Report An Error Question Explanation: MRI of the brain is not part of the intial staging of a sarcoma, MRI of the brain is reserved for cases with neurologic symptoms in association with tumors with knovin metastasis to the brain The other answer choices are part of routine staging, 2:25/2014 6:14:09 PM Mark this question & => Question Td : 107587 Question 9 of 10 Chronic clirical manifestation of sickle disease does NOT result from which of the following? a) "Fishmouth’ vertebrae are produced by bone infarction ) Systolic ejection murmurs are cansed by a hyperdynamic circulation. ) Papillary infarcts produce prolonged paitless hematuria. 4) Renal medullary infarcts cause isosthenuria, with a subsequent risk of overhydeation €) Hyperbiliubinemia causes formation of gallstones. Question Explanation: By adulthood, anatomic or functional damage to tissues is usually apparent. Such damage is frequently due to vasoocelusive episodes. "Fishmouth" or biconcave vertebrae, which result from bone infarction, are almost pathognomonic of sickle cell disease Papillary infarcts produce prolonged painless hemamaria. The acidic, hypertonic environment of the renal medulla produces infarction, which results in isosthenuria Consequentty, the inability to form concentrated urine increases the risk of significant dehydration. Systolic ejection murmurs are cansed by a hyperdynamic circulation. Gallstones result from an elevated nonconjugated byperbilinubinemnia, Report An Error 2:25/2014 6:14:09 PM Mark this question & => Question Td : 107587 Question 9 of 10 Chronic clirical manifestation of sickle disease does NOT result from which of the following? a) "Fishmouth’ vertebrae are produced by bone infarction ) Systolic ejection murmurs are cansed by a hyperdynamic circulation. ) Papillary infarcts produce prolonged paitless hematuria. Y © ) Renal medullary infarcts cause isosthenuria, with a subsequent risk of overhydration. €) Hyperbiliubinemia causes formation of gallstones. Question Explanation: By adulthood, anatomic or functional damage to tissues is usually apparent. Such damage is frequently due to vasoocelusive episodes. "Fishmouth" or biconcave vertebrae, which result from bone infarction, are almost pathognomonic of sickle cell disease Papillary infarcts produce prolonged painless hemamaria. The acidic, hypertonic environment of the renal medulla produces infarction, which results in isosthenuria Consequentty, the inability to form concentrated urine increases the risk of significant dehydration. Systolic ejection murmurs are cansed by a hyperdynamic circulation. Gallstones result from an elevated nonconjugated byperbilinubinemnia, Report An Error “Mark this question Question Td : 974 Question 1 of 30 A 39-year-old presents with atwo week history of recurrent hemoptysis which he has noted over the last 17 months. Having no chest pain, he is a smoker of 5 cigarettes daily. CXR reveals collapse of the leftlower lobe. What is the most likely diagnosis? 8) Bronchial carcinoma ) Bronchiectasis ©) Bronchial Carcinoid 6) Inhaled foreign body €) Pulnonary embolism Anewor [UERIRNSHEN) otnor Usov's Explanat Question Explanation: ‘The most ely diagnosis i of a carcineid tumer. Thece are clow growing tumors of the lung that occur ia younger patiente than bronchial carcinoms, the peak incidence in younger is when patients than bronchial carcinoma, the peak incidence is when patients are in their 40s. They account for between 1-5%of all hing tumors. They are not linked with smoking, The mncidence is equal between men and women. They usually present with recurrent hemoptysis or infections. Chest pain, breathlessness. wheeze and cough are less common. They usually ocour in the major bronchi, 25% can be seen bronchcscopioally. A carcinoid tumour in the leftlower lobe bronchus could distal collapse of the leit lower lobe. Report An Error Mark this question => Question Td : 974 Question 1 of 30 A 39-year-old presents with atwo week history of recurrent hemoptysis which he has noted over the last 17 months. Having no chest pain, he is a smoker of 5 cigarettes daily. CXR reveals collapse of the leftlower lobe. What is the most likely diagnosis? 8) Bronchial carcinoma ) Bronchiectasis Y © ©) Bronchial Carcinoid 6) Inhaled foreign body €) Pulnonary embolism Anewor [UERIRNSHEN) otnor Usov's Explanat Question Explanation: ‘The most ely diagnosis i of a carcineid tumer. Thece are clow growing tumors of the lung that occur ia younger patiente than bronchial carcinoms, the peak incidence in younger is when patients than bronchial carcinoma, the peak incidence is when patients are in their 40s. They account for between 1-5%of all hing tumors. They are not linked with smoking, The mncidence is equal between men and women. They usually present with recurrent hemoptysis or infections. Chest pain, breathlessness. wheeze and cough are less common. They usually ocour in the major bronchi, 25% can be seen bronchcscopioally. A carcinoid tumour in the leftlower lobe bronchus could distal collapse of the leit lower lobe. Report An Error Mark this question & => Question Td : 10403 Question 2 of 30 AT1-vear-old man oresents with weight loss and dyspnea and is diagnosed with small celllung cancer. Which of the following is an adverse prognostic feature? a) Cavitation on X ray +) Decreased alkaline phosphatase o) Finger clubbing ) Hyponatremia ©) Hypertrophic pulmonary osteoarthrepathy Answer | Biplanation) | Other User's Explanation Report An Error Question Explanation: ‘The following are adverse prognostic factors in small cell ling cancer * Serum soduum <132 mEq/L + Weight loss 10% * WHO performance status >2 © Alkaline phosphatese > 1.5 times upper limit of normal + Lactate deaydrogenase (LDH) > 1.5 times upper limit of normal ° Extensive disease (disease occurring outside one hemithorax and ipsilateral supraclavicular fossa nodes) Mark this question & => Question Td : 10403 Question 2 of 30 A T1-vear-old man presents with weight loss and dyspnea and is diagnosed with small cell hing cancer. Which of the following is an adverse prognostic feature? Y a) Cavitation on X ray b) Decreased allcaline phosphatase o) Finger clubbing ©) Hyponatremia ©) Hypertrophic pulmonary osteoarthrepathy Answer | Biplanation) | Other User's Explanation Report An Error Question Explanation: ‘The following are adverse prognostic factors in small cell ling cancer Serum sodium <132 mEq/L ‘Weight loss >10% WHO performance status >2 Alkaline phosphatase > 1.5 times upper limit of normal Lactate dehydrogenase (LDH) > 1.5 times upper limit of normal Entensive disease (disease occurring outside one hemithorax and ipsilateral supractavicular fossa nodes). ‘Mark this question —& => Question Td : 14093 Question 3 of 30 Anew chemotherapeutic agent is said to decrease the five year mortalty rate to 60% which was 80% without treatment. Which of the following represents the absolute risk reduction using this treatment? a) 15% 6) 20% ©) 30% d) 33% 6) 45% Avewor [UBRPINISER) otnor UeorsExplanation Report An Error Question Explanation: The absolute risk reduction is an important figure and should always be quoted instead of the relative risk reduction. Examples: Ifa drag reduces the incidence of heart attacks fiom 10% to 5% then + The control eveat rate (CER) is 10% + The experimental event rate (EER) is 5% + The relative risk reduction (RRB) is 50% + The absolute risk reduction (ARR) is 5%. + The mumber needed to treat (NNT) is 100% / 3% =20. ‘Mark this question —& => Question Td : 14093 Question 3 of 30 Anew chemotherapeutic agent is said to decrease the five year mortalty rate to 60% which was 80% without treatment. Which of the following represents the absolute risk reduction using this treatment? a) 15% VO by 20% c) 30% ) 33% 2) 45% Avewor [UBRPINISER) otnor UeorsExplanation Report An Error Question Explanation: The absolute risk reduction is an important figure and should always be quoted instead of the relative risk reduction. Examples: Ifa drag reduces the incidence of heart attacks fiom 10% to 5% then + The control eveat rate (CER) is 10% + The experimental event rate (EER) is 5% + The relative risk reduction (RRB) is 50% + The absolute risk reduction (ARR) is 5%. + The mumber needed to treat (NNT) is 100% / 3% =20. 2/26/2014 3:17:26 AM “Mark this question e& => Question Id : 23096 Question 4 of 30 A.65 year old man is found to have a nodular prostate on digital rectal examination. Subsequent biopsy confirms cancer of the prostate: which one of the following would have the most prognostic value? a) Pathological grads (Gleason score) b) Age ©} Urinary obstructive symptoms 4) Positive fanny history ¢) Adenosquamous histology 6 Avower [JEINSHER) otnerUsor® Explanation Report An Error Question Explanation: Prognosis for prostate cancer invelwes the following main factors: Stage and Gleason grade or score Stage: As with many other types of cancer, the outcome of prostate cancer depends on whether it has spread when itis diagnosed. Gleason grate or score: With prostate cancer; the prognosis also depends on how the cells look under the microscope, This is called the grade ofthe cancer Th prostate cancer, this is recorded as the Gleason score. A Gleason score of 2, 3 ot 4 indicates a low arade prostate cancer thats icely to grow very slowly; A Gleason score of 5, 6 or 7 is an intermediate grade that vill grow at a moderate rate. A Gleasoa score of 8 to 10 is ahigh grade cancer that is lixely to grow relatively fast. 2/26/2014 3:17:26 AM “Mark this question e& => Question Id : 23096 Question 4 of 30 A.65 year old man is found to have a nodular prostate on digital rectal examination Subsequent biopsy confirms cencer of the prostate: which one of the following would have the most prognostic value? Y © 2a) Pathological grade (Gleason score) b) Age c) Urinary obstructive symptoms 4) Positive fanny history c) Adenosquamous histology 6 Avower [JEINSHER) otnerUsor® Explanation Report An Error Question Explanation: Prognosis for prostate cancer invelwes the following main factors: Stage and Gleason grade or score Stage: As with many other types of cancer, the outcome of prostate cancer depends on whether it has spread when itis diagnosed. Gleason grate or score: With prostate cancer; the prognosis also depends on how the cells look under the microscope, This is called the grade ofthe cancer Th prostate cancer, this is recorded as the Gleason score. A Gleason score of 2, 3 ot 4 indicates a low arade prostate cancer thats icely to grow very slowly; A Gleason score of 5, 6 or 7 is an intermediate grade that vill grow at a moderate rate. A Gleasoa score of 8 to 10 is ahigh grade cancer that is lixely to grow relatively fast. “Mark this question & => Question 5 of 30 A 58 year old woman presents to her family physician with a 3 cm palpable, well circumserbed, non-tender breast mess. She first noticed i several months ago and believes it has increased in size since then She has 2 family history of breast cancer. No abnormalities are found on mammogram. Which appropriate step yon should take nest? a) Prophylactic mastectomy 'b) Repeat mammogram in 6 months 0) Excisional biopsy ) Repeat mammogram in 1 year 8) Ultrasound Question Explanation: Breast cancer most often involves glandular breast cells i the ducts or lobules. Most patients preseat with an asyznptometic lump discovered during exemination or screening marrmography. advanced cancer is suspected based on physical examination, biopsy should be done first. otherwise, the approachis as for breast luxps. A prebiopsy bilateral mammogram may help delineate other areas that chould be biopsied and provides a baseline for r future reference, However, mammogram recults chould not alter the decision to perform a biopsy. Biopsy can be needle or incisional biopsy or, ifthe tumor is small, excisional biopsy. “Mark this question & => Question 5 of 30 A.58 year old woman presents t» her family physician with a 3 cm palpable, well circumscribed, non-tender breast mass. She first noticed é several months ago and believes i has increased in size since then She has a family history of breast cancer. No abnormalities are found on mammogram. Which appropriate step you should take rest? a) Prophylactic mastectomy 's) Repeat mammogram in 6 months Y © c) Excisional biopsy 4) Repeat mammogrzrn in 1 year 2) Ultrasound Question Explanation: Breast cancer most often involves glandular breast cells i the ducts or lobules. Most patients preseat with an asyznptometic lump discovered during exemination or screening marrmography. advanced cancer is suspected based on physical examination, biopsy should be done first. otherwise, the approachis as for breast luxps. A prebiopsy bilateral mammogram may help delineate other areas that chould be biopsied and provides a baseline for r future reference, However, mammogram recults chould not alter the decision to perform a biopsy. Biopsy can be needle or incisional biopsy or, ifthe tumor is small, excisional biopsy. 2126/2014 3:17:53 AM ‘Mark this question & <=> Question 6 of 30 A.36 year old black woman retumed Lome from a vacation in Hawai, She comes to you with a swollen ie lower extremity. She has no previous history of similar problems Homan's sign s positive, and ultrasonography reveals a noncompressile vein in the left popliteal fossa extending distally. True statement regarding situation is which of the following? a) Monotherapy with an initial 10-mg loading dose of warfarin (Coumadin) would be appropriate ') Bnoxapasin (Lovenox) should be administered at a dosage of Img/k subcutaneously twice a day c) The incidence of thrombocytopenia is the same with low molecular weight heparin 4) The dosage of warfarin should be adjusted to maintain the INR. at 2.5-3.5 ©) Anticoagulant therapy should be started ac soon ac poscile and maintained for 1 year to prevent deep vein thrombosis VD) recurence Question Explanation: The use of low molecular weight heparin allows patients with acute deep vein thrombosis (DVT) to be managed as outpatients. The dosage is 1 mg/kg subcutaneously twice daily. Patient chosen for outpatient care should have good cardiopulmonary reserve, nommnal renal finction, and no tisk for excessive bleeding. Oral anticoagulation with werfarin can be initiated on the first day of treatment after heparin loading is completed. ‘Monotherapy with warfarin is inappropriate The incidence of thrombocytopenia with low molecular weight heparin is lower than with conventional heparin. The INR should be maintained at 2 0-3.0 in this patient. The 2.5-3 5 range is used for patients with mechanical heart- valves. The therapeutic INR should be maintained for 3-6 month in a patient with a first DVT related to travel. jon Report An Error 2126/2014 3:17:53 AM ‘Mark this question & <=> Question 6 of 30 A.36 year old black woman retumed Lome from a vacation in Hawai, She comes to you with a swollen ie lower extremity. She has no previous history of similar problems Homan's sign s positive, and ultrasonography reveals a noncompressile vein in the left popliteal fossa extending distally. True statement regarding situation is which of the following? a) Monotherapy with an initial 10-mg loading dose of warfarin (Coumadin) would be appropriate Y © b) Enoxaparin (Lovenox) should be administered at a dosage of Imayk subcutanzously twice a day c) The incidence of thrombocytopenia is the same with low molecular weight heparin 4) The dosage of warfarin should be adjusted to maintain the INR. at 2.5-3.5 ©) Anticoagulant therapy should be started ac soon ac poscile and maintained for 1 year to prevent deep vein thrombosis VD) recurence Question Explanation: The use of low molecular weight heparin allows patients with acute deep vein thrombosis (DVT) to be managed as outpatients. The dosage is 1 mg/kg subcutaneously twice daily. Patient chosen for outpatient care should have good cardiopulmonary reserve, nommnal renal finction, and no tisk for excessive bleeding. Oral anticoagulation with werfarin can be initiated on the first day of treatment after heparin loading is completed. ‘Monotherapy with warfarin is inappropriate The incidence of thrombocytopenia with low molecular weight heparin is lower than with conventional heparin. The INR should be maintained at 2 0-3.0 in this patient. The 2.5-3 5 range is used for patients with mechanical heart- valves. The therapeutic INR should be maintained for 3-6 month in a patient with a first DVT related to travel. jon Report An Error 2/26/2014 3:1 :05 AN “Mark this question <=> Question 7 of 30 Question Td : 54259 ‘An elderly female is on Tamonifen therapy for advanced breast cancer with bone metastases. She develops increased thirst, increased ination, disorientation, nausea, vomiting, confision and agitation. The most lkely cause is a) Tamoifin side effect b) Hypercalcemia ©) Brain metastasis ) SIADA Answer | Explanation Other User's Explanation Report An Error Question Explanation: Hypercalcemia is the most common paraneoplastic syndrome associated with cancer. Symptoms and signs ofhypercelcemia may be noted when total serum Cais > 12 mg/dL (@ 3 mmol/L). These signs cen inchd anorexia, GI refhas, vomiting, letharay or seimres or generelized invtability, and hypertension. Other symptoras and signs include constipation, dehydration, feeding intolerance, end fahure to thrive. 2/26/2014 3:1 :05 AN “Mark this question <=> Question 7 of 30 Question Td : 54259 ‘An elderly female is on Tamonifen therapy for advanced breast cancer with bone metastases. She develops increased thirst, increased ination, disorientation, nausea, vomiting, confision and agitation. The most lkely cause is a) Tamoifin side effect Y © b) Hypercalcemia ©) Brain metastasis ) SIADA Answer | Explanation Other User's Explanation Report An Error Question Explanation: Hypercalcemia is the most common paraneoplastic syndrome associated with cancer. Symptoms and signs ofhypercelcemia may be noted when total serum Cais > 12 mg/dL (@ 3 mmol/L). These signs cen inchd anorexia, GI refhas, vomiting, letharay or seimres or generelized invtability, and hypertension. Other symptoras and signs include constipation, dehydration, feeding intolerance, end fahure to thrive. ‘Mark this question & => Question Td : 59601 Question 8 of 30 A.55 year old black male presents to you 2 days after evaluation in the emergency department for acute dificulty uinating, His acute symptoms have improved, He requests screening for prostate cancer. You are concerned thet his vist to the emergency department may affect the results of prostate cancer screening, and you decide to obtain more information. The postible occurrence 2 days eatlier would NOT preclude prostate specific antigen (PSA) screening is a) Performance of a rectal examination b) Performance of prostate massage ) Performance of Cystoscopy ) A diagnosis of urinary tract infection e) A diagnosis of urinary retention Question Explanatioy Elevated results on prostrate-specfic antigen (PSA) screening are associated with a variety of factors. A simple rectal examination, however, does not elevate the level. Benign prostate examination is a frequent cause of mild elevations in PSA. Interventions such as prostatic massege or cystcscopy will also elevate the level. Other conditions that can elevate PSA inchade urinary tract infections and usinary retention Report An Error ‘Mark this question & => Question Td : 59601 Question 8 of 30 A.55 year old black male presents to you 2 days after evalvation in the emergency department for acute dificulty urinating, His acute symptoms have improved, He requests screening for prostate cancer, You are concerned thet lis visi to the emergency department may affect the results of prostate cancer screening, and you decide to obtain more information. The possible occurrence 2 days earlier would NOT preclude prostate specific antigen PSA) screenng is ¥ © a) Performance of a rectal examination b) Performance of prostate massage ] Performance of Cystoscopy d) A diagnosis of urinary tract infection e) A diagnosis of urinary retention Question Explanatioy Elevated results on prostrate-specfic antigen (PSA) screening are associated with a variety of factors. A simple rectal examination, however, does not elevate the level. Benign prostate examination is a frequent cause of mild elevations in PSA. Interventions such as prostatic massege or cystcscopy will also elevate the level. Other conditions that can elevate PSA inchade urinary tract infections and usinary retention Report An Error Tae ths question @&c=> [Question ta 628] Question9 of 30 A 60-year-old chronic emoker has hyponatremia and hyperosmolar urine. Increased levels of anti-ciuretic hormone are detected “Which one of the following is most licely type of lang cancer in this patient? a) Bronchioloalveolar carcinoma b) Large cell carcinoma ©) Adencearcinoma 4) Smnall cell carcinoma ©) Carcinoid of Lung Anower [JERENEERI) otnor User's Exp Question Explanation: Tn cases of lung cancer in which the hormones produced are anti-diuretic hormone or ACTH, it can be presumed, until proven wrong, that the type of cancer is stall cell carcinoma, Small cell carcinomas, ot oat cell carcinomas, are notorious for producing ectopic hormones. Although large cell carcinomas can produce ectopir hormones, they typicaly produce HEM factor, which is a parathyroid-like hormone, Carcinoids of lung generally are unrelated to such effects jon Report An Error Tae ths question @&c=> [Question ta 628] Question9 of 30 A 60-year-old chronic smoker has hyponatremia and hyperosmolar unne. Increased levels of anti-ciuretic hormone are detected. ‘Which one of the following is most likely type of lang cancer in this patiers? a) Bronchioloatveolar carcinoma 'b) Large cell carcinoma ¢) Adenocarcinoma VM © 4) Small cell carcinoma ¢) Carcinoid of Lung Anower [JERENEERI) otnor User's Exp Question Explanation: Tn cases of lung cancer in which the hormones produced are anti-diuretic hormone or ACTH, it can be presumed, until proven wrong, that the type of cancer is stall cell carcinoma, Small cell carcinomas, ot oat cell carcinomas, are notorious for producing ectopic hormones. Although large cell carcinomas can produce ectopir hormones, they typicaly produce HEM factor, which is a parathyroid-like hormone, Carcinoids of lung generally are unrelated to such effects jon Report An Error Mark this question & => Question Td : 62580 Question 10 of 30 ‘The most characteristic lab fincing in a patient with hemolytic anemia is 2) Increased haptoglobin b) Retculocytosis MCV < 80 4) Abnormal Hb electrophoresis Question Explanation: Hemolysis is the premature destruction of erythrocytes, and i leads to hemolytic anemia when bone marrow activity cannot compensate for the erythrocyte loss. Clinical presentation depends on whether the onset of hemolysis is gradual or abrupt and on the severity of erythrocyte destruction. A patient with mild hemolysis may be asymptomatic. In more serious cases, the anercia can be life threatening, and paticnts can present with angina and cerdiopulmonary decompensation. Aa incrcated reticulocyte countis a criterion for hemolysis butis not specific for hemolysis. Mark this question & => Question Td : 62580 Question 10 of 30 “The most characteristic lab finding in a patient with hemolytic anemia is a) Increased haptoglobin Y © b) Retculocytosis MCV < 80 4) Abnommal Hb electrophoresis Question Explanation: Hemolysis is the premature destruction of erythrocytes, and i leads to hemolytic anemia when bone marrow activity cannot compensate for the erythrocyte loss. Clinical presentation depends on whether the onset of hemolysis is gradual or abrupt and on the severity of erythrocyte destruction. A patient with mild hemolysis may be asymptomatic. In more serious cases, the anercia can be life threatening, and paticnts can present with angina and cerdiopulmonary decompensation. Aa incrcated reticulocyte countis a criterion for hemolysis butis not specific for hemolysis. Mark this question & => Question Ti : 67336 Question 11 of 30 An 82 year old man is talking warfann because of a history of atrial fibrilaton, He is found to have an INR of 7.2 on rowine blood workup, He reports no bleeding. The best immediate management of this problem is a) Vitemnin K 1 1g orally ) Hold the warfarin for 3 days ©) Vitemnin K 10 ing intravenously 6) Fresh frozen plasma intravenously ©) Desmopressin (DDAVP) Anower [UEXPIRNSHEN) otnor User's explana Question Explanation: Ifthe INR is between 5.0-9.0 and bleeding is absent, you should cease warfarin therapy, consider reasons for elevated INR end patient-specific factors IFbleeding risk is high, give vitamin K (1.0-2 0 mg orally ot 0,5-1.0 mg intravenously), Measure INR within 2Ahours; resume werferin at a reduced dose once INR is in therapeutic range. However. ifthere is any clinically significant bleeding where warfarin-induced coaglopathy is considered a contributing factor, you should cease warfarin therapy, give 5.0-10.( mg vitamin K intravenously, as well as fresh frozen plasma (150-300 mL), assess, patient continously until INR < 5.0, and bleeding stops. Report An Error Mark this question & => Question Ti : 67336 Question 11 of 30 An 82 year old man is talking warfarin because of a history of attial brilaton, He is found to have an INR of 7.2 on routine blood workup, He reports no bleeding. The best immediate management of this problem is © @ Vitamin K 1 mg orally ) Hold the warfarin for 3 days ©) Vitamin K 10 mg intravenously 6) Fresh frozen plasma intravenously ©) Desmopressin DDAVP) Anower [UEXPIRNSHEN) otnor User's explana Question Explanation: Ifthe INR is between 5.0-9.0 and bleeding is absent, you should cease warfarin therapy, consider reasons for elevated INR end patient-specific factors IFbleeding risk is high, give vitamin K (1.0-2 0 mg orally ot 0,5-1.0 mg intravenously), Measure INR within 2Ahours; resume werferin at a reduced dose once INR is in therapeutic range. However. ifthere is any clinically significant bleeding where warfarin-induced coaglopathy is considered a contributing factor, you should cease warfarin therapy, give 5.0-10.( mg vitamin K intravenously, as well as fresh frozen plasma (150-300 mL), assess, patient continously until INR < 5.0, and bleeding stops. Report An Error ‘Mark this question Question Td : 87814 Question 12 of 30 An 11 year old child develops a testicular mass and undergoes orchiectomy. The mass shows a variety of appearances and colors on the cut section Histologicaly, many diferent tissues are seen, including cartilage, thyroid, and neural tissue. A stall focus of clear cut squamous cell carcinoma is seen. Which of the following is the most appropriate classification for this tumor? 2) Denmoid cyst b) Embryonal cercinoma ©) Immature teretoma 4) Solid mature teratoma 6) Teratoma with malignant transformation Answer | Explanation Other User's Explanation Report An Error Question Explanation: ‘This is teratoma with malignant transformation, as evidenced by the focus of squamous cell carcinoma. The possibility of malignant transformation is why even obvious mature teratomas with yew well differentiated tissues should be completely excised Malignant transformation ic more commen in teratcmac in adults than in children or babies. Dermcid cyst is a cystic form of mature teratoma, usually found in the ovaries. Embryonal carcinoma is a type of adenocarcinoma rather than squamous cell carcinoma. Immature teratoma, while clincally malignant, shows embryonal tissues and often displays no clear-cut cytological evidence of malignancy. Solid inamure teratoma without the added descriptor with malignant transformation is by definition a benign tumor in females and prepubertal males (all teratomac are considered to have malignant potential in postpubertal males), Careful extensive carmpling is required to exclude minute foci of cancerous transformation, ‘Mark this question Question Td : 87814 Question 12 of 30 An 11 year old child develops a testicular mass and undergoes orchiectomy. The mass shows a variety of appearances and colors on the cut section Histologicaly, many diferent tissues are seen, including cartilage, thyroid, and neural tissue. A stall focus of clear cut squamous cell carcinoma is seen. Which of the following is the most appropriate classification for this tumor? 2) Denmoid cyst b) Embryonal cercinoma ©) Immature teretoma 4) Solid mature teratoma ¥ © 6) Teratoma with malignant transformation Answer | Explanation Other User's Explanation Report An Error Question Explanation: ‘This is teratoma with malignant transformation, as evidenced by the focus of squamous cell carcinoma. The possibility of malignant transformation is why even obvious mature teratomas with yew well differentiated tissues should be completely excised Malignant transformation ic more commen in teratcmac in adults than in children or babies. Dermcid cyst is a cystic form of mature teratoma, usually found in the ovaries. Embryonal carcinoma is a type of adenocarcinoma rather than squamous cell carcinoma. Immature teratoma, while clincally malignant, shows embryonal tissues and often displays no clear-cut cytological evidence of malignancy. Solid inamure teratoma without the added descriptor with malignant transformation is by definition a benign tumor in females and prepubertal males (all teratomac are considered to have malignant potential in postpubertal males), Careful extensive carmpling is required to exclude minute foci of cancerous transformation, 226.2014 3:19:28 AM “Mark this question €e>_ Question 13 of 30 ‘Which cancer does NOT has an increased risk with high fat diet? a) Breast. b) Colorectal c) Prostate. 4) Renal ) Lang Anewor [RESHRENRAY) other Users Explanation Report An Eur Question Explanation: The incidence of renal cell carcinomais not increased with intalces of high fat diets. However, high fat diet increases the risk of breast, colorectal, prostate, and lung carcinoma 226.2014 3:19:28 AM “Mark this question €e>_ Question 13 of 30 ‘Which cancer does NOT has an increased risk with high fat diet? a) Breast. b) Colorectal ©) Prostate. YW © 4) Renal ) Lang Anewor [RESHRENRAY) other Users Explanation Report An Eur Question Explanation: The incidence of renal cell carcinomais not increased with intalces of high fat diets. However, high fat diet increases the risk of breast, colorectal, prostate, and lung carcinoma 2:26:2014 3:19:45 AM ‘Mark this question & => Question Td : 95639 Question 14 of 30 ‘What is the treatment of choice for a two day post cholecystectomy patient with pulmonary erabolism? a) Heparin intravenously. 'b) Heparin subcutaneously. c) Inferior vena cava fitter placement. 4) Coumadin c) Thrombolytic therapy. Question Explanation: IVC filter placementis incicated in patients who are not candidetes for enticoagulation therapy with heperin'warfarin (recent post surgical, active bleeding, neurosurgical patients, severe diastolic hypertension), Thrombolytics would also be contraindicated in this patient for the same reason. Also, thrombolytics have not been definitively shown to be efficacious for treatment of PE. For hemodynamically unstable patients, pulmonary embolectomy would be indicated An IVC filter will nt treat the pulnonary emblus, but will hopefilly prevent further embolito the lungs 2:26:2014 3:19:45 AM ‘Mark this question & => Question Td : 95639 Question 14 of 30 ‘What isthe treatment of choice for a two day post cholecystectomy patient with pulmonary embolism? a) Heparin intravenously. 'b) Heparin subcutaneously, Y © 6) Inferior vena cava filter placement 4) Coumedin ©) Tarembolylic therapy. Question Explanation: IVC filter placementis incicated in patients who are not candidetes for enticoagulation therapy with heperin'warfarin (recent post surgical, active bleeding, neurosurgical patients, severe diastolic hypertension), Thrombolytics would also be contraindicated in this patient for the same reason. Also, thrombolytics have not been definitively shown to be efficacious for treatment of PE. For hemodynamically unstable patients, pulmonary embolectomy would be indicated An IVC filter will nt treat the pulnonary emblus, but will hopefilly prevent further embolito the lungs 2:26/2014 3:20:00 AM ‘Mark this question & => Question Td : 100077 Question 15 of 30 ‘What is the principle therapeutic component of the management of acute lymphocytic leulcernia? 2) Supportive care. ) Antibiotics. ©) Chemotherapy. 4) Radiotherapy. ©) Surgery. Question Explanation: Chemotherapy to eracicete the leukemia cells and to suppress the leulcemic cell ine is the main treatment for leukemia Leukemia should be treated agaressively Antibiotics are usually not indicated unless there is suspicion of infection. Radiotherapy is only used as an adjavant and is not very effective Surgery ic not part of the routine management, though bone marrow transplantation is gaining popularity in cases where patients achieve a complete rerission following chemotherapy. 2:26/2014 3:20:00 AM ‘Mark this question & => Question Td : 100077 Question 15 of 30 ‘What is the principle therapeutic component ofthe management of acute lymphocytic leukemia? 2) Supportive cae. ) Antibiotics. Y © © Chemotherapy. 4) Radiotherapy 6) Surgery. Question Explanation: Chemotherapy to eracicete the leukemia cells and to suppress the leulcemic cell ine is the main treatment for leukemia Leukemia should be treated agaressively Antibiotics are usually not indicated unless there is suspicion of infection. Radiotherapy is only used as an adjavant and is not very effective Surgery ic not part of the routine management, though bone marrow transplantation is gaining popularity in cases where patients achieve a complete rerission following chemotherapy. ‘Mark this question & => Question Td : 113638 Question 16 of 30 A52 year old female develops acute hemolysis after receiving a four unt blood transfusion during pen heart surgery. Antibodies to the major blood groups have been ruled out ac etiologic agents. Which is most Hcely to be responsible fer reaction in thie patient? a) Cytomegalovirus +) Hepatitis B ©) Ant Lewis antbodies 4) Antinitochendeial antibodies ©) Ant-Ro antbodies Question Explanation: Antibodies to the Lewis blood group antigen, a minor blood group antigen, can be responsible for transfusion reactions in ABO compatible transfusions. Cytomegalovirus and hepatitis B do not cause acute hemolysis after transfusion, Antimitochondtial antibodies are ascociated with primary biliary cirrhosis and not with transfusions, Anti-Ro antibodies are associated with systemic lupus erythematosus, not with post transfusion hemolysis, ‘Mark this question & => Question Td : 113638 Question 16 of 30 ‘A.52 year old female develops acute hemolysis after receiving a four unt blood transfusion during open heart surgery. Antibodies to the major blood groups have been ruled out as etiologic agents. Which is most likely to be responsible for reaction in this patient? a) Cytomegalovirus b) Hepatitis B © oc) Ant Lewis antbodies ) Antmitochondrial antibodies €) Ant-Ro antibodies Question Explanation: Antibodies to the Lewis blood group antigen, a minor blood group antigen, can be responsible for transfusion reactions in ABO compatible transfusions. Cytomegalovirus and hepatitis B do not cause acute hemolysis after transfusion, Antimitochondtial antibodies are ascociated with primary biliary cirrhosis and not with transfusions, Anti-Ro antibodies are associated with systemic lupus erythematosus, not with post transfusion hemolysis, 2:26/2014 3:20:22 AM Mark this question & => Question Td : 118847 Question 17 of 30 4.52 year old man presents with fatigue, weight loss, and abdominal pain Exam showed splenomegaly, mild hepatemegely, and pallor Lab data showed neutropenia, anemia, and thrombocytopsna, The peripheral smear and bone marrow revealed 2 Igmphocytosis, composed of cells having cytoplasmic projections. ‘The cells exhibit a strong acid phosphetase reaction. The most likely diagnosis is @) Chronic lymphocytic leukemia ) Hairy cell leukemia ©) Chronic myelogenous leulcemia 8) Acute lymphocytic leukemia ©) Infectious monomucleesis Question Explanation: Chronic myelogenous leukemia is a myeloproliferative disease associated with splenomegaly and an elevated white count Leukocyte alkaline phosphates would be positive. Acute lymphocytic leukemia is common in the extremes of lif children and the elderly. ‘Massive splenomegaly would be rare. Infectious mononucleosis can be associated with splenomegaly, but the neutropenia and thrombocytopenia would he rare, as well as the cytoplasmic projections on the abnormal lymphocytes Tafectious mononucleosis is associated with Epstein Barr virus Report An Error 2:26/2014 3:20:22 AM Mark this question & => Question Td : 118847 Question 17 of 30 A 52 year old man presents with Fatigue, weight loss, end abdominal pain. Fam showed splenomegaly, mild hepatcmegely, and pallor. Lab data showed neutropenia, anemia, and thrombocytopenia. The peripheral sinear and bone marrow revealed @ lymphocytosis, composed of cells having cytoplasmic projections. The cells exhibit a strong acid phosphatase reaction. ‘The most likely diagnosis is 2) Chronic lymphocytic leukemia V © b) Hairy cell leukemia ©) Chronic myelogenous leukemia 6) Acute lymphocytic leukemia ©) Infectious monomucleesis Question Explanation: Chronic myelogenous leukemia is a myeloproliferative disease associated with splenomegaly and an elevated white count Leukocyte alkaline phosphates would be positive. Acute lymphocytic leukemia is common in the extremes of lif children and the elderly. ‘Massive splenomegaly would be rare. Infectious mononucleosis can be associated with splenomegaly, but the neutropenia and thrombocytopenia would he rare, as well as the cytoplasmic projections on the abnormal lymphocytes Tafectious mononucleosis is associated with Epstein Barr virus Report An Error 2:26/2014 3:20:35 AM ‘Mark this question & => Question Td : 119684 Question 18 of 30 ‘Which ofthe fellowing is NOT usefil in the management of von Willebrand's disease? a) Cryoprecipitate ) Notmel plasma c) Desmopressin. 4) Aspirin €) Hormonal suppression. Question Explanation: Aspirin products should be avoided in patients with von Willebrand's disease, which is a result of ebnormnal platelet adhesion and low Factor 8 activity. Aspirin inhibits platelet aggregation and would compound this disorder. Normal plasma and crvoprecipitate have high levels of Factor 8, and transfssion of these blood products are the treatment of choice for severe forms of this condition Desmopressin is useful in mild forms, and honmonal suppression is useful in pregnancy. 2:26/2014 3:20:35 AM ‘Mark this question & => Question Td : 119684 Question 18 of 30 ‘Which ofthe fellowing is NOT usefil in the management of von Willebrand's disease? a) Cryoprecipitate ) Notmel plasma c} Desmopressin Y © 4) Aspirin. ©) Hormonal suppression Question Explanation: Aspirin products should be avoided in patients with von Willebrand's disease, which is a result of ebnormnal platelet adhesion and low Factor 8 activity. Aspirin inhibits platelet aggregation and would compound this disorder. Normal plasma and crvoprecipitate have high levels of Factor 8, and transfssion of these blood products are the treatment of choice for severe forms of this condition Desmopressin is useful in mild forms, and honmonal suppression is useful in pregnancy. Mark this question & => Question Td : 140060 Question 19 of 30 A.20 year old college student presents because of'a progressive growth on her jaw that has been getting larger over the past $ ‘weeks. She is an exchange student from Aftica, Biopsy of a frm 5 cm tumar on the left lateral aspect of her mandible shows neoplastic B cells with frequent mitotic figures and a 'sterry-siey' appearance. The microorgasism causing this disease is also associcted with a) Bladder carcinorra ) Cervical carcinoma d 4) #) Nasopharyngeal carcinoma Answer | Booanation) Other User's Explanation Report An Error Question Explanation: This patient has the Affican form of Burkitt lymphoma, which is a B-cell lymphoma that typically affects the jaw. Tt is most common in children, buthas been seen in acuits up to 40 years of age The Epstein Barr virus (EBV) is associated with up to 90% of cases of the Afticen form of Burkitt lymphoma, EBV is alse associated with nasopharyngeal carcinoma, which typically presents late in the disease course with obstructive symptoms, epistanis, and cranial neuropathies. Bladder carcinoma is associated with Schistosoma haematobium, cigarette smoking, and polycyclic aromatic hydrocarbons. It typically presents with hemataria and urinary frequency. Cervical carcinoma is associated with the human papillomavins Tt can be diagnosed at an early stage with a Pap smear Invasive carcinoma may present with postcoital spotting and vagnal discharge Hepatocelliar carcinomais associated with the hepatitis E and Cvinses Most mmors occur in patiemts with chronic infections and cirrhosis. The presentation is usually abdomninal pain and a mass in the setting of citthosis. Other causes of hepatocellular carcinoma include alcoholic iver disease and aflatoxin B1 Raposi sarcoma is associated with the human immunodeficiency virus and typically presents with mufiple vascular lesions on the skin The lesions are usually raised, redilista, purple macules in sun exposed areas. A proliferation of spindle and endothelial cells with extravasation of red blood cells is seen histologically. Hepatocellular carcinoma Kaposi sarcoma Mark this question & => Question Td : 140060 Question 19 of 30 A.20 year old college student presents because of'a progressive growth on her jaw that has been getting larger over the past $ ‘weeks. She is an exchange student from Aftica, Biopsy of a frm 5 cm tumar on the left lateral aspect of her mandible shows neoplastic B cells with frequent mitotic figures and a 'sterry-siey' appearance. The microorgasism causing this disease is also associcted with a) Bladder carcinorra ) Cervical carcinoma d 4) ¥ © ¢) Nasopharyngeal carcinoma Answer | Booanation) Other User's Explanation Report An Error Question Explanation: This patient has the Affican form of Burkitt lymphoma, which is a B-cell lymphoma that typically affects the jaw. Tt is most common in children, buthas been seen in acuits up to 40 years of age The Epstein Barr virus (EBV) is associated with up to 90% of cases of the Afticen form of Burkitt lymphoma, EBV is alse associated with nasopharyngeal carcinoma, which typically presents late in the disease course with obstructive symptoms, epistanis, and cranial neuropathies. Bladder carcinoma is associated with Schistosoma haematobium, cigarette smoking, and polycyclic aromatic hydrocarbons. It typically presents with hemataria and urinary frequency. Cervical carcinoma is associated with the human papillomavins Tt can be diagnosed at an early stage with a Pap smear Invasive carcinoma may present with postcoital spotting and vagnal discharge Hepatocelliar carcinomais associated with the hepatitis E and Cvinses Most mmors occur in patiemts with chronic infections and cirrhosis. The presentation is usually abdomninal pain and a mass in the setting of citthosis. Other causes of hepatocellular carcinoma include alcoholic iver disease and aflatoxin B1 Raposi sarcoma is associated with the human immunodeficiency virus and typically presents with mufiple vascular lesions on the skin The lesions are usually raised, redilista, purple macules in sun exposed areas. A proliferation of spindle and endothelial cells with extravasation of red blood cells is seen histologically. Hepatocellular carcinoma Kaposi sarcoma 2126/2014 3:21:01 AM “Mark this question ec=> Question Id : 166957 Question 20 of 30 A Syear old child has chroric fatigue, malaise, and an 8 pound weight loss. Examination shows a pale child with meny small bruises on her forearms and shine, Investigations show anemia, thrombocytopenia, and anormal WBC coust. Blast forms are seen in the peripheral smear. Bone marrow aspirate and biopsy demonstrate partial replacement of the marrow by smal blasts with uniform, round nuclei, and scant cytoplasm, Inmunohistochemical studies show marking of these blast cell with anibodies directed against B cell antigens. Her disease should be classfied as which type of acute leukemia according to the French-American-British classification? aL b) 12 OM a M2 oy MB Anower [JBINEHER) otter UscrsExplanation Report An Error Question Explanation: ‘The French-American-Britich classification of acute leulcemias is in widespread use, and you should become faraliar with its structure. This patient has an acute lymphoblastic leukemia (indicated by B cell markers) rater than an acute myelogenous leukemia, so the appropriate letter designation is L rather than IM. The lymphoblasts have uniform nuclei and scant cytoplasm typical of the L1 subtype. Acute lymphoblastic leukemia is more common in chidhood than adulthood. The presentation ilusirated is typical. At the time of diagnosis anemia and thrombocytopenia cre common, The white cell count may be low, normal, or high and its abscluts value should not be used to either include or exclude the diagnosis. Type L2is characterized by lymphoblasts with more variability in appearance, often irregularity of nuclei, and more cytoplasm than type L1. Type M1 is undifferentiated myeloblastic leukemia and the leukemic cells do not show cytoplasmic granulation. Type Mis differentiated myeloblastic leukemia and hes at least a few cells showing sperse granulation. Type M3 is promyelocytic loukerria and shows cells resembling promyslocyies with often abundant granulation. 2126/2014 3:21:01 AM “Mark this question ec=> Question Id : 166957 Question 20 of 30 A S year old child has chronic fatgue, malaise, and an 8 pound weightloss. Examination shows a pale child with many small bmaises on her forearms and shine. Investigations show anemia, thrombocytopenia, and anctmal WBC count. Blast forms are seenin the peripheral smear. Bone marrow acpirate and biopsy demonstrate partial replacement of the marrow by smal blasts with uniform, round nuclei, and scant cytoplasm, Inmunohistochemical studies show marking of these blast cell with anibodies directed against B cell antigens. Her disease should be classed as which type of acute leukemia according to the French-American-British classification? Y@al 12 M1 aM ) M3 Anower [JBINEHER) otter UscrsExplanation Report An Error Question Explanation: ‘The French-American-Britich classification of acute leulcemias is in widespread use, and you should become faraliar with its structure. This patient has an acute lymphoblastic leukemia (indicated by B cell markers) rater than an acute myelogenous leukemia, so the appropriate letter designation is L rather than IM. The lymphoblasts have uniform nuclei and scant cytoplasm typical of the L1 subtype. Acute lymphoblastic leukemia is more common in chidhood than adulthood. The presentation ilusirated is typical. At the time of diagnosis anemia and thrombocytopenia cre common, The white cell count may be low, normal, or high and its abscluts value should not be used to either include or exclude the diagnosis. Type L2is characterized by lymphoblasts with more variability in appearance, often irregularity of nuclei, and more cytoplasm than type L1. Type M1 is undifferentiated myeloblastic leukemia and the leukemic cells do not show cytoplasmic granulation. Type Mis differentiated myeloblastic leukemia and hes at least a few cells showing sperse granulation. Type M3 is promyelocytic loukerria and shows cells resembling promyslocyies with often abundant granulation. 2:26/2014 3:21:16 AM ‘Mark this question & => Question Td : 202149 Question 21 of 30 A previously healthy 31 year old man has a two months history of weight loss, tiredness and nausea, Full blood count is normal Serum sodutn is low, potassium is increased. Urea is 3.0 mmolL, creatinine is 78.moLL, total T4 55 mmol/L (50-150) and TSHis 8 mnU/L. The next useful diagnostic testis, a) Anti-thyroid peroxidase antibody titre ) Insulin tolerance test c) Free thyroxine concentration 4) Short eynacthen test e) TRH test Answer | Brrtanation | Other User's Explanation Report An Error Question Explanation: This patient presents with weight loss, tiredness and nausea. He has hyponatraemia, hyperkelaemia and whet appears to be a mild primary hypothyroidism The diagnosis is Heely to be Addison's (primary hypoadrenalism) disease and the most appropriate test would be a short synacthen test, Tue link berween Addison's and primary hypothyroidism is that they a both conditions in the complex of autoimmune polyendocrine syndrome. Other possible associations of this chuster would be Type 1 Diabetes, vitligo, pemicious anaemia and chronic active hepatitis. Aa insulin tolerance test is contra-indicated in patent's in whom cortisol is less than 100nmoVL. A TRB testis rarely performed these days and really is an irrelevance, 2:26/2014 3:21:16 AM ‘Mark this question & => Question Td : 202149 Question 21 of 30 A previously healthy 31 year old man has a two monthe history of weight loss, irednecs and nausea, Full blood count is normal Serum sodium is low, potassium is increased, Urea is 3.0 mmolL, creatinine is T8snolL, total T4 55 mol/L (50-150) and TSHis 8 mU/L ‘The next usefl diagnostic testis a) Anti-thyroid peroxidase antibody titre ) Insulin tolerance test c) Free thyrozine concentration Y © 4) Short synacthen test ¢) TRH test Answer | Brrtanation | Other User's Explanation Report An Error Question Explanation: This patient presents with weight loss, tiredness and nausea. He has hyponatraemia, hyperkelaemia and whet appears to be a mild primary hypothyroidism The diagnosis is Heely to be Addison's (primary hypoadrenalism) disease and the most appropriate test would be a short synacthen test, Tue link berween Addison's and primary hypothyroidism is that they a both conditions in the complex of autoimmune polyendocrine syndrome. Other possible associations of this chuster would be Type 1 Diabetes, vitligo, pemicious anaemia and chronic active hepatitis. Aa insulin tolerance test is contra-indicated in patent's in whom cortisol is less than 100nmoVL. A TRB testis rarely performed these days and really is an irrelevance, 2:26/2014 3:21:30 AM Mark this question & => Question Td : 202179 Question 22 of 30 A boy aged 19 years with alucose-6-phesphate dehydrogenase deficiency wishes travel to Affica, He should be advised to awoid which one of the following? a) Primaquine ) Loperamide ©) Mefloquine 4) Ibuprofen 6) Yellow fever vaccine Question Explanation: GEPD deficiency is inherited in an X-Inked fashion and predisposes RCs to haemolysis, Drugs recognised to predispose to acute haemolysis in GOPD deficiency include antimalarials such as-Primaquine, suiphonamides, Nitrofurantoin and Nalidixic acid. 2:26/2014 3:21:30 AM Mark this question & => Question Td : 202179 Question 22 of 30 A boy aged 19 years with alucose-6-phesphate dehydrogenase deficiency wishes travel to Affica, He should be advised to awoid which one of the following? Y © 2) Primaquine ) Loperamide ©) Mefloquine 4) Ibuprofen 6) Yellow fever vaccine Question Explanation: GEPD deficiency is inherited in an X-Inked fashion and predisposes RCs to haemolysis, Drugs recognised to predispose to acute haemolysis in GOPD deficiency include antimalarials such as-Primaquine, suiphonamides, Nitrofurantoin and Nalidixic acid. 2126/2014 3:21.43 AM “Mark this question => Question Id : 202209 Question 23 of 30 Armale aged 60 years has bruising and tiredness. Exam reveals splenomegaly and labs show hemoglobin of 11 g/dL, WEC count of 100 x109/L and platelet count of 900 x109/L. Blood film reveals a neutrophilia, basophila, numerous myelocytes and 4% sryeloblasts. What is licely to be present in hinn? 2) BCR-ABL gene fission only 6) Deletion chromosome 13 6) Deletion 11q13 6) Nosmal chromosomal analysis 6) Translocation 9;22 Question Explanati ‘The Philadelphia chromosome (translocation 9, 22) is present in approximately 90% of subjects with CML. The molecular consequences of this translocation is the generation of the fision ber-abl gene which creates an abnormal protein stimulating white cell growth. Only 5% of cases have the ber-ebl fusion gone only without the typical Philadelphia chromosome, Deletion of Ch13 is associated with a poorer prognosis in Multiple Myeloma, 2126/2014 3:21.43 AM “Mark this question => Question Id : 202209 Question 23 of 30 Armale aged 60 years has bruising and tiredness. Exam reveals splenomegaly and labs show hemoglobin of 11 g/dL, WEC count of 100 x109/L and platelet count of 900 x109/L. Blood film reveals a neutrophilia, basophila, numerous myelocytes and 4% sryeloblasts. What is licely to be present in hinn? 2) BCR-ABL gene fission only 6) Deletion chromosome 13 6) Deletion 11q13 6) Nosmal chromosomal analysis SY © 2) Translocation 9;22 Question Explanati ‘The Philadelphia chromosome (translocation 9, 22) is present in approximately 90% of subjects with CML. The molecular consequences of this translocation is the generation of the fision ber-abl gene which creates an abnormal protein stimulating white cell growth. Only 5% of cases have the ber-ebl fusion gone only without the typical Philadelphia chromosome, Deletion of Ch13 is associated with a poorer prognosis in Multiple Myeloma, 2/26/2014 3:21:58 AM ‘Mark this question & => Question Id : 202310 Question 24 of 30 4.27 year old pregnant lady is being treated for a deep vein thrombosis with unfractionated heparin. She hes Hb of 9.8 gidL, WBC count of 9.5 x10°L and platelet count of 35 210°/L. What would be the best course of action for her? a) Change to hirudin 'b) Change to low molecular weight heparia c) Change to warfarin 4) Danaparcid e) No change in treatment and observe Answer | Bolanation Other User's Explanation Report An Error Question Explanation: This patient appears to have Heperin Induced Thrombocytopaemia. When HITis suspected, heparin treatment should be discontinued and alternative anticoagulation should be started The heparinoid daneparoid appeers to he the drug of choice for acute treatment and prophylasis because of ts low placental permeability. Himndin should only be used when either cross-reactivity with heparin induced antibodies or cutaneous allergy against heparinoids are observed. 2/26/2014 3:21:58 AM ‘Mark this question & => Question Id : 202310 Question 24 of 30 4.27 year old pregnant lady is being treated for a deep vein thrombosis with unfractionated heparin. She hes Hb of 9.8 gidL, WBC count of 9.5 x10°L and platelet count of 35 210°/L. What would be the best course of action for her? a) Change to hirudin 'b) Change to low molecular weight heparia c) Change to warfarin Y © dh Danaparcid e) No change in treatment and observe Answer | Bolanation Other User's Explanation Report An Error Question Explanation: This patient appears to have Heperin Induced Thrombocytopaemia. When HITis suspected, heparin treatment should be discontinued and alternative anticoagulation should be started The heparinoid daneparoid appeers to he the drug of choice for acute treatment and prophylasis because of ts low placental permeability. Himndin should only be used when either cross-reactivity with heparin induced antibodies or cutaneous allergy against heparinoids are observed. 2/26/2014 3:22:13 AM ‘Mark this question & => Question Id : 202441 Question 25 of 30 TEa patient with chronic renal failure is treated with erythropoietin (EPO), what would be expected in this patient? a) Decreased pure red cell aplasia 'b) Decreased tisk of hypertension c) Decreased risk of thrombosis ) Increased well being €) Reduced appetite Question Explanation: Increased viscosity is seen in EPO therapy which may exacerbate hypertension and thers is also increased risk of thrombosis. Pure red cell aplasia is a rare unwanted effect due to stimulation of antibodies by aciinistered EPO which cross reacts with patients endogencus EPO. Improvement in haemoglobin level results in the increased well being and better appetite. 2/26/2014 3:22:13 AM ‘Mark this question & => Question Id : 202441 Question 25 of 30 Ea paticet with chronic renal felure is treated with erythropoietin (EPO), what would be expected in this patient? a) Decreased pure red cell aplasia +b) Decreaced tisk of hypertencion c} Decreased tisk of thrombosis VY © 4) Increased well being ©) Reduced appetite Question Explanation: Increased viscosity is seen in EPO therapy which may exacerbate hypertension and thers is also increased risk of thrombosis. Pure red cell aplasia is a rare unwanted effect due to stimulation of antibodies by aciinistered EPO which cross reacts with patients endogencus EPO. Improvement in haemoglobin level results in the increased well being and better appetite. 2i26:2014 3:22:27 AM ‘Mark this question & => Question Td : 202977 Question 26 of 30 ‘A.56 year old man has anorexia and weightloss for 12 months. He had two DV'Ts whilst his INR was 2 (<1.4), He remained on long term warfarin therapy with an INR above 2.6. On exam he is pigmented and has a postural drop in his BP of 15 mmHg. Shor: synacthen test reveals a baseline cortisol concentration at time 0 of 120 nmol/L which rises to 15SnmoV/L after 30 minates (Mormal response >550 nmol/L). The Lkely diagnosis is a) Addison's disease ) Anti-phospholipid syndrome ©) Autoimmune polyendocrine Syndrome (Schmidt's disease) ) Protein $ deficiency ©) Pituitary infarction Question Explanation: With a history of recurrent DVT and confirmed hypoadrenalism this patient is likely to have the antiphosoholipid syndrome “Antiphospholipid syndrome is a primary diagnosis or may co-exist with SLE. Anti-Cardiolipin antibodies or Lupus anticoagulant may be present, Ibis associated with arterial and venousthrombosis and has a predilection for the adrenal veins causing adrenal infarction with consequent hypoadrenalism, Addison's disease is an autoimmune phenomenon and is not associated with DVT. The pigmentation (due to incteased ACTH in hypoadrenalism) would exclude pituitary infarction the cause of the hypoadrenelism, Hypoadrenalism is not associated with protein $ deficiency. Autoimmune Polyendocrine syndrome is associated with hypothyroidism, ype 1 dabetes, addison’s disease 2i26:2014 3:22:27 AM ‘Mark this question & => Question Td : 202977 Question 26 of 30 ‘A.56 year old man has anorexia and weightloss for 12 months. He had two DV'Ts whilst his INR was 2 (<1.4), He remained on long term warfarin therapy with an INR above 2.6. On exam he is pigmented and has a postural drop in his BP of 15 mmHg. Shor: synacthen test reveals a baceline cortisol concentration at time 0 of 120 nmol/L which risec to 15SamoV/L after 30 minates (ormal response >550 nmol/L). The Lkely diagnosis is a) Addison's disease ¥ © >) Anti-phospholipid syndrome ©) Autoimmune polyendocrine Smdrome (Schmidt's disease) ) Protein $ deficiency ®) Pituitary infarction Question Explanation: With a history of recurrent DVT and confirmed hypoadrenalism this patient is likely to have the antiphosoholipid syndrome “Antiphospholipid syndrome is a primary diagnosis or may co-exist with SLE. Anti-Cardiolipin antibodies or Lupus anticoagulant may be present, Ibis associated with arterial and venousthrombosis and has a predilection for the adrenal veins causing adrenal infarction with consequent hypoadrenalism, Addison's disease is an autoimmune phenomenon and is not associated with DVT. The pigmentation (due to incteased ACTH in hypoadrenalism) would exclude pituitary infarction the cause of the hypoadrenelism, Hypoadrenalism is not associated with protein $ deficiency. Autoimmune Polyendocrine syndrome is associated with hypothyroidism, ype 1 dabetes, addison’s disease 2126/2014 3:22:38 AM Mark this question << => Question Ta : 203017 Question 27 of 30 ATT year old woman who is on warfarin for atrial fibrillation presents with melaena, Her blood pressure is 90/60 mmHg and the heat rate is 100/min, Her Hb is 9gidL (12-16), MCV is 87 l (83-95) and INK is 7.2 (=1.4). PR exam confirms melaena, The best option for correcting the coagulopathy is a) FFP 6) IV Vitamin K ©) Stop warfarin and give TV vitarnin 4) Stop warfarin and give Vitamin and Prothrombin complex: concentrate €) Stop warfarin Answer | Explanation Other User's Explanation Report An Error Question Explanatio ‘This patientis hypotersive and tachycardic with melaena suggesting major bleeding episode on warfarin Jn these circumstances, carrent ghidelines suggest stopping warfarin, giving IV vitamin K, and ether FFP, or Prothrombin complex concentrate, Local guidelines will be available, and if in doubt consult with the haematologist on call, FFP may not completely reverse the effects of warfarin, so it may now be preferable to consider Prothrombin complex concentrate (PCC) if available. The rate of fatal haemorrhage in patients receiving warfarin approaches 1%, Iris therefore essertial that knowledge regarding the reversal of warfarin coagulation is serviceable. 2126/2014 3:22:38 AM Mark this question << => Question Ta : 203017 Question 27 of 30 A.77 yea eld woman who is on warfarin for atval brillation precents with meleena, Her blood pressure is 90/60 mmllg and the heat rate is 100/min, Her Hb is 9gidL (12-16), MCV is 87 l (83-95) and INK is 7.2 (=1.4). PR exam confirms melaena, The best option for correcting the coagulopathy is 2) FEP 6) IV Vitamin K ©) Stop warfarin and give IV vitamin K. Y © 4) Stop warfarin and give Vitamin and Prothrombin complex concentrate 6) Stop warfarin Answer | Explanation Other User's Explanation Report An Error Question Explanatio ‘This patientis hypotersive and tachycardic with melaena suggesting major bleeding episode on warfarin Jn these circumstances, carrent ghidelines suggest stopping warfarin, giving IV vitamin K, and ether FFP, or Prothrombin complex concentrate, Local guidelines will be available, and if in doubt consult with the haematologist on call, FFP may not completely reverse the effects of warfarin, so it may now be preferable to consider Prothrombin complex concentrate (PCC) if available. The rate of fatal haemorrhage in patients receiving warfarin approaches 1%, Iris therefore essertial that knowledge regarding the reversal of warfarin coagulation is serviceable. 2126/2014 3:22:51 AM “Mark this queston => ‘Question Id : 203047 Question 28 of 30 An 81 year old woman kas a3 month history of progressive mumbaess and unsteadiness of her gait. On exam, there is a mild spastic paraparesis, with brisk knee reflexes, extensor planters, sensory loss in the legs with a sensory level at T10, impaired joint position sense in the toss, and loss of vibration sense below the tac crests. Her Hh is 12.2 g/dl. and MCV is 95 fL. The tcely diagnosis is 2) Anterior spinal artery occlusion ) Dorsal meningioma ©) Multiple sclerosis & Subacute combined degeneration of the cord «) Tabes dorsalis Anower (UBQRRESEN) ter veers Explanation Report An Error Question Explanation: ‘The presence of'a sensory loss at T10 indicates a thoracic myelopathy. Subacute combined degeneration of the cord is unlikely as the haemoglobin concentration and mean corpuscular volume (MCV) are normal, Antenor spinal artery occlusion is unkcely as the history is progressive and chronic. 2126/2014 3:22:51 AM “Mark this queston => ‘Question Id : 203047 Question 28 of 30 An B1 year old woman has a 3 month history of progressive numbness and unsteadiness of her gait. On exam, there is a mild spastic paraparesis, with brisk knee reflexes, extensor planters, sensory loss in the legs with a sensory level at T10, impaired joint position sense in the toes, and loss of vibration sense below the iiac crests, Her Hib is 12.2 gidL. and MCV is 95 fL., The likely diagnosis is a) Anterior spinal artery occlusion Y © b) Dorsal meningioma ¢) Multiple sclerosis 4) Subacute combined degeneration of the cord ¢) Tabes dorsalis, Anower (UBQRRESEN) ter veers Explanation Report An Error Question Explanation: ‘The presence of'a sensory loss at T10 indicates a thoracic myelopathy. Subacute combined degeneration of the cord is unlikely as the haemoglobin concentration and mean corpuscular volume (MCV) are normal, Antenor spinal artery occlusion is unkcely as the history is progressive and chronic. 2126/2014 3:23:07 AM ‘Mark this question & => Question Td : 203150 Question 29 of 30 An 83 year old women kas tiredness. On exam she is anaemic but has no palpable spleen. Her haemoglobin is 9.7 g/dL. She is started on oral iron therapy for one month and her Hb remained unchanged. Further report shows MICV of 102 EL with aomal serum ferritin, vitamin B12 and red cell folate, Blood film show marked arisepcikilorytosis. The most likely diagnosis is a) Aplastic anaemia ) Anaemia due to renal disease ©) Hypothyroidism @) Iron deficiency anzemia ¢) Sideroblastic anaemia Anower (UBNREEEH) omer veers Explanation Report An Eo Question Explanation Idiopathic sideroblastic anzemia is a member of the myelodysplastic syndromes. In this condition, the red blood cells are normal or macrocytic and there is anisocytosis and poililocytosis on the peripheral films. This isn‘tiron deficiency anaemia as the ferritin normal and a microcytic anaemia would be expected. A normochromic normocytic anaemia is expected in renal disease (epo deficiency) [ivcictiverentearn gay cause a shattelevation of MACY bub no amon oidiocyhocs ic evident 2126/2014 3:23:07 AM ‘Mark this question & => Question Td : 203150 Question 29 of 30 An 83 year old women kas tiredness. On exam she is anaemic but has no palpable spleen. Her haemoglobin is 9.7 g/dL. She is started on oral iron therapy for one month and her Hb remained unchanged. Further report shows MICV of 102 EL with aomal serum ferritin, vitamin B12 and red cell folate, Blood film show marked arisepcikilorytosis. The most likely diagnosis is a) Aplastic anaemia ) Anaemia due to renal disease ©) Hypothyroidism @) Iron deficiency anaemia ¥ © 2) Sideroblastic anaemia Anower (UBNREEEH) omer veers Explanation Report An Eo Question Explanation Idiopathic sideroblastic anzemia is a member of the myelodysplastic syndromes. In this condition, the red blood cells are normal or macrocytic and there is anisocytosis and poililocytosis on the peripheral films. This isn‘tiron deficiency anaemia as the ferritin normal and a microcytic anaemia would be expected. A normochromic normocytic anaemia is expected in renal disease (epo deficiency) [ivcictiverentearn gay cause a shattelevation of MACY bub no amon oidiocyhocs ic evident 2:26/2014 3:23:23 AM “Mark tis question Question 30 of 30 ‘True statement regarding the treatment of iron deficiency anaemia is which one of the following? 4) Iron ie absorbed in the dicta jejunum ) Absorption of iron is increased by ascorbic acid ¢) Sustained release iron is a usefil way of given lager doces 4) Ferrous sulphate 200 mg has less elemental iron than the same dose of ferrous gluconate ¢) Parenteral iron is indicated when the anzcmia responds slowly to oral iron Anower (EIRREREN) ner veers Explanation Report An Evo Question Explanation: A. Iron is absorbed in the upper small intestine B. Absorption of oral iron is improved by ascorbic acid. C. Sustained release preparations may improve tolerance of oral iron but do not aid absorption, D. Feerous suiphate has mare elemental iron by mass E, Parenteral iron acts no faster than oral iron. Itis indicated when oral iron cannot be telerated ois not absorbed. 2:26/2014 3:23:23 AM “Mark tis question Question 30 of 30 ‘True statement regarding the treatment of iron deficiency anaemia is which one of the following? 4) Iron ie absorbed in the dicta jejunum Y © b) Absorption of iron is increased by ascorbic acid ¢) Sustained release iron is a usefil way of given lager doces 4) Ferrous sulphate 200 mg has less elemental iron than the same dose of ferrous gluconate ¢) Parenteral iron is indicated when the anzcmia responds slowly to oral iron Anower (EIRREREN) ner veers Explanation Report An Evo Question Explanation: A. Iron is absorbed in the upper small intestine B. Absorption of oral iron is improved by ascorbic acid. C. Sustained release preparations may improve tolerance of oral iron but do not aid absorption, D. Feerous suiphate has mare elemental iron by mass E, Parenteral iron acts no faster than oral iron. Itis indicated when oral iron cannot be telerated ois not absorbed. 2/26:2014 3:24:25 AM “Mark this question => Question Id : 9697 Question 1 of 30 A 25-year-old man with a teratoma of the testis attended for review following chemotherapy. Which one of the following serum tumour markers ic of most value in monitoring the clinioal progression of bic disease? a) Carbohydrate antigen CA 15-3 b) Carbohydrate antigen CAA 19-9 ©) Alpha-fetoprotein 4) Carbohydrate antigen CA 125 ©) Carcinoembryonic antigen never [EESRINGHRA) other sors Explanation | Report An Evor Question Explanation: “Alphafetoprotein (AFP), Beta-hCG and PLAP (placental lice isoenzyme of alkaline phosphetase) are the major tumor markers in use for the monitoring of testicular teratoma CA 125, 15-3 and 19-9 are usefil in bowel, pancreatic and ovarian tumours, and carcinoembryonic antigen (CEA) in large bowel tumours, 2/26:2014 3:24:25 AM “Mark this question => Question Id : 9697 Question 1 of 30 A 25-year-old man with a teratoma of the testis attended for review fellowing chemotherapy. Which one of the following serum tumour markers ic of most value in monitoring the clinioal progression of bic disease? a) Carbohydrate antigen CA 15-3 b) Carbohydrate antigen CAA 19-9 Y © oc) Alpha-fetoprotein 4) Carbohydrate antigen CA 125 ©) Carcinoembryonic antigen never [EESRINGHRA) other sors Explanation | Report An Evor Question Explanation: “Alphafetoprotein (AFP), Beta-hCG and PLAP (placental lice isoenzyme of alkaline phosphetase) are the major tumor markers in use for the monitoring of testicular teratoma CA 125, 15-3 and 19-9 are usefil in bowel, pancreatic and ovarian tumours, and carcinoembryonic antigen (CEA) in large bowel tumours, ‘Mark this question —& => Question Td : 10385 Question 2 of 30 ‘Which of the following is the mode of action of Imatinib? 8) Angiegenesis inhibitor ) Epidermal growth factor inhibitor «) Interferon a) Proteosome inhibitor 2) Signal transductace inhibitor Anower [UETHRISERY) other ucersExplanation Report An Error Question Explanation: Tmatinib is a tyrosine kinase inhibiter which blocks the ber/zbl protein from inducing cell growth NICE recommended that imatinib should bs used to treat peoples in the accelerated or blast crisis phase of CML. ‘Mark this question —& => Question Td : 10385 Question 2 of 30 ‘Which of the following is the mode of action of Imatinib? 8) Angiegenesis inhibitor ) Epidermal growth factor inhibitor «) Interferon a) Proteosome inhibitor Y © 2) Signal transductase inhibitor newer (UETHRISEN) other ucersExplanation Report An Error Question Explanation: Tmatinib is a tyrosine kinase inhibiter which blocks the ber/zbl protein from inducing cell growth NICE recommended that imatinib should bs used to treat peoples in the accelerated or blast crisis phase of CML. 2:26/2014 3:24:48 AM Marke this question <& => Question Td : 10827 Question3 of 30 A 66-year-old woman has a month old leftleg pain and weight loss. Examination shows a left dank mass associated with a DVT and further investigations confirm a renal cell carcinoma with lang metastases. Being treated for her DVT with LMWH followed by warfarin, what is the most appropriate treatment for her renal carcinoma’? 4) Platinum besed chemetherepy b) Nephrectomy: ©) Palliative therapy alone 4) Interleukin 2 €) Radiotherapy Question Explanation: ‘This is stage IV renal carcinoma and the prognosis s poor. There is no evidence to support nephrectomy in this situation, but drugs such as interferon and more recently interleukin 2 show promises with reported carcinorca response rates with reduction in tumour bulk of approximately 15%. 2:26/2014 3:24:48 AM Marke this question <& => Question Td : 10827 Question3 of 30 A 66-year-old woman has a month old leftleg pain and weight loss. Examination shows a left dank mass associated with a DVT and further investigations confirm a renal cell carcinoma with lang metastases. Being treated for her DVT with LMWH followed by warfarin, what is the most appropriate treatment for her renal carcinoma’? 2) Platinum based chemotherapy b) Nephrectomy: ©) Palliative therapy alone Y © 4) Interleukin 2 «) Radiotherapy Question Explanation: ‘This is stage IV renal carcinoma and the prognosis s poor. There is no evidence to support nephrectomy in this situation, but drugs such as interferon and more recently interleukin 2 show promises with reported carcinorca response rates with reduction in tumour bulk of approximately 15%. Mark this question & => Question Td : Question 4 of 30 ‘Which of the following statements is correct concerning the relationship between Type 2 Diabetes and colonic cancer? 8) The increased risk of colorectal cancer in diabetes is related to BMT 'b) The increased risk of colorectal cancer in diabetes is related to total cholesterol ©) Insulin treatment increases recurrence-free survival after treatment of colonic cancer 4) Low concentrations of C-peptide indicate low colorectal cancer risk ©) Type 1 diabetes has similar risks of colonic cancer as does type 2 diabetes Answer | Baplanation Other User's Explanation Report An Error Question Explanation: ‘Type 2 Gabetes is associated with a 40-60% increase in the risk of cancer of the large bowel. This increase is linked to changes in HbAlc.Type 2 diabetes is associated with significantly higher sates of overall moctalty and seduced disease free and recurrence free survivals after chemotherapylradiotherapy, and insulin has not been found between colonic malignancy and type | diabetes nor gestational diabetes Mark this question & => Question Td : Question 4 of 30 “Which of the following statements is correct concerning the relationship between Type 2 Diabetes and colonic cancer? 2) The increased risk of colorectal cancer in diabetes is related to EMI +) The increased risk of colorectal cancer in diabetes is related to total cholesterol ©) Insulin treatment increases recurrence-free survival after treatment of colonic cancer ¥ © 4) Low concentrations of C-peptide indicate low colorectal cancer risk, ©) Type 1 diabetes has similar risks of colonic cancer as does type 2 diabetes Answer | Baplanation Other User's Explanation Report An Error Question Explanation: ‘Type 2 Gabetes is associated with a 40-60% increase in the risk of cancer of the large bowel. This increase is linked to changes in HbAlc.Type 2 diabetes is associated with significantly higher sates of overall moctalty and seduced disease free and recurrence free survivals after chemotherapylradiotherapy, and insulin has not been found between colonic malignancy and type | diabetes nor gestational diabetes 2:26/2014 3:25:14 AM Mark this question = => Question Td : 20399 Question 5 of 30 A 28 year old male presents with painless swelling in the testicle A testicular ultrasound confitms the diagnosis of testicular tumour, and an abdominal CT reveals normal retroperitoneal nodes. Which statement is correct with respect to this case? a) Fine-needle biopsy is contraindicated +) His overall prognosis is very poor ©) The testicular mass is likely a secondary tumour 4) A needle aspiration of the tumour is indicated ¢) He will tcely require chemotherapy Answer [UEGIRRRTNY other users Explanation Repos An Feat Question Explanation: ‘Testicular cancer may cause an enlarged testis or a lump elsewhere in the scrotum Physical examination and ultrasound scanning may indicate whether a lump is part of the testis and whether itis sold (and thus more likely to be cancer) or filed with fluid (cystic) Determining the blood levels of two proteins, alpha-fetoprotein and human chorionic gonedetropia, may help in diagnosis, “A biopsy or transerotal approach orchiectomy can cause possible spread of tumor Both a fine-needle biopsy and a transcratal approach are contraindicated. 2:26/2014 3:25:14 AM Mark this question = => Question Td : 20399 Question 5 of 30 A.28 year old male presents with painless swelling in the testicle. A testicular ultrasound confirms the diagnosis of testicular timour, and an abdominal CT reveals normal retroperitoneal aodes. Which statement is correct with respect to this case? Y © a) Fine-needle biopsy is contraindicated +b) His overall prognosis is very poor ©) The testicular mass is ikely a secondary tumour 4) A needle aspiration of the tumour is indicated e) He will likely require chemotherapy Answer [UEGIRRRTNY other users Explanation Repos An Feat Question Explanation: ‘Testicular cancer may cause an enlarged testis or a lump elsewhere in the scrotum Physical examination and ultrasound scanning may indicate whether a lump is part of the testis and whether itis sold (and thus more likely to be cancer) or filed with fluid (cystic) Determining the blood levels of two proteins, alpha-fetoprotein and human chorionic gonedetropia, may help in diagnosis, “A biopsy or transerotal approach orchiectomy can cause possible spread of tumor Both a fine-needle biopsy and a transcratal approach are contraindicated. ‘Mark this question —& => Question Td : 22574 Question 6 of 30 A.15 year old black female presents for a routine physical checkup. On examination, you note a well-defined, nontender nibbery breast mass approximately 2 cm in diameter. No history of skin changes, breast tendemess or nipple discharge is present. The most likely diagnosis is a) Hbrocysiic breast disease b) Benign breast cyst €) Cystosarcoma phyllodes Fbroadencma ¢) Iatraductal papilloma Question Explanation: ‘Most breast masses in adolescent gris are berign. Fibroadenoma is the most common, accounting for approximately two-thirds of all adolescent breast masses. It is characterized by a slow growing, nontender, rubbery, well-defined mass, most commonly located in the upper, cnter quadrant Size varies, and is most commonly in the range of 2-3 cm Fibrocystic disease is foundin older adolescents and is characterized by bilateral nodularity and cyclic tendernsss- Benign breast cysts are characterized by a spongy, tender mass with symptoms exacerbated by menses, Cysts are frequently, muliple, and spontaneous regression occurs in 50% of patients. Cystosarcoma phylodes is a rare tumor with malignant potential, though most are benign. It presents as a frm, mibbery mass the may enlarge rapidly Skin necrosis is usually associated with the tumor Tntrachictal papilloma are usvally benign but do have malignant potential. They are commonly subarcolar and ars associated with nipple discharge. These tumors are rare in the adolescent population. ‘Mark this question —& => Question Td : 22574 Question 6 of 30 A 15 year old black female presents for a routine physical checkup. On examination, younote a well-defined, nontender rubbery breast mass approximately 2 cm in diameter. No history of skin changes, breast tendemess or nipple discharge is present. The most likely diagnesic is a) Hbrocystic breast disease +) Berign breast cyst ©) Cystosarcoma phyliodes Y © & Fibroadenoma 6) Intraductal papilloma Question Explanation: ‘Most breast masses in adolescent gris are berign. Fibroadenoma is the most common, accounting for approximately two-thirds of all adolescent breast masses. It is characterized by a slow growing, nontender, rubbery, well-defined mass, most commonly located in the upper, cnter quadrant Size varies, and is most commonly in the range of 2-3 cm Fibrocystic disease is foundin older adolescents and is characterized by bilateral nodularity and cyclic tendernsss- Benign breast cysts are characterized by a spongy, tender mass with symptoms exacerbated by menses, Cysts are frequently, muliple, and spontaneous regression occurs in 50% of patients. Cystosarcoma phylodes is a rare tumor with malignant potential, though most are benign. It presents as a frm, mibbery mass the may enlarge rapidly Skin necrosis is usually associated with the tumor Tntrachictal papilloma are usvally benign but do have malignant potential. They are commonly subarcolar and ars associated with nipple discharge. These tumors are rare in the adolescent population. 226.2014 3: 42 AM, “Mark this qnestion <=> Question Id : 54239 Question 7 of 30 A siclde cell patient presents with abdominal pain and fever He is given narcotics for pain and IV Suids. Blood cultures are obtained. ‘The appropriate next step in his management is a) Antibiotics b} Hydrosyurea ) Transfision 4) Folic acid Question Explanation: Sickle cell crisis may require hospitalization, The person is given large amounts of fluid intravenously ancl drugs to relieve pin. Blood transfisions and oxygen may be given ifa doctor suspects that anemtia is severe enough to pose a sisk of stroke, heart attack. or lung damage. Conditions thet may have caused the crisis, such as an infection, are treated. Drugs can help control sickle cell disease Hydrozyuea increases the production of a form of hemoglobin found predominantly in fetuses, which decreases the number of red, blood cells becoming sickle-shaped. Therefore, it reduces the frequency of sickle cell crises. n Report An Error

Вам также может понравиться